Tuesday, July 8, 2008

CAT Sample Questions

Verbal Section : Sentence Completion Sample Questions
Directions: Each sentence below has one or two blanks. Each blank shows that something has been omitted. Under each sentence five words are given as choice. Choose the one correct word for each blank that best fits the meaning of the sentences as a whole.
1. The fact that the- of confrontation is no longer as popular as it once was - procatss in race relations.
A. insidiousness - reiterates
B. practice - inculcates
C. glimmer - foreshadows
D. technique - presages
E. reticence - indicates
Ans :D
2. A child should not be - as being either very shy or over - agcatssive.
A. categorized
B. instructed
C. intoned
D. distracted
E. refrained
Ans :A
3. President Anwar el - Sadat of Egypt, disregarding - criticism in the Alab world and in his own Government, - accepted prime minister Menahem Begin's invitation to visit Israel in order to address the Israeli parliament.
A. acrimonious - formally
B. blemished - stiffly
C. categorical - previously
D. malignant - plaintively
E. charismatic - meticulously
Ans :A
4. In his usual - manner, he had insured himself against this type of loss.
A. pensive
B. providential
C. indifferent
D. circumspect
E. caustic
Ans :D
5. We never believed that he would resort to - in order to achieve his goal; we always regarded him as a - man.
A. charm - insincere
B. necromancy - pietistic
C. logic - honorable
D. prestidigitation - articulate
E. subterfuge - honest
Ans :E
6. The Sociologist responded to the charge that her new theory was - by pointing out that it did not in fact contradict accepted sociological principles.
A. unproven
B. banal
C. superficial
D. complex
E. heretical
Ans :E
7. Despite assorted effusion to the contrary, there is no necessary link between scientific skill and humanism, and quite possibly, there may be something of a - between them.
A. dichotomy
B. congruity
C. reciprocity
D. fusion
E. generosity
Ans :E
8. The most technologically advanced societies have been responsible for the catatest - indeed savagery seems to be indirect proposition to -
A. inventions - know-how
B. wars - viciousness
C. triumphs - civilizations
D. atrocities - development
E. catastrophes - ill-will
Ans :D
9. Ironically, the party leaders encountered no catater - their efforts to build as Procatssive Party than the - of the procatssive already elected to the legislature.
A. obstacle to - resistance
B. support for - advocacy
C. praise for - reputation
D. threat to - promise
E. benefit - success
Ans :A
10. The simplicity of the theory - its main attraction - is also its - for only by - the assumptions of the theory is it possible to explain the most recent observations made by researchers.
A. glory - rejecting
B. liability - accepting
C. undoing - supplementing
D. downfall - considering
E. virtue - qualifying
Ans : C
11. That the Third Battalion's fifty percent casually rate transformed its assault on Hill 306 from a brilliant stratagem into a debacle does not - eyewitness reports of its commander's extra-ordinary - in deploying his forces.
A. invalidate - brutality
B. gainsay - cleverness
C. underscore - ineptitude
D. justify - rapidity
E. corroborate -determination
Ans : B
12. No longer - by the belief that the world around us was expressly designed for humanity, many people try to find intellectual - for that lost certainty in astrology and in mysticism.
A. satisfied - reasons
B. reassured - justifications
C. restricted - parallels
D. sustained - substitutes
E. hampered - equivalents
Ans : D
13. In eighth-century Japan, people who - wasteland were rewarded with official ranks as part of an effort to overcome the shortage of - fields.
A. cultivated - domestic
B. located - desirable
C. conserved - forested
D. reclaimed - arable
E. irrigated - accessible.
Ans :D
14. Clearly refuting sceptics, researchers have - not only that gravitational radiation exists but that it also does exactly what the theory- it should do.
A. assumed - deducted
B. estimated - accepted
C. supposed - asserted
D. doubted - warranted
E. demonstrated - predicted.
Ans :E
15. Melodramas, which presented stark oppositions between innocence and criminality, virtue and corruption, good and evil, were popular precisely because they offered the audience a world - of -
A. deprived - polarity
B. full - circumstantiality
C. bereft - theatricality
D. devoid - neutrality
E. composed - adversity.
Ans :D
16. Sponsors of the bill were-because there was no opposition to it within the legislative, until after the measure had been signed into law.
A. well-intentioned
B. persistent
C. detained
D. unreliable
E. relieved.
Ans :B
17. Ecology, like economics, concerns itself with the movement of valuable - through a complex network of producers and consumers.
A. nutrients
B. dividends
C. communications
D. artifacts
E. commodities.
Ans :C
18. Having fully embraced the belief that government by persuasion is preferable to government by - the leaders of the movement have recently - most of their previous statements supporting totalitarianism.
A. proclamation - codified
B. coercion - repudiated
C. participation - moderated
D. intimidation - issued
E. demonstration - deliberated.
Ans :B
19. It would be difficult for one so - to be led to believe that all men are equal and that we must disregard race, color and creed.
A. tolerant
B. democratic
C. broadminded
D. emotional
E. intolerant.
Ans :E
20. Many philosophers agree that the verbal aggression of profanity in certain redical newspapers is not - or childish, but an assault on - essential to the revolutionary's purpose.
A. insolent - sociability
B. trivial - decorum
C. belligerent - fallibility
D. serious - propriety
E. deliberate - affectation.
Ans :B
21. The - tones of the flute succeeded in - his tense nerves.
A. rhapsodic - minimising
B. blatant - enhancing
C. hovendous - calming
D. vibrant - portraying
E. mellifluous - soothing.
Ans :E
22. Without the psychiatrist's promise of confidentiality, trust is - and the patient's communication limited; even though confidentiality can thus be seen to be precious in thercopy, moral responsibility sometimes requires a willingness to - it.
A. lost - forget
B. implicit - extend
C. impaired - sacrifise
D. ambiguous - apply
E. assumed - examine.
Ans :C
23. Parts of seventeenth-century Chinese pleasure gardens were not necessarily intended to look -they were designed expressly to evoke the agreeable melancholy resulting from a sense of the - of natural beauty and human glory.
A. great - immutability
B. joyful - mortality
C. conventional - wildness
D. cheerful - transitoriness
E. colorful - abstractness.
Ans :D
24. Despite the - of many of their colleagues, some scholars have begun to emphasize ''pop culture'' as a key for - the myths, hopes, and fears of contemporary society.
A. pedantry - reinstating
B. enthusiasm - symbolizing
C. skepticism - deciphering
D. antipathy - involving
E. discernment - evaluating.
Ans :C
25. If duty is the natural - of one's the course of future events, then people who are powerful have duty placed on them whether they like it or not.
A. outgrowth - control over
B. arbiter - responsibility for
C. correlate - understanding of
D. determinant - involvement in
E. mitigant - preoccupation with .
Ans :A
26. Clearly refuting sceptics, researches have - not only that gravitational radiation exists but that it also does exactly what the theory - it should do.
A. supposed - asserted
B. voubted -warranted
C. assumed - deduced
D. demonstrated - predicted
E. estimated - accepted
Ans :D
27. The Neolatonists' conception of a deity, in which perfection was measured by abundant fecundity, was contradicted by that of the Aristotelians, in which perfection was displayed in the - of creation.
A. variety
B. economy
C. profusion
D. clarity
E. precision.
Ans :B
28. It is a great - to be able to transfer useful genes with as little extra gene material as possible, because the donor's genome may contain, in addition to desirable genes, many genes with - effects.
A. Disappointment - superfluous
B. Convenience - exquisite
C. Advantage - deleterious
D. Accomplishment - profound
E. Misfortune - unpredictable.
Ans :C
29. While admitting that the risks incurred by use of the insecticide were not - the manufacturer's spokesperson argued that effective - were simply not available.
A. indeterminable - safeguards
B. unusual - alternatives
C. inconsequential - substitutes
D. proven - antidotes
E. increasing - procedures.
Ans :C
30. Human reaction to the realm of though is often as strong as that to sensible presences; our higher moral life is based on the fact that - sensations actually present may have a weaker influence on our action than do ideas of - facts.
A. emotional - impersonal
B. familiar : symbolic
C. disturbing - ordinary
D. material - remote
E. defenitive - controvoisial.
Ans :D
31. Some scientists argue that carbon compounds play such a central role in life on earth because of the possibility of - resulting from the carbon atom's ability to form an unending series of different molecules.
A. variety
B. stability
C. deviations
D. invigorations
E. reproduction.
Ans :A
32. It would be difficult for one so - to be led to believe that all men are equal and that we must disregard race, color and creed.
A. intolerant
B. democratic
C. emotional
D. patient
E. broadminded.
Ans :A
33. An occasional - remark spoiled the - that made the paper memorable.
A. colloquial
B. trite - cliches
C. urbane - sophisticated
D. hackneyed - originality
E. jovial - fun.
Ans :D
34. Broadway audiences have become inured to - and so - to be pleased as to make their ready ovations meaningless as an indicator of the quality of the production before them.
A. cleverness : eager
B. condescension : disinclined
C. sentimentality : reluctant
D. mediocrity : desperate
E. histrionics : unlikely
Ans :D
35. Nineteenth - century scholars, by examining earlier geometric Greek art, found that classical Greek art was not a magical - or a brilliant - blending Egyptian and Assyruin art, but was independently evolved by Greeks in Greece.
A. conversion - annexation
B. apparition - amalgam
C. stratagem - appropriation
D. paradigm - construct
E. example - synthesis
Ans :B
36. The struggle of the generations is one of the obvious constants of human affairs; therefore, it may be presumptuous to suggest that the rivalry between young and old in western society during the current decade is - critical.
A. archetypally
B. perennially
C. disturbingly
D. uniquely
E. cautiously
Ans :D
37. Even though in today's Soviet union the - Muslim clergy have been accorded power and privileges, the Muslim laity and the rank - and - file clergy still. Have little - to practice their religion.
A. adversaries of - inclination
B. traditionalists among - incentive
C. practitioners among - opportunity
D. leaders of - latitude
E. dissidents within -obligation
Ans :D
38. Unlike the Shakespearean plays, The ''closet dramas'' of the nineteenth century were meant to be - rather than -
A. seen - acted
B. read - acted
C. produced - acted
D. quiet - loud
E. sophisticated - urbane
Ans :B
39. The little - known but rapidly expanding use of computers in mapmaking is technologically similar to the more - uses in designing everything from bolts to satellites.
A. ingenuous
B. recent
C. secure
D. publicized
E. successful
Ans :D
40. Although his out numbered troops fought bravely, the general felt he had no choice but to - defeat and - a retreat.
A. oversee - reject
B. acknowledge - order
C. hasten - suggest
D. seek - try
E. overcome - request
Ans :B
41. No hero of ancient or modern times can surpass the Indian with his lofty contempt of death and the - with which he sustained the cruelest coffliction.
A. guide
B. assent
C. reverence
D. fortitude
E. concern
Ans :D
42. The hostess attempted to - a romantic atmosphere that would bring the two young people together in -
A. expand - fealty
B. present - collusion
C. simulate - conflict
D. introduce - cacophony
E. contrive - matrimony
Ans :E
43. Employers who retire people who are willing and able to continue working should realize that - age is not an effective - in determining whether an individual is capable of working.
A. intellectual - criterion
B. Chronological - criterion
C. Physical - barrier
D. deteriorating - value
E. chronological - factor
Ans :B
44. As the sun rose, the morning mists were borne away on the - like strands of -
A. whirlwind - flotsam
B. wind - cactus
C. morass - tundra
D. zephyr - gossamer
E. holocaust - taffeta
Ans :D
45. The playwright was known not for his original ideas that had been propounded by others.
A. rejection
B. consideration
C. invention
D. reiteration
E. plagiarism
Ans :E
46. The gypsy girl, decked out in - finery, and with her disheveled hair streaming over shoulders, was indeed a - sight.
A. verdant - wistful
B. sartorial - flagrant
C. specious - poignant
D. tawdry - bizarre
E. opulent - debonair
Ans :D
47. Yellow fever, the disease that killed 4,000 Philadelphians in 1793, and so - Memphis, Tennessee, that the city lost its charter, has reappeared after nearly two decades in - in the western hemisphere.
A. disabled - quarantine
B. decimated - abeyance
C. terrorized - contention
D. ravaged - secret
E. coupled - quiescence
Ans :B
48. The painting was larger than it appeared to be, for hanging in a darkened recess of the chapel, it was - by the perspective.
A. embellished
B. improved
C. jeopardised
D. aggrandized
E. diminished
Ans :E
49. We have in America - speech that is neither American, Oxford English, nor English but a - of all three.
A. motley - miracle
B. nasal - blend
C. feigned - patchwork
D. mangled - medley
E. hybrid - combination
Ans :E
50. Old beliefs die hard, even when jobs become - the long - standing fear that unemployment could return at a moments notice -
A. protected - subsided
B. vacant - perished
C. available - receded
D. plentiful - persisted
E. easier - charged
Ans :D
51. Not only the - are fooled by pcopagandas we can all be misled if we are not -
A. people - mature
B. ignorant - cynical
C. masses - cautious
D. uncultured - concerned
E. gullible - wary
Ans :E
52. -- merciful by nature, he was - toward the murderer.
A. although - unmoving
B. while - unjust
C. truly - indicative
D. though - kind
E. albeit - implacable
Ans :E
53. When the news of his - with the enemy become known, he was hanged in -
A. collusion - effigy
B. conversation's - earnest
C. involvement - martyrdom
D. complacency - retaliation
E. bickering - response
Ans :D
54. He was so - by the interplay of the colors that varied in brilliance and pattern as the music rose and fell, that he asked the price of the device.
A. overwrought
B. penalized
C. repelled
D. inteugued
E. penalized
Ans :D
55. The absence of a sense of outrage and grief at national tragedy is an - of moral responsibility.
A. intervention
B. energising
C. abdication
D. administration
E. actuation.
Ans :C
56. In an effort to - its operations, the corporation announced it was acquiring a - company in a different type of manufacturing.
A. diversify - subsidiary
B. adumberate - solvent
C. multiply - protracted
D. intensify - fluctuating
E. establish - sequential.
Ans :A
57. Samuel Clemens chose the - Mark Twain as a result of his knowledge of river boat piloting.
A. mountebank
B. protagonist
C. misanthrope
D. hallucination
E. pseudonym.
Ans :E
58. To meet all - a source of - electrical power was added to the train's engine.
A. Integuments - parallel
B. possibilities - incidental
C. amenities - diverse
D. contingencies - auxiliary
E. conveniences - automatic.
Ans :D
59. Since the escaping vapors proved to be highly -, measures were at once taken for the - of the experiments.
A. Volatile - ratification
B. Observable - insulation
C. Gaseous - reduction
D. Noxious - cessation
E. Incriminating - destruction.
Ans :D
60. Eric Fromm does not agree that man is - in Freudian sexual dilemmas for if the - that man creates can be changed for the better, there is hope that the state of man can be changed as well.
A. Tortured - goals
B. Trapped - institutions
C. Caught - symbols
D. Engulfed - life
E. Confused - meanings.
Ans :B
61. As man reached the stars, a booming population threatened to destroy the - of life on his home planet and even its chances for -
A. Quality - survival
B. Basis - growth
C. Existence - upliftment
D. chances - improvement
E. meaning - understanding.
Ans :A
62. Until the current warming trend exceeds the range of normal climatic fluctuations, there will be, among scientists, considerable - the possibility that increasing levels of atmosphere Co2 can cause long term warming effects
A. interest in
B. uncertainty about
C. experimentation on
D. enthusiasm for
E. worry about
Ans :B
63. Having no sense of moral obligation, shipler was as little subject to the - of conscience after he acted as he was motivated by its - before he acted.
A. balm - eloquence
B. qualms - atonement
C. reproaches - prompting's
D. rewards - chastisement
E. ridicule - allure
Ans :C
64. Slang is a language that rolls up its sleeves, spits on its hands and -
A. goes to work
B. stays cool
C. embarrasses its user
D. communicates
E. puts its foot down
Ans :B
65. Famous among job seekers for its - , the company, quite apart from generous salaries, bestowed on its executives annual bonuses and such - as low - interest home mortgages and company cars.
A. largesse - perquisites
B. altruism - credits
C. magnanimity - reparations
D. discernment - prerogatives
E. inventiveness - benefits
Ans :A
66. Moving and parked, the automobile devours urban land, leaving the buildings as mere - of habitable space in a sea of dangerous and ugly traffic
A. asylums
B. remnants
C. blocks
D. mountains
E. islands
Ans :E
67. In the current research program, new varieties of apple trees are evaluated under different agricultural - for tree size, bloom density, fruit size,- to various soils, and resistance to pests and disease.
A. conditions - adaptability
B. configurations - propensity
C. circumstances - proximity
D. auspices - susceptibility
E. regulations - conformity
Ans :A
68. For many young people during the roaring twenties, a disgust with the excesses of American culture - a wanderlust to provoke an exodus abroad.
A. reflected
B. stymied
C. conflicted with
D. overwhelmed
E. combined with
Ans :E Verbal Section: Analogy Sample Questions
Directions:
Each of the questions below consists of two words that have a certain relationship to each other, followed by five lettered pairs of related words. Select the lettered pair of words.
1. ANGLE : DECATE
A. area : square inch
B. milk : quart
C. society : classes
D. letter : alphabet
E. time : minutes
Ans : A
2. CONFIRMED : INVETERATE
A. knowledge : supposed
B. financial : bankrupt
C. immature : callow
D. credible : incredible
E. careful: punishing
Ans :B
3. LULLABY : BARCAROLE
A. birth : marriage
B. night : morning
C. cradle : gondola
D. song : poem
E. carol : sonneteer
Ans :C
4. ZOOLOGY : ANIMALS
A. ecology : pollution
B. botany : plants
C. chemistry : atoms
D. history : people
E. mathematics : geometry
Ans :A
5. DORY : VAN
A. dairy : cow
B. fish : vehicle
C. freighter : caisson
D. runners : wheels
E. danish : Dutch
Ans : C
6. PARQUET : WOOD
A. color : painting
B. mosaic : glass
C. potpourri : medley
D. collage : tapestry
E. linoleum : marble
Ans : B
7. SAW : CARPENTER
A. Scissors : tailor
B. Wagon : farmer
C. Brush : painter
D. Typewriter : author
E. Trowel : bricklayer
Ans : A
8. LURK : WAIT
A. boost : elevate
B. deplete : drain
C. abscond : depart
D. bilk : cheat
E. topple : stabilize
Ans : C
9. ALCHEMY : SCIENCE
A. nostrum : remedy
B. sideshow : carnival
C. ploy : tactic
D. forgery : imitation
E. burlesque : comedy
Ans : A
10. NEEDLE : KNIT
A. bait : fish
B. match : fire
C. loom : weave
D. soap : wash
11. bed : sleep PARENTHESIS : EXPLANATION
A. ellipsis : omission
B. asterisk : exaggeration
C. synopsis : affectation
D. apostrophe : annotation
E. synthesis : interpolation
Ans : A
12. CENSUS : POPULATION
A. manifest : debts
B. roster : audience
C. itinerary : journeys
D. inventory : merchandise
E. state : incumbents
Ans : D
13. STANZA : POEM
A. mimicry : pantomime
B. duet : chorus
C. act : opera
D. rhyme : verse
E. pirouette : ballet
Ans : C
14. EXHORT : SUGGEST
A. conspire : plan
B. tamper : adjust
C. crave : accept
D. goad : direct
E. instruct : teach
Ans : D
15. SAND PAPER : ABRASIVE
A. gasoline : refined
B. grativity : irritant
C. polish : floors
D. acrylic : emulsion
E. oil : lubricant.
Ans :E
16. DIAPHANOUS : CACOPHONOUS
A. translucent : transparent
B. transparent : noisy
C. sheer : opaque
D. harmonious : discordant
E. twofold : multiple.
Ans :B
17. INFANCY : SENILITY
A. january : October
B. incipient : critical
C. day : night
D. conclusion : climax
E. dawn : dusk.
Ans :E
18. RIG : CONTEST
A. solve : conundrum
B. predict : race
C. repudiate : thesis
D. gerrymander : district
E. incriminate : evidence
Ans :D
19. ARBORETUM : TREES
A. aviary : birds
B. catenhouse : garden
C. museum : painters
D. grove : forest
E. zoo : range
Ans :D
20. MENDICANT : IMPECUNIOUS
A. hat : askew
B. liar : poor
C. complainer : petulant
D. critic : quizzical
E. philanthrophist : prodigal.
Ans :C
F.
Ans : C
21. RELAPSE : CONVALESCENCE
A. dissonance : harmony
B. feudalism : industrialization
C. repetition : monotony
D. impasse : debate
E. recidivism : rehavbilitation.
Ans :E
22. BOUQUET : FLOWERS
A. corn : husk
B. woodpile : logs
C. forest : thicket
D. mist : fog
E. drift : snow.
Ans :B
23. TRIANGLE : QUADRILATERAL
A. rectangle : octagon
B. cone : cube
C. pentagon : hexagon
D. plane : solid
E. regular : symmetrical.
Ans :C
24. SARTORIAL : TAILOR
A. thespian : designer
B. rhetorical : questioner
C. pictorial : musician
D. histrionic : singer
E. terpsichorear : dancer.
Ans :E
25. NECROMANCY : GHOSTS
A. magic : legerdemain
B. alchemy : gold
C. sorcery : spirits
D. fortune_telling : gypsies
E. romance : stories.
Ans :C
26. DRUM : TYMPANI
A. piano : orchestra
B. cornet : percussion
C. stick : baton
D. violin : viola
E. oboe : woodwind.
Ans :E
27. EXTROVERT : RETICENT
A. reprobate : humility
B. strategist : decisiveness
C. zealot : loyalty
D. maverick : conformity
E. renegade : ambition.
Ans :D
28. HYGROMETER : BAROMETER
A. snow : rain
B. humidity : pressure
C. water : mercury
D. temperature : weather
E. forecast : rain.
Ans :B
29. EXEMPTION : EXCLUSIONS
A. discharge : elimination
B. debarment : prevention
C. immunity : isolation
D. forgive : condone
E. enclosure : open.
Ans :C
30. FEBRILE : ILLNESS
A. classic : cultivation
B. delusional : insanity
C. eccentric : discrimination
D. tenacious : astonishment
E. juvenile : maturity.
Ans :B

31. DISAPPROBATION : CONDEMN
A. calumny : eulogise
B. enigma : enlighter
C. fallacy : diseminate
D. exhortation : urge
E. solvency : deploy.
Ans :D
32. GEM : TURQUOISE
A. lettuce : green
B. pear : orange
C. stone : magnetta
D. vine : cherry
E. flower : violet.
Ans :E
33. WINE : GRAPES
A. liquor : intoxicating
B. whiskey : hops
C. champagne : raisins
D. vodka : potatoes
E. vineyard : winery.
Ans :D
34. DEBATE : FORENSIC
A. concerto : harmonizing
B. drama : histrionic
C. opera : spoken
D. argument : domestic
E. novel : original.
Ans :B
35. NOISOME : GARBAGE
A. heavy : metal
B. warmth : snow
C. fragrant : incense
D. liquid : perfume
E. loud : music.
Ans :C
36. CONDUIT : WATER
A. behaviour : liquid
B. electricity : television
C. artery : blood
D. wire : sound
E. pump : oil.
Ans :C
37. BIZARRE : EXOTIC
A. wild : tame
B. lively : livid
C. stage : dancer
D. commonplace : routine
E. ordinary : exceptional.
Ans :D
38. ENTREPRENEUR : LABORER
A. mediator : conflict
B. capitalism : communism
C. profits : wages
D. arbitrator : capitalist
E. moonlighting : worker.
Ans :C
39. ANTIMACASSAR : SOFA
A. picture : frame
B. rug : floor
C. pillow : bed
D. door : window
E. table : chair.
Ans :B
40. NOTABLE : NOTORIOUS
A. heinous : atrocious
B. philandering : pleasant
C. philanthropic : miserly
D. nefarious : secret
E. philanthropic : benevolent.
Ans :E
41. BABBLE : TALK
A. though : blank
B. look : espy
C. wink : eye
D. leer : ogle
E. simper : smile.
Ans :E
42. ALCOVE : RECESS
A. column : entrance
B. foundation : building
C. dome : roof
D. turret : chimney
E. foyer : ballroom
Ans :C
43. FIRM : INTRANSIGHT
A. faithful : resolute
B. improvident : industrious
C. vague : inattentive
D. concerned : obsessed
E. malleable : tractable
Ans :D
44. EPAULET : SHOULDER
A. medal : chest
B. decoration : uniform
C. knapsack : back
D. sword : scabbard
E. sash : window
Ans :A
45. ANACHRONISM : CHRONOLOGY
A. tradition : custom
B. variations : incongruity
C. fallacy : logic
D. archetype : paradigm
E. debauchery : appetites
Ans :C
46. DETRITUS : GLACIERS
A. thaw : cold
B. snow : icebergs
C. sediment : bottom
D. silt : rivers
E. dregs : society
Ans :D
47. OUTSKIRTS : TOWN
A. water : goblet
B. margin : page
C. rung : ladder
D. hangar : airplane
E. trunk : tree
Ans :B
48. EQUIVOCATE : COMMITMENT
A. collaborate : falsification
B. fabricate : explanation
C. procrastinate : action
D. expostulate : confusion
E. implicate : exposition
Ans :C
49. MORPHINE : SEDATES
A. oil : smears
B. bandage : protects
C. drug : addicts
D. liquor : sedates
E. medicine : soothes
Ans :D
50. STICKLER : APPROXIMATION
A. Lluggard : indolence
B. connoisseur : anachronism
C. scientist : theorizing
D. leader : guidance
E. purist : adulteration
Ans :E
51. LOOM : WEAVE
A. couch : sleep
B. needle : knit
C. soap : wash
D. machine : stitch
E. bail : fish
Ans :B
52. SUBPOENA : WITNESS
A. hire : laborer
B. tax : worker
C. elect : officer
D. suborn : judge
E. conscript : soldier
Ans :E
53. INVINCIBLE : SUBDUED
A. expensive : bought
B. inconsistent : expressed
C. bolted : separated
D. impervious : damaged
E. imprudent : enacted
Ans :D
54. BURLESQUE : PLAY
A. operetta : symphony
B. limerick : sonnet
C. doggerel : verse
D. table : narration
E. sketch : drawing
Ans :C
55. GROW : BURGEON
A. flourish : thrive
B. transport : enrapture
C. beat : palpitate
D. evolve : multiply
E. wrot : decay
Ans :C
56. HOAX : DECEIVE
A. gimmick : wheedle
B. filibuster : delay
C. boast : cajole
D. lottery : disburse
E. scandal : vilify
Ans :B
57. BODY GUARD : PERSON
A. teacher : pupil
B. mayor : city
C. police officer : traffic
D. soldier : country
E. secretary : office
Ans :D
58. MUFFLE : SOUND
A. conceal : secret
B. assuage : grief
C. maul : object
D. extract : flavor
E. endure : agony
Ans :B
59. CENSORSHIP : INFORMATION
A. cultivation : erosion
B. philanthropy : generosity
C. frugality : constraint
D. sampling : measurement
E. sanitation : disease
Ans :E
60. DUPLICATE : ALTER
A. greet : ignore
B. exchange : return
C. shake : stabilize
D. stretch : shrink
E. eradicate : implicate
Ans :D
61. HYPOTHESIS : EXPERIMENTATION
A. reality : fantasy
B. opinion : debate
C. film : camera
D. predication : conclusion
E. science : success
Ans :B
62. TANGO : DANCE
A. stanza : line
B. tonality : instrumentation
C. arabesque : theme
D. rhyme : pattern
E. elegy : poem
Ans :E
63. CHRONOMETER : SUNDIAL
A. measurement : visibility
B. chronology : analogy
C. computer : abacus
D. watch : ray
E. reduction : enlargement
Ans :C
64. FOOTBALL : GRIDIRON
A. wrestling : mat
B. court : tennis
C. bowling : floor
D. rugby : arena
E. baseball : diamond
Ans :E
65. EXPURGATE : PASSAGE
A. abridge : text
B. filter : water
C. irritate : wound
D. burn : book
E. cancel : plan
Ans :B
66. IMPECUNIOUS : HOVEL
A. progress : prosper
B. mendicant : evasion
C. prosperity : poverty
D. mendacious : cringe
E. affluent : mansion
Ans :E
67. APIARY : BEE
A. museum : painting
B. dam : water
C. arboretum : tree
D. forum : speech
E. planetarium : star
Ans :C
68. BULLET : BARREL
A. fame : films
B. train : track
C. idea : brain
D. plane : clouds
E. water : boat
Ans :B
69. VINDICATE : REPREHENSIBLE.
A. sad : sorrow
B. bitter : sad
C. mild : serious
D. solid : porous
E. vivid : obsequious.
Ans :C
70. TERMAGANT : SHREW
A. virago : harpy
B. anteater : mouse
C. supporter : nag
D. single : married
E. male : female.
Ans :A
71. APOSTATE : RELIGION
A. traitor : country
B. renegade : Indian
C. loyal : faith
D. vital : church
E. diloyal : colonies.
Ans :A
72. PLEBISCITE : UKASE
A. lack : abundance
B. vote : musical instrument
C. cancel : construct
D. public : ruler
E. written : oral
Ans :D
73. DEBATER : LARYNGITIS
A. pedestrian : lameness
B. actor : aplause
C. doctor : diagnosis
D. swimmer : wet
E. writer : paper
Ans :A
74. INKBLOT : EYECHART
A. oculist : ophthalmologist
B. blotter : spectacles
C. psychiatrist : optometrist
D. physician : specialist
E. blurs : letters
Ans :C
75. LIGNEOUS : WOOD
A. cellular : microbe
B. nautical : water
C. igneous : rock
D. osseous : bone
E. fossilized : plant
Ans :D
76. SHRINE : PILGRIM
A. defeat : loser
B. peak : climber
C. rescue : danger
D. election : contestant
E. direction : driver.
Ans :B
77. RIVAL : COMPETITION
A. litigant : morality
B. maverick : co-operation
C. mentor : praise
D. sycophant : flattery
E. medicant : confusion.
Ans :D
78. SPIKE : TACK
A. bullet : wound
B. knife : cut
C. arrow : bow
D. spear : dart
E. pin : needle
Ans :D
79. INIQUITOUS : DISOBEDIENT
A. adult : child
B. hostile : cool
C. quiescent : lethargic
D. inflammable : flammable
E. inequitable : equitable
Ans :B
80. BALEFUL : MENACE
A. brusque : retort
B. competent : achievement
C. placid : boredom
D. flirtatious : affection
E. solicitous : concern
Ans :E
81. ALACRITY : APATHETIC
A. compliance : deft
B. temerity : timid
C. despotism : arrogant
D. candor : bungling
E. tenacity : eager
Ans :B
82. FIRE : STORM
A. whale : minnow
B. speech : shout
C. plant : flower
D. wind : temperature
E. tornado : hurricane
Ans :E
83. COLLUSION : CONSPIRATORS
A. identification : arbitrators
B. co-operation : partners
C. conclusion : messengers
D. revision : corespondents
E. attribution : interpreters
Ans :B
84. LIQUEFY : PETRIFY
A. cash in : strengthen
B. insolvent : bankrupt
C. water : stone
D. soften : frighten
E. solvent : rich
Ans :C
85. AMBULATORY : BEDRIDDEN
A. strong : weak
B. wheelchair : bed
C. free : confined
D. healthy : sick
E. broken : arm
Ans :C
86. CYNOSURE : BRILLIANT
A. word : common
B. student : attentive
C. rock : large
D. magnet : attractive
E. map : legible Ans :D
Reading Comprehension Sample Questions
Directions:
Each reading passage in this section is followed by questions based on the content of the reading passage. Read the passage carefully and chose the best answer to each question. The questions are to be answered on the basis of what is stated or implied in the passage.
1. But man is not destined to vanish. He can be killed, but he cannot be destroyed, because his soul is deathless and his spirit is irrepressible. Therefore, though the situation seems dark in the context of the confrontation between the superpowers, the silver lining is provided by amazing phenomenon that the very nations which have spent incalculable resources and energy for the production of deadly weapons are desperately trying to find out how they might never be used. They threaten each other, intimidate each other and go to the brink, but before the total hour arrives they withdraw from the brink.
1. The main point from the author's view is that
A. Man's soul and spirit can not be destroyed by superpowers.
B. Man's destiny is not fully clear or visible.
C. Man's soul and spirit are immortal.
D. Man's safety is assured by the delicate balance of power in terms of nuclear weapons.
E. Human society will survive despite the serious threat of total annihilation.
Ans : E
2. The phrase 'Go to the brink' in the passage means
A. Retreating from extreme danger.
B. Declare war on each other.
C. Advancing to the stage of war but not engaging in it.
D. Negotiate for peace.
E. Commit suicide.
Ans : C
3. In the author's opinion
A. Huge stockpiles of destructive weapons have so far saved mankind from a catastrophe.
B. Superpowers have at last realized the need for abandoning the production of lethal weapons.
C. Mankind is heading towards complete destruction.
D. Nations in possession of huge stockpiles of lethal weapons are trying hard to avoid actual conflict.
E. There is a Silverlining over the production of deadly weapons.
Ans : D
4. 'Irrepressible' in the second line means
A. incompatible
B. strong
C. oppressive
D. unrestrainable
E. unspirited
Ans : D
5. A suitable title for the above passage is
A. Destruction of mankind is in evitable.
B. Man's desire to survive inhibits use of deadly weapons.
C. Mounting cost of modern weapons.
D. Threats and intimidation between super powers.
E. Cowardly retreat by man
Ans : B
2. Disequilibrium at the interface of water and air is a factor on which the transfer of heat and water vapor from the ocean to the air depends. The air within about a millimeter of the water is almost saturated with water vapor and the temperature of the air is close to that of the surface water. Irrespective of how small these differences might be, they are crucial, and the disequilibrium is maintained by air near the surface mixing with air higher up, which is typically appreciably cooler and lower in water vapor content. The turbulence, which takes its energy from the wind mixes the air. As the speed of wind increases, so does the turbulence, and consequently the rate of heat and moisture transfer. We can arrive at a detailed understanding of this phenomenon after further study. The transfer of momentum from wind to water, which occurs when waves are formed is an interacting-and complicated phenomenon. When waves are made by the wind, it transfers important amounts of energy-energy, which is consequently not available for the production of turbulence.
1. This passage principally intends to:
A. resolve a controversy
B. attempt a description of a phenomenon
C. sketch a theory
D. reinforce certain research findings
E. tabulate various observations
Ans : B
2. The wind over the ocean usually does which of the following according to the given passage?
I. Leads to cool, dry air coming in proximity with the ocean surface.
II. Maintains a steady rate of heat and moisture transfer between the ocean and the air.
III. Results in frequent changes in the ocean surface temperature.
A. I only
B. II only
C. I and II only
D. II and III only
E. I, II, and III
Ans : A
3. According to the author the present knowledge regarding heat and moisture transfer from the ocean to air as
A. revolutionary
B. inconsequential
C. outdated
D. derivative
E. incomplete
Ans : E
4. According to the given passage, in case the wind was to decrease until there was no wind at all, which of the following would occur?
A. The air, which is closest to the ocean surface would get saturated with water vapor.
B. The water would be cooler than the air closest to the ocean surface.
C. There would be a decrease in the amount of moisture in the air closest to the ocean surface.
D. There would be an increase in the rate of heat and moisture transfer.
E. The temperature of the air closest to the ocean and that of the air higher up would be the same.
Ans : A
3. The Food and Drug Administration has formulated certain severe restrictions regarding the use of antibiotics, which are used to promote the health and growth of meat animals. Though the different types of medicines mixed with the fodder of the animals kills many microorganisms, it also encourages the appearance of bacterial strains, which are resistant to anti-infective drugs.
It has already been observed that penicillin and the tetracyclines are not as effective therapeutically as they once used to be. This resistance to drugs is chiefly caused due to tiny circlets of genes, called plasmids, which are transferable between different species of bacteria. These plasmids are also one of the two kinds of vehicles on which molecular biologists depend on while performing gene transplant experiments. Existing guidelines also forbid the use of plasmids, which bear genes for resistance to antibiotics, in the laboratories. Though congressional dabate goes on as to whether these restrictions need to be toughened with reference to scientists in their laboratories, almost no congressional attention is being paid to an ill advised agricultural practice, which produces deleterious effects.
1. In the present passage, the author's primary concern is with:
A. The discovery of methods, which eliminate harmful microorganisms without generating drug-resistant bacteria.
B. Attempting an explanation of the reasons for congressional inaction about the regulation of gene transplant experiments.
C. Portraying a problematic agricultural practice and its serious genetic consequences
D. The verification of the therapeutic ineffectiveness of anti-infective drugs
E. Evaluation of the recently proposed restrictions, which are intended to promote the growth of meat animals.
Ans : C
2. As inferred from the above passage, the mutual transfer of plasmids between different bacteria can result in which of the following?
A. Microorganisms, which have an in-built resistance to drugs
B. Therapeutically useful circlets of genes
C. Penicillin like anti-infective drugs
D. Viruses used by molecular biologists
E. Carriers for performing gene transplant experiments.
Ans : A
3. According to the above passage the author believes that those who favor the stiffening of restrictions on gene transplant research should logically also.
A. Approve and aid experiments with any plasmids except those, which bear genes for antibiotic resistance.
B. Inquire regarding the addition of anti-infective drugs to livestock feeds
C. Oppose the using of penicillin and tetracyclines in order to kill microorganisms
D. Agree to the development of meatier live-stock through the use of antibiotics
E. Approve of congressional debate and discussion regarding science and health issues.
Ans : B
4. The attitude the author has with reference to the development of bacterial strains that render antibiotic drugs in effective can best be described as
A. indifferent
B. perplexed
C. pretentious
D. insincere
E. apprehensive
Ans : E
4. Roger Rosenblatt's book Black Fiction, manages to alter the approach taken in many previous studies by making an attempt to apply literary rather than sociopolitical criteria to its subject. Rosenblatt points out that criticism of Black writing has very often served as a pretext for an expounding on Black history. The recent work of Addison Gayle's passes a judgement on the value of Black fiction by clearly political standards, rating each work according to the ideas of Black identity, which it propounds.
Though fiction results from political circumstances, its author react not in ideological ways to those circumstances, and talking about novels and stories primarily as instruments of ideology circumvents much of the fictional enterprise. Affinities and connections are revealed in the works of Black fiction in Rosenblatt's literary analysis; these affinities and connections have been overlooked and ignored by solely political studies.
The writing of acceptable criticism of Black fiction, however, presumes giving satisfactory answers to a quite a few questions. The most important of all, is there a sufficient reason, apart from the racial identity of the authors, for the grouping together of Black authors? Secondly, what is the distinction of Black fiction from other modern fiction with which it is largely contemporaneous? In the work Rosenblatt demonstrates that Black fiction is a distinct body of writing, which has an identifiable, coherent literary tradition. He highlights recurring concerns and designs, which are independent of chronology in Black fiction written over the past eighty years. These concerns and designs are thematic, and they come form the central fact of the predominant white culture, where the Black characters in the novel are situated irrespective of whether they attempt to conform to that culture or they rebel against it.
Rosenblatt's work does leave certain aesthetic questions open. His thematic analysis allows considerable objectivity; he even clearly states that he does not intend to judge the merit of the various works yet his reluctance seems misplaced, especially since an attempt to appraise might have led to interesting results. For example, certain novels have an appearance of structural diffusion. Is this a defeat, or are the authors working out of, or attempting to forge, a different kind of aesthetic? Apart from this, the style of certain Black novels, like Jean Toomer's Cane, verges on expressionism or surrealism; does this technique provide a counterpoint to the prevalent theme that portrays the fate against which Black heroes are pitted, a theme usually conveyed by more naturalistic modes of expressions?
Irrespective of such omissions, what Rosenblatt talks about in his work makes for an astute and worthwhile study. His book very effectively surveys a variety of novels, highlighting certain fascinating and little-known works like James Weldon Johnson's Autobiography of an Ex-Coloured Man. Black Fiction is tightly constructed, and levelheaded and penetrating criticism is exemplified in its forthright and lucid style.
1. The author of the passage raises and objection to criticism of Black fiction like that by Addison Gayle as it:
A. Highlights only the purely literary aspects of such works
B. Misconceive the ideological content of such fiction
C. Miscalculate the notions of Black identity presented in such fiction
D. Replaces political for literary criteria in evaluating such fiction
E. Disregards the reciprocation between Black history and Black identity exhibited in such fiction.
Ans : D
2. The primary concern of the author in the above passage is:
A. Reviewing the validity of a work of criticism
B. Comparing various critical approaches to a subject
C. Talking of the limitations of a particular kind of criticism
D. Recapitulation of the major points in a work of criticism
E. Illustrating the theoretical background of a certain kind of criticism.
Ans : A
3. The author is of the opinion that Black Fiction would have been improved had Rosenblatt:
A. Undertaken a more careful evaluation of the ideological and historical aspects of Black Fiction
B. Been more objective in his approach to novels and stories by Black authors
C. Attempted a more detailed exploration of the recurring themes in Black fiction throughout its history
D. Established a basis for placing Black fiction within its own unique literary tradition
E. Calculated the relative literary merit of the novels he analyzed thematically.
Ans : E
4. Rosenblatt's discussion of Black Fiction is :
A. Pedantic and contentious
B. Critical but admiring
C. Ironic and deprecating
D. Argumentative but unfocused
E. Stilted and insincere.
Ans : B
5. According to the given passage the author would be LEAST likely to approve of which among the following?
A. Analyzing the influence of political events on the personal ideology of Black writers
B. Attempting a critical study, which applies sociopolitical criteria to the autobiographies of Black authors
C. A literary study of Black poetry that appraises the merits of poems according to the political acceptability of their themes
D. Studying the growth of a distinct Black literary tradition within the context of Black history
E. Undertaking a literary study, which attempts to isolate aesthetic qualities unique to Black fiction.
Ans : C
6. From the following options, which does the author not make use of while discussing Black Fiction?
A. Rhetorical questions
B. Specific examples
C. Comparison and contrast
D. Definition of terms
E. Personal opinion.
Ans : D
7. The author makes a reference to James Weldon Johnson's Autobiography of an Ex-colored Man most probably to:
A. Highlight the affinities between Rosenblatt's method of thematic analysis and earlier criticism
B. Elucidate regarding the point made regarding expressionistic style earlier in the passage
C. Qualify the assessment of Rosenblatt's book made in the first paragraph of the passage
D. Demonstrate the affinities among the various Black novels talked of by Rosenblatt's literary analysis
E. Present a specific example of one of the accomplishments of Rosenblatt's work.
Ans : E
5. Some modern anthropologists hold that biological evolution has shaped not only human morphology but also human behavior. The role those anthropologists ascribe to evolution is not of dictating the details of human behavior but one of imposing constraints - ways of feeling, thinking, and acting that ''come naturally'' in archetypal situations in any culture. Our ''frailties'' - emotions and motivs such as rage, fear, greed, gluttony, joy,lust, love-may be a very mixed assortment quality: we are, as we say, ''in the grip'' of them. And thus they give us oursense of constraints.
Unhappily, some of those frailties our need for ever-increasing security among them are presently maladaptive. Yet beneath the overlay of cultural detail, they, too, are said to be biological in direction, and therefore as natural to us as are our appendixes. We would need to comprehend throughly their adaptive origins in order to understand how badly they guide us now. And we might then begin to resist their pressure.
1. The author implies that control to any extent over the ''frailties'' that constrain our behavior is though to presuppose
A. That those frailties and adaptive are recognized as currently beneficial and adaptive
B. That there is little or no overlay of cultural detail that masks their true nature.
C. That there are cultures in which those frailties do not ''come naturally'' and from which such control can be learned
D. A full understanding of why those frailties evolved and of how they function now
E. A thorough grasp of the principle that cultural detail in human behavior can differ arbitrarily from society to society.
Ans : D
2. It can be inferred that in his discussion of maladaptive frailties the author assumes that
A. Evolution does not favor the emergence of adaptive characteristics over the emergence of maladaptive ones
B. Any structure or behavior not positively adaptive is regarded as transitory in evolutionary theory
C. Maladaptive characteristics, once fixed, make the emergence of other maladaptive characteristics more likely
D. The designation of a characteristic as being maladaptive must always remain highly tentative
E. Changes in the total human environment can outpace evolutionary change.
Ans : E
3. The primary purpose of the passage is to present
A. A position on the foundations of human behavior and on what those foundations imply
B. A theory outlining the parallel development of human morphology and of human behavior
C. A diagnostic test for separating biologically determined behavior patters from culture - specific detail
D. An overview of those human emotions and motive's that impose constraints on human behaviour
E. A practical method for resting the pressures of biologically determined drives.
Ans : A
4. Which of the following most probably provides an appropriate analogy from human morphology for the ''details'' versus ''constraints'' distinction made in the passage in relation to human behaviour?
A. The ability of most people to see all the colors of the visible spectrum as against most peoples inability to name any but the primary colors
B. The ability of even the least fortunate people to show compassion as against people's inability to mask their feelings completely
C. The ability of some people to dive to great depths as against most people's inability to swim long distance
D. The psychological profile of those people who are able to delay gratification as against people's inability to control their lives completely
E. The greater lung capacity of mountain peoples that helps them live in oxygen-poor air as against people's inability to fly without special apparatus.
Ans : E
6. The existence of mammals on the earth can be traced back to at least the Triassic time. The rate of development was retarded, till evolutional change suddenly accelerated in the oldest Paleocene. This resulted in an increase in average size, larger mental capacity, and special adaptations for different modes of life, during the Eocene time. Further improvement was seen during the Oligocene Epoch, with the appearance of some new lines and extinction of others. The Miocene and Pliocene times are especially significant as they mark the culmination of various groups and a continued approach toward modern characters. It is in the Miocene time that the mammals reached their peak with reference to variety and size.
The ability of the mammals to adapt to various modes of life finds a parallel in the reptiles of the Mesozoic time, and apart form their greater intelligence, the mammals apparently have not done much better than the corresponding reptilian forms. Undoubtedly the bat is a better flying animal than the pterosaur, but at the same time the dolphin and whale are hardly more fish like than the ichthyosaur. Quite a few of the swift-running mammals inhabiting the plains, like the horse and the antelope, must excel any of the dinosaurs. Although the tyrannosaur was a more weighty and robust carnivore than perhaps any carnivorous mammal, the lion and the tiger, by virtue of their superior brain are far more efficient and dangerous beasts of prey. It is significant to note that various species of mammals gradually adapted themselves to various kinds of lifestyles, some took to grazing on the plains and were able to run swiftly (horse, deer, bison), others started living in rivers and swamps (hippopotamus, beaver), inhabiting trees (sloth, monkey), burrowing underground (rodent, mole), feeding on flesh (tiger, wolf), swimming in the water (dolphin, whale, seal), and flying in the air (bat). Human beings on account of their superior brain have been able to harness mechanical methods to conquer the physical world and adapt to any set of conditions.
Such adaptation to different conditions leads to a gradual change in form and structure. This is a biological characteristic of the youthful, plastic stage of a group. It is seen that early in its evolutional cycle animals possess the capacity for change, but as the animal progresses in its cycle becoming old and fixed, this capacity for change disappears. The generalized types of organisms retain longest the ability to make adjustments when required, and it is from them that new, fecund stocks take origin-certainly not from any specialized end products. With reference to mammals, we see their birth, plastic spread in many directions, increased specialization, and in some cases, extinction; this is a characteristic of the evolution of life, which can be seen in the geologic record of life.
1. From the following, choose the most appropriate title for the above passage?
A. From Dinosaur to Man
B. Adaptation and Extinction
C. The Superior Mammals
D. The Geologic Life Span
E. Man, the Vanquisher of the Physical World.
Ans : B
2. According to the passage the chronological order of the geologic periods is:
A. Paleocene, Miocene, Triassic, Mesozoic
B. Paleocene, Triassic, Mesozoic, Miocene
C. Miocene, Paleocene, Triassic, Mesozoic
D. Mesozoic, Oligocene, Paleocene, Miocene
E. Mesozoic, Paleocene, Eocene, Miocene
Ans : E
3. From the above passage, we can infer that, the pterosaur
A. resembled the bat
B. was a Mesozoic mammal
C. was a flying reptile
D. inhabited the seas
E. evolved during the Miocene period
Ans : C
4. As inferred from the passage, the largest number of mammals were found in which of the following periods?
A. Triassic period
B. Eocene period
C. Oligocene epoch
D. Pliocene period
E. Miocene period
Ans : E
5. Among the following statements, which statement, if true, would weaken the argument put forth in the first sentence of Paragraph 1?
A. It has been found that the tryannosaur had a larger brain, than was previously known.
B. Within the next thousand years, mammals will become extinct.
C. Recently certain forms of flying ichthyosaurs have been discovered.
D. It has now been proved, that the tiger is more powerful than the carnivorous reptiles.
E. It is now possible to double human mental capacity, by the use of certain recently developed computers.
Ans : A
6. It is clear from the passage, that the evidence used to discuss the life of past time periods
A. was developed by Charles Darwin
B. was unearthed by the author
C. has been negated by more recent evidence
D. was never truly established
E. is based on fossilized remains
Ans : E
7. As inferred from the passage, which of the following proverbial expressions is the author most likely to agree with?
A. It's a cruel world.
B. All the world's a stage.
C. The more things change, the more they remain the same.
D. Footprints in the sands of time.
E. A short life, but a merry one.
Ans : D
7. For a period of more than two centuries paleontologists have been intrigued by the fossilized remains of pterosaurs, the first flying vertebartes. The issues, which puzzle them, are how these heavy creatures, having a wingspan of about 8-12 meters managed the various problems associated with powered flight and whether these creatures were reptiles or birds.
Perhaps the least controversial assertion about the pterosaurs is that they were reptiles. Their skulls, pelvises, and hind feet are reptilian. The anatomy of their wings suggests that they did not evolve into the class of birds. In pterosaurs a greatly elongated fourth finger of each forelimb supported a winglike membrane. The other fingers were short and reptilian, with sharp claws. In birds the second finger is the principal strut of the wing, which consists primarily of feathers. If the pterosaurs walked on all fours, the three short fingers may have been employed for grasping. When a pterosaurs walked or remained stationary, the fourth finger, and with it the wing, could only urn upward in an extended inverted V- shape along each side of the animal's body.
In resemblance they were extremely similar to both birds and bats, with regard to their overall body structure and proportion. This is hardly surprising as the design of any flying vertebrate is subject to aerodynamic constraints. Both the pterosaurs and the birds have hollow bones, a feature that represents a savings in weight. There is a difference, which is that the bones of the birds are more massively reinforced by internal struts.
Although scales typically cover reptiles, the pterosaurs probably had hairy coats. T.H. Huxley reasoned that flying vertebrates must have been warm-blooded because flying implies a high rate of metabolism, which in turn implies a high internal temperature. Huxley speculated that a coat of hair would insulate against loss of body heat and might streamline the body to reduce drag in flight. The recent discovery of a pterosaur specimen covered in long, dense, and relatively thick hair like fossil material was the first clear evidence that his reasoning was correct.
Some paleontologists are of the opinion that the pterosaurs jumped from s dropped from trees or perhaps rose into the light winds from the crests of waves in order to become airborne. Each theory has its associated difficulties. The first makes a wrong assumption that the pterosaurs hind feet resembled a bat's and could serve as hooks by which the animal could hang in preparation for flight. The second hypothesis seems unlikely because large pterosaurs could not have landed in trees without damaging their wings. The third calls for high aces to channel updrafts. The pterosaurs would have been unable to control their flight once airborne as the wind from which such waves arose would have been too strong.
1. As seen in the above passage scientists generally agree that:
A. the pterosaurs could fly over large distances because of their large wingspan.
B. a close evolutionary relationship can be seen between the pterosaurs and bats, when the structure of their skeletons is studied.
C. the study of the fossilized remains of the pterosaurs reveals how they solved the problem associated with powered flight
D. the pterosaurs were reptiles
E. Pterosaurs walked on all fours.
Ans : D
2. The view that, the pterosaurs rose into light winds from the crest of the waves to become airborne, is viewed by the author as
A. revolutionary
B. unlikely
C. unassailable
D. probable
E. outdated.
Ans : B
3. As inferred from the passage, the skeleton of a pterosaur is distinguishable from that of a bird by the
A. length of its wingspan
B. hollow spaces in its bones
C. anatomic origin of its wing strut
D. evidence of the hooklike projections on its hind feet
E. location of the shoulder joint joining the wing to its body.
Ans : C
4. From the viewpoint of T.H.Huxley, as given in the passage, which of the following statements is he most likely to agree with?
A. An animal can master complex behaviors irrespective of the size of it's brain.
B. Environmental capabilities and physical capabilities often influence the appearance of an animal.
C. Usually animals in a particular family group do not change their appearance dramatically over a period of time
D. The origin of flight in vertebrates was an accidental development rather than the outcome of specialization or adaption
E. The pterosaurs should be classified as birds, not reptiles.
Ans : B
5. According to the passage which of the following is a characteristic of the pterosaurs?
A. The pterosaurs were not able to fold their wings when not in use
B. Like the bats, they hung upside down from branches
C. They flew in order to capture prey
D. They can be said to be an earlier stage in the evolution of the birds
E. They lived principally in a forest like habitat.
Ans : A
6. The organization of the last paragraph of the passage can best be described as:
A. New data is introduced in order to support a traditional point of view
B. Three explanations are put forth and each of them is disputed by means of specific information
C. An outline of three hypotheses are given and evidence supporting each of them is given
D. Description of three recent discoveries is presented, and their implications for future study are projected
E. The material in the earlier paragraphs is summarized and certain conclusions are from it.
Ans : B
7. According to the passage, some scientists believe that pterosaurs
A. Lived near large bodies of water
B. Had sharp teeth for tearing food
C. Were attacked and eaten by larger reptiles
D. Had longer tails than many birds
E. Consumed twice their weight daily to maintain their body temperature.
Ans : A
8. Certain scraps of evidence bear out those who hold a very high opinion of the average level of culture among the Athenians of the great age. Pericles's funeral speech is undoubtedly the most famous evidence from Athenian literature, that its level was indeed high. However, Pericles was a politician, and it is possible that he was flattering his audience. We know that thousands of Athenians sat hour after hour in the theater listening to the plays of the great Greek dramatists. The Greek plays, particularly the tragedies, maintained an extremely high intellectual level throughout, with no letdowns, no concessions to the lowbrows or to the demands of ''realism'', like the gravediggers scene in Shakespeare's Hamlet. The music and dancing seen in these plays were also of an equally high level. The best modern parallel can be seen in the restrained, difficult opera of the 18th century. The comparison is no doubt dangerous, but can you imagine almost the entire population of an American city (in suitable installments, of course) sitting through performances of Mozart's Don Giovanni or Gluck's Orpheus? Perhaps the Athenian masses went to these plays because of a lack of other amusements. They could at least understand something of what went on, since the subjects were part of their folklore. Undoubtedly the theme of grand opera is not part of the folklore of the American people.
1. From the passage it is evident that the author seems to question the sincerity of
A. politicians
B. playwrights
C. opera goers
D. ''low brows''
E. gravediggers.
Ans : A
2. According to the author the average American
A. Enjoys Hamlet
B. Loves folklore
C. Is not able to understand grand opera
D. Seeks a high cultural level
E. Lacks entertainment.
Ans : C
3. From the passage, we can say that the author's attitude toward Greek plays is one of
A. Qualified approval
B. Grudging admiration
C. Studied indifference
D. Partial hostility
E. Great respect.
Ans : E
4. The author makes a suggestion that Greek plays
A. Were demanding on the actors
B. Flattered their audiences
C. Were focussed on a limited audience
D. Were dominated by music and dancing
E. Stimulated their audiences.
Ans : E
9. Everyone conforms to infancy, infancy conforms to nobody, so that one babe commonly makes four or five out of the adults who prattle and play to it. So God has armed youth and puberty and manhood no less with its own piquancy and charm, and made it enviable and gracious and its claims not to be put by, if it will stand by itself. Do not think the youth has no force, because he cannot speak to you and me. Hark! In the next room his voice is sufficiently clear and emphatic. It seems he knows how to speak to his contemporaries. Bashful or bold, then, he will know how to make us seniors very unnecessary.
The healthy attitude of human nature can be seen in the nonchalance of boys who are sure of a dinner, and would disdain as much as a lord to do or say aught to conciliate one. A boy is in the parlor what the pit is in the playhouse; independent, irresponsible, looking out from his corner on such people and facts as pass by, he tries and sentences them on their merits, in the swift, summary way of boys, as good, bad, interesting, silly, eloquent, troublesome. He never cumbers himself regarding consequences, about interests and he gives an independent, genuine verdict. You should court him: he will not court you. But the man is, as it were, clapped into jail by his consciousness. As soon as he has once acted or spoken with eclat, he is a committed person, watched by the sympathy or the hatred of hundreds, whose affections must now enter into his account. There is no Lethe for this. Ah, that he could pass again into his neutrality.
These are the voices, which we hear in solitude, but they grow faint and inaudible as we enter into the world. Everywhere society is conspiring against the manhood of every one of its members. Society is joint – stock company, in which members agree, for the better securing of his bread to each shareholder, to surrender the liberty and culture of the eater. The virtue in most request is conformity. It is averse to self-reliance. What it loves is names and customs and not realities and creators.
Whosoever is a man has to be a nonconformist. He who would gather immortal palms must not be hindered by the name of goodness, but must explore if it be goodness. Nothing is at last sacred but the integrity of your own mind.
No law can be sacred to me but that of my nature. Good and bad are but names very readily transferable to that to this; the only right is what is after my constitution, the only right is what is after me constitution, the only wrong what is against it. A man is to carry himself in the presence of all opposition as if every thing were titular and ephemeral but he. I am ashamed to think how easily we capitulate to badges and names, to large societies and dead institutions. Every decent and well-spoken individual affects and sways me more than is right. I ought to go upright and vital, and speak the rude truth in all ways.
I shun father and mother and wife and brother, when my genius calls me. I would write on the lintels of the doorpost, whim. I hope it is somewhat better than whim at last, but we cannot spend the day in explanation. Except me not to show cause why I seek or why I exclude company. Then, again, do not tell me, as a good man did not to-day, of my obligation to put all poor men in good situations. Are they my poor? I tell thee, thou foolish philanthropist, that I grudge the dollar, the time, the cent, I give to such men as do not belong to me and to whom I do not belong. There is a class of person to whom by all spiritual affinity I am bought and sold; for them I will go to prison, if need be; but your miscellaneous popular charities; the education at collage of fools; the building of meeting – house to the vain end to which many now stand; alms to sots; and the thousandfold Relief Societies; - though I confess with shame I sometimes succumb and give the dollar, it is a wicked dollar which by and by I shall have the manhood to withhold.
If you refuse to conform, you can experience the displeasure of the world. Hence, a man should know how to estimate a sour face. The by – standers look askance on him in the public street or in the friend's parlor. In case this aversion originates from contempt and resistance similar to his own, it might result in a sad countenance; but the sour faces of the multitude, like their sweet faces, have no deep cause, but are caused by reasons as diverse as the direction of the wind and what he reads in the newspapers. Yet is the discontent of the multitude more formidable than that of the senate and the collage.
Another factor, which frightens us from self – trust in our consistency; a reverence for our past act or word, because the eyes of others have no other data for computing our orbit than our past acts, and we are loath to disappoint them.
But why should you keep your head over your shoulder? Why drag about this corpse of your memory, lest you contradict somewhat you have stated in this or that public place? Suppose you should contradict yourself; what then?
This is a rather silly consistency in our minds, which is adored by little statesmen and philosophers and divines. Uniformly a great soul has almost nothing to do, he could just occupy himself with his shadow on the wall. Speak what you think now in hard words; and to-morrow speak what tomorrow thinks in hard words again, though it contradict everything you said to-day. – ''Ah, so you shall be sure to be misunderstood.'' - Is it so bad, then, to be misunderstood? Pythagoras was misunderstood, and Socrates, and Jesus, and Luther, and Copernicus, and Galileo, and Newton, and every pure and wise spirit that ever took flesh. What can be considered to be truly great is to be misunderstood.
1. Which of the following statements would best describe the main theme of the above passage?
A. "A foolish consistency is the hobgoblin of little mind."
B. "Eternal youth means eternal independence."
C. "Whoso would be a man must be a nonconformist."
D. "Colleges are designed to educate fools."
E. "Infancy conforms to nobody."
Ans : C
2. When is the period during which we are most nonconformist?
A. infancy
B. puberty
C. youth
D. manhood
E. old age
Ans : A
3. In his statement ''What can be considered to be truly great is to be misunderstood'' the author means:
A. One should refrain from saying, what one exactly means
B. Being misunderstood, equals being great
C. All great man have always been misunderstood
D. Even though a person might be considered inconsistent, he shouldn't hesitate to change his mind if he feels the need to.
E. It is seldom, that nice people succeed
Ans : D
4. As inferred from the passage, the refusal of young people to cater to accept public opinion is:
A. A feature of the rebelliousness of youth
B. A healthy attitude of human nature
C. A manifestation of deep- seated immaturity
D. Simply bad manners
E. Part of growing up
Ans : B
5. "Society is a joint-stock company etc." is one way which the author shows
A. The anti-culture attitude of the public
B. Society is highly organized and structured
C. The self-rejection of society
D. The lack of room for solitude in our world
E. The public's interest in the stock market
Ans : C
6. " I would write on the lintels of the doorpost, whim." What does the author mean by this statement:
A. That one should renounce his immediate family
B. That signposts have an important educational function in our society’
C. That an impulsive action may have a subsequent rational explanation
D. That one must never be held responsible for what one says and does
E. That everyone should do foolish things occasionally
Ans : C
7. Which of the following statements best summarizes the spirit and sense of the above passage?
A. "Nothing is at last sacred but the integrity of your own mind."
B. "With consistency, a great soul; has simply nothing to do."
C. "Do not think the youth has no force, because cannot speak to you and me."
D. "The virtue in most request is conformity."
E. "A man must know how to estimate a sour force."
Ans : A
10. Furthermore, insofar as any conclusion about its author can be drawn from five or six plays attributed to him, the Wakefield Master is without exception considered to be a man of sharp contemporary observation. He was, probably clerically educated, as indicated by his Latin and music, his Biblical and patristic lore. Even today he is remembered for his his quick sympathy for the oppressed and forgotten man, his sharp eye for character, a ready ear for colloquial, vernacular turns of speech and a humor alternately rude and boisterous, coarse and happy. Therefore in spite of his conscious artistry as can be seen in his feeling for intricate metrical and stanza forms, he is regarded as a kind of medieval Steinbeck, indignantly angry at, uncompromisingly and even brutally realistic in presenting the plight of the agricultural poor.
It is now fairly accepted to regard the play as a kind of ultimate point in the secularization of the medieval drama. Therefore more stress has been laid on it as depicting realistically humble manners and pastoral life in the bleak of the west riding of Yorkshire on a typically cold night of December 24th. After what are often regarded as almost ''documentaries'' given in the three successive monologues of the three shepherds, critics go on to affirm that the realism is then intensified into a burlesque mock-treatment of the Nativity. Finally as a sort of epilogue or after-thought in deference to the Biblical origins of the materials, the play slides back into an atavistic mood of early innocent reverence. In actuality, the final scene is the culminating scene and also the raison d’etre of the introductory ''realism.''
Superficially the present play supports the conventional view of its mood of secular realism. At the same time, the ''realism'' of the Wakefield Master is of a paradoxical turn. His wide knowledge of people, as well as books indicates no cloistered contemplative but one in close relation to his times. Still, that life was after all a predominantly religious one, a time which never neglected the belief that man was a rebellious and sinful creature in need of redemption . So deeply (one can hardly say ''naively'' of so sophisticated a writer) and implicitly religious is the Master that he is less able (or less willing) to present actual history realistically than is the author of the Brome Abraham and Isaac. His historical sense is even less realistic than that of Chaucer who just a few years before had done for his own time ''costume romances,'' such as The Knight's Tele, Troilus and Cressida, etc. Furthermore, used highly romantic materials, which could excuse his taking liberties with history.
1. Of the following statements, which is not true of Wakefield Master?
A. He and Chaucer were contemporaries.
B. Wakefield Master is remembered as having written five or six realistic plays.
C. His plays realistically portray the plight of the country folk of his day
D. His writing was similar to that of John Steinbeck.
E. He was an accomplished artist.
Ans : D
2. The word 'patristic' in the first paragraph is used to mean:
A. patriotic
B. superstitious
C. folk
D. relating to the Christian Fathers
E. realistic
Ans : D
3. The statement about the ''secularization of the medieval drama'' (opening sentence of the second paragraph) refers to the
A. Introduction of religious themes in the early days
B. Presentation of erudite material
C. Use of contemporary materials
D. Return to early innocent reverence at the end of the play
E. Introduction of mundane matters in religious plays
Ans : E
4. From the following what would the writer be expected to do in the subsequent paragraphs:
A. Make a justification for his comparison with Steinbeck
B. Put forth a view point, which would take up the thought of the second paragraph
C. Point out the anachronisms in the play
D. Discuss the works of Chaucer
E. Talk about the lack of realism in the works of the Wakefield Master.
Ans : B
11. The establishment of the third Reich influenced events in American history by starting a chain of events which culminated in war between Germany and the United States. The complete destruction of democracy, the persecution of laws, the war on religion, the cruelty and barrbarism of the Nazis and especially, the plans of Germany and her allies, Italy and Japan, for world conquest caused great indignation in this country and brought on fear of another world war. While speaking out against Hitler's atrocities, the American profile generally favored isolationist policies, and neutrality. The neutrality acts of 1935 and 1936 prohibited trade with any belligerents or loans to them. In 1937 the president was empowered to declare an arms embargo in wars between nations at his discretion
American opinion began to change somewhat after President Roosevelt's quarantine the aggvessor speech at Chicago (1937) in which he severely criticized Hitler's policies. Germany's seizure of Austria and Munich pact for the partition of Czechoslovakia (1938) also around the American people. The conquest of Czechoslovakia in March 1939 was another rude awakening to the menace of the third Reich. In August, 1939, came the shock of the Nazi - Soviet pact and in September the attack on Poland and the outbreak of European war. The United States attempt to maintain neutrality in spite of sympathy for the democracies arranged against the Third Reich. The Neutrality act of 1939 repeated the arms embargo and permitted 'cash' and 'carry' exports of arms to belligerent nations. A strong national defense program was begun. A draft act was passed (1940) to strengthen the military services. A Lend - Lease Act (1940) authorized the president to sell, exchange or lend materials to any county deemed necessary by him for the defense of the United States. Help was given to Britain territory in the western Hemisphere. In August 1941, President Roosevelt and prime minister Churchill met and issued the Atlantic Charter which proclaimed the kind of a world which should be established after the war. In December 1941, Japan launched the unprovoked attack on the United States at Pearl harbor, immediately thereafter Germany declared war on the united states.
1. USA entered the war against Germany
A. because Pearl Harbor was attacked
B. after peaceful efforts had failed
C. because Germany declare war against it
D. because Japan was an ally of Germany
E. after Germany had signed the Nazi - Soviet pact
Ans : C
2. The Neutrality Act of 1939 favored Great Britain because
A. the British had command of the sea
B. the law permitted U.S.A. to trade only with the allies.
C. it antagonized Japan
D. it led to the Land - Lease Act
E. it agreed with the British on the principle of the Atlantic Charter
Ans : A
3. An event that did not occur in 1939 was the
A. invasion of Poland
B. invasion of Czechoslovakia
C. passing of the Neutrality Act
D. passing of the Land - Lease act
E. outbreak of the war in Europe
Ans : D
4. One item occurring 1937 that the author does not mention in the list of actions that alienated the American Public was
A. The persecution of religious groups
B. Nazi barbarism
C. The pacts with Italy
D. German plans for conquest of the world
E. The burning of the Reich tag.
Ans : E
5. The Land - Lease Act has designed to
A. Strengthen USA's national defense
B. Provide battle shit to the Allies
C. Help the British
D. the Atlantic Charter
E. Avenge Pearl Harbor
Ans : A
6. The Neutrality Act of 1939
A. restated America's isolationist policies
B. proclaimed American neutrality
C. permitted the selling of arms to belligerent nation
D. was cause of USA's entrances in to WORLD WAR II
E. started USA's national defense programs
Ans : C
7. During the years 1933-36, American policy may be described as having been
1. watchful
2. isolationist
3. pacific
4. incorrect
5. discretionary
Ans : B
12. There was in increase of about 10 % in the investment in the public sector, like electricity, irrigation quarrying, public services and transport; even though the emphasis leaned towards transport and away from the other sectors mentioned. A 16-17% growth in investment, including a 30% increase in investment in business premises has been recorded in trade and services. Although there continued to be a decline in the share of agriculture in total gross investment in the economy, investment grew by 9% in absolute terms, largely spurred on by a 23% expansion of investment in agriculture equipment. Housing construction had 12% more invested in it in 1964, not so much owing to increase demand, as to fears of impending new taxes and limitation of building.
There was a rise of close to 11% in the total consumption in real terms during 1964 and per capita personal consumption by under 7%, as in 1963. The undesirable trend towards a rapid rise in consumption, evident in previous years, remains unaltered. Since at current prices consumption rose by 16% and disposable income by 13%, there was evidently a fall in the rate of saving in the private sector of the economy. Once again a swift advance in the standard of living was indicated in consumption patterns. Though fruit consumption increased, expenditure on food, especially bread and staple items, declined significantly. There was a continuing increase in the outlay on furniture and household equipment, health, education and recreation. The greatest proof of altered living standards was the rapid expansion of expenditure on transport (including private cars) and personal services of all kinds, which occurred during 1964. The changing composition if purchased durable goods demonstrated the progressive affluence of large sectors of the public. On the one hand increased purchase of automobiles and television sets were registered, a point of saturation was rapidly being approached for items like the first household radio, gas cookers, and electric refrigerators.
1. It is possible to to conclude from this passage, that the people of the country were
A. spending more money than they earn
B. investing and consuming at an accelerated pace
C. saving more money than previously
D. spending their money wisely
E. lacking in necessities
Ans : B
2. According to the author the trend towards a rapid rise in consumption is "undesirable" as:
A. there was an increase in the expenditure on frills and luxuries
B. the people were affluent
C. there was a rise in the standard of living
D. people were eating less
E. people were saving less
Ans : E
3. It is possible to conclude that the United States is not the discussed country as:
A. there was a decline in the expenditures for food
B. From the statement that the saturation point was rapidly being approached for first household radios
C. there is no mention of military expenditures
D. the people were affluent
E. the people were not saving their money
Ans : B
4. The area, which saw the greatest expenditure of investment funds was
A. The public sector
B. Business premises
C. Housing construction
D. Agricultural equipment
E. A field which cannot be determined
Ans : E
13. Visual recognition involves storing and retrieving memories. Neural activity, triggered by the eye, forms an image in the brains memory system that constitutes an internal representation of the viewed object. When an object is encountered again, it is matched with its internal representation and thereby recognized. Controversy surrounds the question of whether recognition is a parallel, one-step process or a serial, step-by-step one. Psychologists of the Gestalt school maintain that object are recognized as wholes in a parallel procedure : , the internal representation is matched with the retinal image in a single operation. Other psychologists have proposed that internal representation features are matched serially with an object's features. Although some experiments show that, as an object become familiar, its internal representation becomes more familiar, its internal representation becomes more holistic and the recognition process correspondingly more parallel, the weight of evidence seems to support the serial hypothesis, at least for objects that are not notably simple and familiar.
1. It can be inferred from the passage that the matching process in visual recognition is
A. Not a natural activity.
B. Not possible when an object is viewed for the very first time.
C. Not possible if a feature of a familiar object is changed in same way.
D. Only possible when a retinal image is received in the brain as a unitary whole.
E. Now fully understood as a combination of the serial and parallel process.
Ans : A
2. In terms of its tone and form, the passage can best be characterized as
A. A biased exposition
B. A speculative study
C. A dispassionate presentation
D. An indignant denial
E. A dogmatic explanation.
Ans : C
3. The author is primarily concerned with
A. Explaining how the brain receives images
B. Synthesizing hypotheses of visual recognition
C. Examining the evidence supporting the serial recognition hypothesis
D. Discussing visual recognition and some hypotheses proposed to explain it.
E. Reporting on recent experiments dealing with memory systems and their relationship to neural activity.
Ans : B
4. According to the passage, Gestalt psychologists make which of the following suppositions about visual recognition?
I A retinal image is in exactly the same form as its internal representation
II An object is recognized as a whole without any need for analysis into component parts.
III The matching of an object with its internal representation occurs in only one step
A. II only
B. III only
C. I and III only
D. II and III only
E. I, II and III
Ans : D
14. According to Albert Einstein the non mathematician, is seized by a mysterious shuddering when he hears of 'four-dimensional' things, he is seized by a feeling, which is very similar to the thoughts awakened by the occult. And at the same time the statement that the world in which we live is a four-dimensional space - time continuum is quite a common place statement.
This might lead to an argument regarding the use of the term ''commonplace'' by Einstein. Yet the difficulty lies more in the wording than the ideas. Einstein's concept of the universe as a four-dimensional space-time continuum becomes plain and clear, when what he means by ''continuum'' becomes clear. A continuum is something that is continuous, A ruler, for example, is a one-dimensional space continuum. Most rulers are divided into inches and fractions, scaled down to one-sixteenth of an inch.
Will it be possible to conceive a ruler, which is calibrated to a millionth or billionth of an inch. In theory there is no reason why the steps from point to point should not be even smaller. What distinguishes a continuum is the fact that the space between any two points can be sub-divided into an infinite number of smaller divisions.
A railroad track is a one-dimensional space continuum and on it the engineer of a train can describe his position at any time by citing a single co-ordinate point - i.e., a station or a milestone. A sea captain, however, has to worry about two dimensions. The surface of the sea is a two-dimensional continuum and the co-ordinate points by which sailor fixes his positions in his two dimensional continuum are latitude and longitude. An airplane pilot guides his plane through a three - dimensional continuum, hence he has to consider not only latitude and longitude, but also his height above the ground. The continuum of an airplane pilot constitutes space as we perceive it. In other words, the space of our world is a three-dimensional continuum.
Just indicating its position in space is not enough while describing any physical event, which involves motion. How position changes in time also needs to be mentioned. Thus to give an accurate picture of the operation of a New York - Chicago express, one must mention not only that it goes from New - York to Albany to Syracuse to Cleveland to Toledo to Chicago, but also the times at which it touches each of those points. This can be done either by means of a timetable or a visual chart. If the miles between New York and Chicago are plotted horizontally on a piece of ruled paper and the hours and minutes are plotted vertically, then a diagonal line properly drawn across the page illustrates the progress of the train in two - dimensional space - time continuum. This type of graphic representation is familiar to most newspaper readers; a stock market chart, for example, pictures financial events in a two - dimensional dollar - time continuum. Similarly for the best picturization of the flight of an airplane from New York to Los Angeles a four - dimensional space - time continuum is essential. The latitude, longitude and altitude will only make sense to the traffic manager of the airline if the time co - ordinate is also mentioned. Therefore time is the fourth dimension. If a flight has to be looked at, perceived as a whole, it wouldn't work if it is broken down into a series of disconnected take - offs, climbs, glides, and landing, it needs to be looked at and perceived as a continuous four - dimensional space - time continuum curve.
1. In order to explain a difficult topic, the author use
A. Simply phrased definition's
B. An incessant metaphor
C. A plain writing style
D. Familiar images
E. A quotation from Einstein
Ans : D
2. The significant feature of a continuum, according to the passage, revolves around
A. The divisibility of the interval between any two points.
B. An ordinary ruler's caliber for marking
C. Its unending curve
D. Its lucid from providing comprehensibility to the non - scientists as well
E. Its variety of co - ordinates.
Ans : A
3. The purpose of this passage is to highlight the point that
A. Plots and sea captains have something in common
B. Stock market charts may be helpful to physicists
C. The fourth dimension is time.
D. Non - mathematician's are often afraid of the commonplace
E. There is a marked quality to distance
Ans : C
4. According to the passage, an airlines traffic manager depends upon all of the following EXCEPT
A. latitude
B. altitude
C. the time co - ordinate
D. longitude
E. the continuous curve in co four
Ans : E
5. The underlying tone of this selection is
A. persuasive
B. deferential
C. candid
D. instructive
E. gently condescending
Ans : D
6. According to the author if on wishes portray a physical event in which motion plays a role - one has to
A. Make use of a time-table
B. Indicate how position changes in time
C. Be conversant with the scientist's theories
D. Describe it graphically
E. Be aware of altitude, latitude and longitude
Ans : B
7. The sea-captain's example has been cited in order to
A. Help understand a two - dimensional continuum
B. Set up a logical progression
C. Simplify what ever is too elaborate
D. Mitigate the gap between the engineer and pilot
E. To sustain out interest in the reading of the passage.
Ans : A
15. From the 197 million square miles, which make up the surface of the globe, 71 per cent is covered by the interconnecting bodies of marine water; the Pacific Ocean alone covers half the Earth and averages near 14,000 feet in depth. The portions which rise above sea level are the continents-Eurasia, Africa; North America, South America, Australia, and Antarctica. The submerged borders of the continental masses are the continental shelves, beyond which lie the deep-sea basins.
The ocean are deepest not in the center but in some elongated furrows, or long narrow troughs, called deeps. These profound troughs have a peripheral arrangement, notably around the borders of the pacific and Indian oceans. The position of the deeps, like the highest mountains, are of recent origin, since otherwise they would have been filled with waste from the lands. This is further strengthened by the observation that the deeps are quite often, where world-shaking earthquakes occur. To cite an example, the "tidal wave" that in April, 1946, caused widespread destruction along Pacific coasts resulted from a strong earthquake on the floor of the Aleutian Deep.
The topography of the ocean floors is none too well known, since in great areas the available soundings are hundreds or even thousands of miles apart. However, the floor of the Atlantic is becoming fairly well known as a result of special surveys since 1920. A broad, well-defined ridge-the Mid-Atlantic ridge-runs north and south between Africa and the two Americas and numerous other major irregularities diversify the Atlantic floor. Closely spaced soundings show that many parts of the oceanic floors are as rugged as mountainous regions of the continents. Use of the recently perfected method of submarine topography. During world war II great strides were made in mapping submarine surfaces, particularly in many parts of the vast Pacific basin.
Most of the continents stand on an average of 2870 feet above sea level. North America averages 2300 feet; Europe averages only 1150 feet; and Asia, the highest of the larger continental subdivisions, averages 3200 feet. Mount Everest, which is the highest point in the globe, is 29,000 feet above the sea; and as the greatest known depth in the sea is over 35,000 feet, the maximum relief (that is, the difference in altitude between the lowest and highest points) exceeds 64,000 feet, or exceeds 12 miles. The continental masses and the deep-sea basins are relief features of the first order; the deeps, ridges, and volcanic cones that diversify the sea floor, as well as the plains, plateaus, and mountains of the continents, are relief features of the second order. The lands are unendingly subject to a complex of activities summarized in the term erosion, which first sculptures them in great detail and then tends to reduce them ultimately to sea level. The modeling of the landscape by weather, running water, and other agents is apparent to the keenly observant eye and causes thinking people to speculate on what must be the final result of the ceaseless wearing down of the lands. Much before there was any recognizable science as geology, Shakespeare wrote "the revolution of the times makes mountains level."
1. The peripheral furrows or deeps are found
A. only in the pacific and Indian oceans
B. near earthquakes
C. near the shore
D. in the center of the ocean
E. to be 14,000 feet in depth in the pacific.
Ans : C
2. The largest ocean is the
A. Atlantic
B. pacific
C. Aleutian deep
D. arctic
E. Indian.
Ans : B
3. We may conclude from this passage that earth quakes
A. Occur more frequently in newly formed land or sea formations
B. Are caused by the weight of the water
C. Cause erosion
D. Occur in the deeps
E. Will ultimately "make mountains level".
Ans : A
4. The highest mountains are
A. oldest
B. in excess of 12 miles
C. near the deeps
D. relief features of the first order
E. of recent origin.
Ans : E
5. The science of geology was started
A. By the Greeks
B. During world war II
C. April 1946
D. After 1600
E. In 1920
Ans : D
6. The highest point on North America is
A. 2870 feet above sea level
B. not mentioned in the passage
C. higher than the highest point in Europe
D. 2300 feet above sea level
E. in Mexico.
Ans : B
7. The deeps are subject to change caused by
A. erosion
B. soundings
C. earthquakes
D. waste
E. weathering
Ans : C
8. The continental masses
A. Rise above sea level
B. Consist of six continents
C. Are relief features of the second order
D. Are partially submerged
E. Comprise 29 per cent of the earth's surface.
Ans : D
16. A clear answer to whether the languages of the ancient American peoples were made use of for expressing abstract universal concepts can be sought in the case of Nahuatl, which like Greek and German, is a language that allows the formation of extensive compounds. By combining radicals or semantic elements, single compound words can express complex conceptual relations, often of an abstract universal character.
The tlamatinime ("those who know") were able to use this rich stock of abstract terms to express the nuances of their thought. They also availed themselves of other forms of expression with metaphorical meaning, some probably original, some derived from Toltec coinages. Of these forms the most characteristic in Nahuatl is the juxtaposition of two words that, because they are synonyms, associated terms, or even contraries, complement each other to evoke one single idea. The juxtaposed terms, used as metaphor, suggest specific or essential traits of the being they refer to, introducing a mode of poetry as an almost habitual form of expression.
1. According to the passage, some abstract universal ideas can be expressed in Nahuatl by
A. Putting various meaningful elements together in one word
B. Taking away from a word any reference to particular instances
C. Turning each word of a phrase into a poetic metaphor
D. Giving a word a new and opposite meaning
E. Removing a word from its associations with other words.
Ans : A
2. It can be inferred solely from the information in the passage that
A. Metaphors are always used in Nahuatl to express abstract conceptual relationships
B. There are many languages that, like Greek or German, allow extensive compounding
C. The abstract terms of the Nahuatl language are habitually used in poetry
D. Some record or evidence of the though of the tlamatinime exists
E. All abstract universal ideas are ideas of complex relations.
Ans : D
3. A main purpose of the passage is to
A. Argue against a theory of poetic expression by citing evidence about the Nahuatl
B. Delineate the function of the tlamatinime in Nahuatl society
C. Explore the rich metaphorical heritage the Nahuatl received from the toltecs
D. Describe some conceptual and aesthetic resources of the Nahuatl language
E. Explain the abstract philosophy of the Nahuatl thinkers.
Ans : D
17. Few areas of neuron behavioral research seemed more promising is the early sixties than that investigating the relationship between protein synthesis and learning. The conceptual framework for the research was derived directly from molecular biology, which had shown that genetic information is stored in nucleic acids and expressed in proteins why not acquired information as well.
The first step towards establishing a connection between protein synthesis and learning seemed to be to block memory (cause adhesion) by interrupting the production of proteins. We were fortunate in finding a non lethal dosage of puromycin that could, it first appealed, thoroughly inhibit brain protein synthesis as well as reliability produce amnesia.
Before the actual connection between protein synthesis and learning could be established however we began to have douche about whether inhibition of protein synthesis was in fact the method by which puromycin produced amnesia. First, ocher drugs, glutavimides themselves potent protein synthesis inhibitors either failed to cause amnesia in some situations where it could easily be induced by puromycin or produced an amnesia with a different time course from that of puromycin. Second, puromycin was found to inhabit protein synthesis by breaking certain amino acid chaim, and the resulting fragments were suspected of being the actual cause of amnesia is some eases. Third, puromycin was reported to cause abnormalities in the train, including seizures. Thus, not only were decreased protein synthesis and amnesia dissociated, but alternative mechanism for the amnestic action of puromycin were readily suggested.
So, puromycin turned out to be a disappointment. It came to be regarded as a poor agent for amnesia studies, although, of course, it was poor only in the context of our original paradigm of protein synthesis inhibition. In our frustration, our initial response was simply to change dregs rather than our conceptual orientation. After many such disappointments, however, it now appears unlikely, that we will make a firm connection between protein synthesis and learning merely by pursuing the approaches of the past our experience with drugs has shown that all the amnestic agents, often interfere with memory in ways that seem unrelated to their inhibition of protein synthesis. More importantly, the notion that the interruption or intensification of protein production in the train can be related in cause and affect fashion to learning non seems simplistic and unproductive. Remove the battery from a car and the car will not go Drive the car a long distance at high speed and the battery will become more highly charged. Neither of these facts proves that the battery power the car, only knowledge of the overall automotive system will reveal it mechanism of locomotion and the role of the battery with in the system.
1. The primary purpose a the passage is to show that extensive experimentation has
A. Mot supported the hypothesis that learning is directly dependent on protein synthesis
B. Cast doubt on the value of puromycin in the newer behavioral study of learning
C. Revealed the importance of amnesia in the neuron behavioral study of learning
D. Demonstrated the importance of amino acid fragmentation in the induction of amnesia.
E. Not yet demonstrated the applicability of molecular biology to behavioral research.
Ans : A
2. According to the passage, neuron behaviorists initially based their belief that protein synthesis was related to learning on which of the following?
A. Specific research into learning on which of the following
B. Traditional theories about learning
C. Historic experiments on the effects puromycin
D. Previous discoveries in molecular biology
E. Now technique in protein synthesis.
Ans : D
3. This passage was most likely excepted from
A. A book review in a leading journal devoted to genetic research.
B. A diary kept by a practicing neuron behavioral research
C. An article summarizing a series of scientific investigations in neuron behavioral research.
D. A news paper article on recent advances in the biochemistry of learning
E. A technical article on experimental techniques in the field of molecular biology.
Ans : C
4. It can be inferred from the passage that after puromycin was perceived to be a disappointment, researches did which of the following?
A. They continued to experiment with puromycin until a neuron anatomical framework was developed.
B. They continued to experiment with puromycin, but also tried other protein synthesis inhibitors
C. They ceased to experiment with puromycin and shifted to other promising protein synthesis inhibitors.
D. They ceased to experiment with puromycin and reexamined through experiments the relationship between genetic information and acquired information.
E. They continued to experiment with puromycin, but applied their results to other facts of memory research.
Ans : C
5. In the example of the car (lines 62-70) the battery is meant to represent which of the following elements in the neuron behavioral research program?
A. glutarimides
B. acquired information
C. puromycin
D. amnesia
E. protein synthesis
Ans : E
6. The passage all of the following as effects of puromycin except
A. Fragmentation of amino-acid chaim
B. Inhibition of protein synthesis
C. Brain seizures
D. Memory loss
E. Destruction of genetic information
Ans : E
7. Which of the following statements would be most likely to come after the last sentences of the passage?
A. It is important in the future, therefore for behavioral bio- chemist to focus on the several components of the total learning system.
B. The ambivalent status of current research, however should not deter neuron behaviorists from exploring the deeper connection between protein production and learning.
C. The failures of the past, however must not impede further research into the amnestic of protein-synthesis inhibitors.
D. It is important in the future, therefore, for behavioral biochemist to emphasize more strongly place of their specific findings within the overall protein synthesis model of learning.
E. It is a legacy of this research, therefore, that molecular biology's genetic models have led to disagreement among neuron behaviorists.
Ans : A
18. In any country, the wages commanded by the laborers who have comparable skills but who work in various industries are determined by the productivity of the least productive unit of labour, i.e. the unit of labour which works in the industry which has catatest economic disadvantages. We will represent the various opportunities of employment in a country like united states by Symbols. A standing for a group of industries in which we have exceptional economic advantage over foreign countries; B for a group in which our advantages are less; E , one in which they are still less; D, the group of industries in which they are the least of all.
When our population is so small that all our labour can be engaged in the group represented by A, productivity of labour and (therefore wages) will be at their maximum. when our population increases so that some of the labour will have to work in group B, the wages of all labour must decline to the level of productivity in that group. But no employer, without government aid, will yet be able to afford to hire labour to exploit the opportunities, represented by E and D, unless there is a further increase in population.
But suppose that the political party in power holds the belief that we should produce everything that we consume, that the opportunities represented by E and D should also be exploited. The commodities, that the industries composing C and D will produce have been hitherto obtained from abroad in exchange for commodities produce by A and B. The government now renders this difficult by imposing high duties upon the former class of commodities. This means that workers in A and B must pay higher prices for what they buy, but do not receive higher prices for what they sell.
After the duty has gone into effect and the prices of commodities that can be produced by C and D have risch sufficiently enterprises will be able to hire labour at the wages prevailing in A and B and establish industries in C and D. So far as the remaining labours in A and B buy the products of C and D ,the difference between the price which they pay for these product and the price they would pay it they were permitted to import those products duty-free is a tax paid not to the government, but to the producers in C and D, to enable the later to remain in business. It is on uncompensated deduction from the natural earnings of the labourers in A and B. nor are the workers in C and D paid as much, estimated in purchasing power as they would have received if they had been allowed to remain in A and B under the earlier conditions.
1. The authors main point is that
A. The government ought to subsidize C and D
B. Wages ought to be independent of international trade
C. It is impossible to attain national self sufficiency
D. The varying productivity of the various industries leads tot he inequalities in wages of workers in these industries
E. A policy that draws labour from the fields of catater natural productiveness to fields of lower natural productiveness tends to redirect purchasing power.
Ans : E
2. No employer, without government aid will yet be able to afford to hire labour to exploit the opportunities represented by C and D because
A. The population has increased
B. Productivity of labour is not at the maximum
C. Productivity would drop correspondingly with the wages of labour
D. We cannot produce everything we consume
E. Enterprises would have to pay wages equivalent to those obtained by workers in A and B while producing under catater disadvantages.
Ans : E
3. When C and D are established, workers in these industries
A. Receives wages equal to those workers in A and B
B. Receives higher wages than do the workers in A and B
C. Are not affected so adversely by the levying of duties as are workers in A and B
D. Must be paid by government funds collected from the duties on imports.
E. Receive lower wages than do the workers in A and B.
Ans : A
4. We cannot exploit C and D unless
A. The producers in E and D are compensated for the disadvantages under which they operate.
B. We export large quantities of commodities produced by A and B
C. The prices of commodities produced by A and B are raised
D. The productivity of labour in all industries is increased
E. We allow duties to be paid to the producers in C and D rather than to the government.
Ans : A
19. MARK HUGHES is a master of the fine art of survival. His Los Angeles-based Herbalife International Inc. is a pyramid outfit that peddles weight-loss and nutrition concoctions of dubious value. Bad publicity and regulatory crackdowns hurt his U.S. business in the late 1980s. But Hughes, 41, continues to enjoy a luxurious lifestyle in a $20 million Beverly Hills mansion. He has been sharing the pad and a yacht with his third wife, a former Miss Petite U.S.A. He can finance this lavish lifestyle just on his salary and bonus, which last year came to $7.3 million.
He survived his troubles in the U.S. by moving overseas, where regulators are less zealous and consumers even more naive, at least initially. Today 77% of Herbalife retail sales derive from overseas. Its new prowling grounds: Asia and Russia. Last year Herbalife's net earnings doubled, to $45 million, on net sales of $632 million. Based on Herbalife's Nasdaq-traded stock, the company has a market capitalization of $790 million, making Hughes 58% worth $454 million.
There's a worm, though, in Hughes apple. Foreigners aren't stupid. In the end they know when they've been had. In France, for instance, retail sales rose to $97 million by 1993 and then plunged to $12 million last year. In Germany sales hit $159 million in 1994 and have since dropped to $54 million.
Perhaps aware that the world may not provide an infinite supply of suckers, Hughes wanted to unload some of his shares. But in March, after Herbalife's stock collapsed, he put off a plan to dump about a third of his holdings on the public.
Contributing to Hughes' woes, Herbalife's chief counsel and legal attack dog, David Addis, quit in January. Before packing up, he reportedly bellowed at Hughes, "I can't protect you anymore." Addis, who says he wants to spend more time with his family, chuckles and claims attorney-client privilege.
Trouble on the home front, too. On a recent conference call with distributors, Hughes revealed he's divorcing his wife, Suzan, whose beaming and perky image adorns much of Herbalife's literature.
Meanwhile, in a lawsuit that's been quietly moving through Arizona's Superior Court, former Herbalife distributor Daniel Fallow of Sandpoint, Idaho charges that Herbalife arbitrarily withholds payment to distributors and marks up its products over seven times the cost of manufacturing. Fallow also claims Hughes wanted to use the Russian mafia to gain entry to that nation's market.
Fallow himself is no angel, but his lawsuit, which was posted on the Internet, brought out other complaints. Randy Cox of Lewiston, Idaho says Herbalife "destroyed my business" after he and his wife complained to the company that they were being cheated out of their money by higher-ups in the pyramid organization.
Will Hughes survive again? Don't count on it this time.
1. Herbalife Inc is based in:
A. Los Angeles
B. Columbus
C. New York
D. Austin
Ans : A
2. Daniel Fallow:
A. Was a former attorney for Hughes
B. Was a former distributor of Herbalife
C. Co-founded Herbalife
D. Ran Herbalife's German unit
Ans : B
3. Which of the following countries is mentioned where Hughes operated Herbalife?
A. India
B. China
C. Germany
D. Ukraine
Ans : C
4. The complaint of Randy Cox of Lewiston, Idaho, against Herbalife was:
A. The company did not pay them their dues
B. The products supplied by Hughes were inferior
C. Their higher-ups in the pyramid cheated them
D. Hughes had connections with the Russian mafia
Ans : C
5. Which of the following countries is NOT mentioned in the passage?
A. Russia
B. USA
C. France
D. Italy
Ans : D
6. In the year in which Hughes' salary and bonuses came to US$ 7.3 million, what was the retail sales for Herbalife in France?
A. $12 million
B. $159 million
C. $54 million
D. $97 million
Ans :A
7. At the time when this article was written, if Herbalife had had a market capitalisation of $ 1 billion, what would have been Hughes' share?
A. $420 million
B. $580 million
C. $125 million
D. $500 million
Ans : B





















Verbal Section : Antonyms Sample Questions
Directions:
Each of the CAT sample antonyms questions below consists of a word printed in Italics, followed by five words or phrase as choices. Choose the word or phrase which is most nearly opposite in meaning to the word in capitals and shade the alphabets marked in the grid on your answer sheet.
Following are some CAT sample antonyms questions.
1. ABOMINATE :
A. loathe
B. despise
C. adore
D. abhor
E. attach
Ans : C
2. OBSEQUIOUS :
A. servile
B. first
C. fawning
D. supercilious
E. improper
Ans : D
3. OROTUND :
A. not resonant
B. not reddish
C. not eager
D. pompous
E. loud
Ans : A
4. RECANT :
A. entangle
B. rescue
C. fail
D. assert
E. predict
5. Ans : D SQUALID
A. fervid
B. florid
C. pristine
D. extraneous
E. abundant
Ans : C
6. SCOTCH
A. renovate
B. entrust
C. unfasten
D. encourage
E. emphasize
Ans : D
7. PERFIDY
A. tact
B. generosity
C. thoroughness
D. loyalty
E. gratitude
Ans : D
8. OUTLANDISH
A. conventional
B. prolific
C. unchanging
D. transparent
E. noticeable
Ans : A
9. PLUMB
A. reversed
B. lofty
C. horizontal
D. thin
E. light
Ans : C
10. FERVID
A. undistinguished
B. unexpected
C. stubborn
D. restrained
E. discouraged
Ans : D
11. VACUITY
A. quality
B. certainty
C. plenitude
D. stability
E. incontinence
Ans : C
12. RAVEL
A. knit
B. omit
C. remain silent
D. measure
E. increase in value
Ans : A
13. PERSISTENCE
A. irrelevance
B. inconstancy
C. inequality
D. intemperance
E. incompetence.
Ans : B
14. SUBROSA
A. openly
B. fashionably
C. under the owse
D. simply
E. clandestinely
Ans : A

15. UPBRAID :
A. defer
B. vacillate
C. sever
D. conjoin
E. laud
Ans : E
16. PLENITUDE :
A. luxury
B. magnificence
C. richness
D. contentment
E. scarcity
Ans : E
17. SCURRILOUS :
A. decent
B. savage
C. major
D. volatile
E. scabby
Ans : A
18. FULMINATION :
A. praise
B. repetition
C. escape
D. ratification
E. addition
Ans : A
19. DISTEND
A. deteriorate
B. weaken
C. constrict
D. concentrate
E. fold
Ans : C
20. TOUT
A. cast aspersions on
B. deny the relevance of
C. placate
D. withhold consent
E. misrepresent
Ans : E
21. PREFATORY :
A. intelligent
B. outstanding
C. predatory
D. conclusive
E. magnificent
Ans : E
22. CONCILIATE :
A. arrive
B. appeal
C. retaliate
D. estrange
E. lie
Ans : B
23. SUBSERVIENT :
A. fawning
B. obsequious
C. miserly
D. omnipresent
E. haughty
Ans : C
24. VAUNTED :
A. berated
B. belittled
C. lauded
D. wicked
E. worried
Ans : C
25. QUOTA :
A. Anonymous remark
B. decisive action
C. debatable issue
D. unlimited number
E. irrelevant topic
Ans : D
26. CONTENTIOUS :
A. satisfied
B. pacific
C. hungry
D. bellicose
E. dissatisfied
Ans : D
27. OBLOQUY :
A. fame
B. name
C. inquiry
D. shame
E. collogue
Ans : D
28. PENCHANT :
A. distaste
B. scabbard
C. agreement
D. earring
E. beginning
Ans : C
29. BALEFUL :
A. empty
B. tasty
C. gaudy
D. full
E. congenial
Ans : D
30. CURT :
A. contractual
B. precise
C. honest
D. voluble
E. peremptory
Ans : D
31. ANIMOSITY
A. parody
B. retardation
C. sincerity
D. refutation
E. canaraderie
Ans : E
32. INVETERATE
A. uninvited
B. illiterate
C. cumulative
D. beginning
E. incompetent
Ans : D
33. SCOTCH
A. renovate
B. encourage
C. entrust
D. ameliorate
Ans : B
34. PREDILECTION
A. ambiguity
B. unwillingness to choose
C. desire to please
D. propensity to dislike
E. stereotype
Ans : D
35. CHOLERIC
A. good-natured
B. spoiled
C. irascible
D. immune
E. idiotic.
Ans : A
36. EXACERBATE
A. contemplate
B. bewilder
C. reward
D. better
E. horify
Ans : D
37. EQUANIMITY
38. clamour CUPIDITY
A. generosity
B. love
C. anxiety
D. entertainment
E. tragedy.
Ans : A
39. ANIMOSITY
A. parody
B. retardation
C. sincerity
D. refutation
E. canaraderie
Ans : B
40. INVETERATE
A. uninvited
B. illiterate
C. cumulative
D. beginning
E. incompetent
Ans : A
41. SATURNINE :
A. quick – wilted
B. genial
C. heavy – handed
D. distinguished
E. devout
Ans : E
42. PERSPICACIOUS :
A. Insufficiently precise
B. of indefinite duration
C. dull wilted
D. lacking intrinsic value
E. condemnatory
Ans : D
43. INCARCERATE :
A. summon
B. assist
C. liberate
D. anticipate
E. confide
Ans : C
44. INSOLVENCY :
A. ability to sustain growth
B. concentration
C. coherence
D. ability to pay one’s debts
E. compatibility
Ans : D
45. EFFLUVIA :
A. controlled reactions
B. predictable results
C. important examples
D. descried products
E. relevant theories
Ans : C
46. APPOSITE :
A. parallel
B. synonymous
C. hostile
D. inappropriate
E. vague
Ans : D
47. GRATUITOUS :
A. frank
B. pithy
C. warranted
D. frugal
E. ingenuous
Ans : A
F.
G. volume
H. disparity
I. agitation
J. caution
Ans : D
48. ANIMADVERSION
A. gullibility
B. precision
C. praise
D. sobriety
E. criticize
Ans : C
49. EXHUME
A. enter
B. fertilize
C. inter
D. decay
E. clarify
Ans : C
50. CALLOW
A. rustic
B. crude
C. exquisite
D. experienced
E. migratory
Ans : D
51. PREFATORY :
A. intelligent
B. outstanding
C. predatory
D. conclusive
E. magnificent
Ans : E
52. CONCILIATE :
A. arrive
B. appeal
C. retaliate
D. estrange
E. lie
Ans : B
53. SUBSERVIENT :
A. fawning
B. obsequious
C. miserly
D. omnipresent
E. haughty
54. Ans : C INVIDIOUS :
A. candid
B. stubborn
C. defensive
D. hostile
E. inoffensive
Ans : E
55. MACERATE :
A. cover by painting
B. assess by observing
C. harden by drying
D. influence by lying
E. cure by medicating
Ans : B
56. SKEPTICISM :
A. reason
B. conviction
C. plausibility
D. audricty
E. argument
Ans : D
57. IGNOMINIOUS :
A. scholarly
B. incognito
C. laudatory
D. disgraceful
E. erudite
Ans : B
58. CODA :
A. creflain
B. crescendo
C. prelude
D. improvisation
E. solo
Ans : A
59. PALTRY :
A. farm
B. scanty
C. excessive
D. friendly
E. benevolent
Ans : A
60. PUISSANCE :
A. strength
B. knowledge
C. liberality
D. skepticism
E. powerlessness
Ans : E
61. MANUMIT :
A. print
B. impress
C. enslave
D. fail
E. endeavor
Ans : D
62. GENUFLECT :
A. pronounce correctly
B. falsify
C. trick
D. stand erect
E. project
Ans : E
63. INNOCUOUS :
A. toxic
B. large
C. sober
D. impeccable
E. spotless
Ans : C

64. VAUNTED :
A. berated
B. belittled
C. lauded
D. wicked
E. worried
Ans : C
65. QUOTA :
A. Anonymous remark
B. decisive action
C. debatable issue
D. unlimited number
E. irrelevant topic
Ans : D
66. CONTENTIOUS :
A. satisfied
B. pacific
C. hungry
D. bellicose
E. dissatisfied
Ans : D
67. OBLOQUY :
A. fame
B. name
C. inquiry
D. shame
E. collogue
Ans : D
68. PENCHANT :
A. distaste
B. scabbard
C. agreement
D. earring
E. beginning
Ans : C
69. BALEFUL :
A. empty
B. tasty
C. gaudy
D. full
E. congenial
Ans : D
70. CURT :
A. contractual
B. precise
C. honest
D. voluble
E. peremptory
Ans : D
71. BAROQUE :
A. rococo
B. simple
C. common
D. stupid
E. boat like
Ans : B
72. MYOPIC :
A. blind
B. moral
C. visionary
D. farsighted
E. glassy
Ans : C
73. NASCENT :
A. loyal
B. fading
C. unnamed
D. treacherous
E. reoccuring
Ans : C
74. LOLL :
A. describe exactly
B. insist strongly
C. comply readily
D. notice incidentally
E. move vigorously
Ans : E
75. TURBULENCE :
A. immunity
B. tranquility
C. meditation
D. moderation
E. co – ordination
Ans : E
76. BANAL :
A. inclined
B. faithful
C. elaborate
D. forced
E. arresting
Ans : C
77. GERMINAL
A. sterilized
B. strategic
C. fully developed
D. primitive
E. excused .
Ans : D
78. GASCONADE
A. modesty
B. transparency
C. seizure
D. cleanliness
E. imposture
Ans : A
79. MIASMA
80. scenario CENSURE
A. uncertainity
B. encomium
C. criticism
D. legal contual
E. matrimony
Ans : B
81. COMMODIOUS
A. product
B. space
C. cramped
D. company
E. roomy.
Ans : E
82. EFFRONTERY
A. modesty
B. confrontation
C. avoidance
D. shamelessness
E. impudence
Ans : B
83. OBSTREPEROUS
A. noisy
B. defiant
C. permeable
D. quiet
E. stubborn
Ans : E
84. PACIFY
A. ameliorate
B. patchup
C. truce
D. tormented
E. agitated
Ans : C
85. AMBIGUOUS
A. confusing
B. lucid
C. desirous
D. obfuscate
E. pun
Ans : A
86. MILITANT
A. Dramatic
B. combative
C. religious
D. pacific
E. quaint.
Ans : D
87. MOTILITY :
A. static
B. tension
C. ascent
D. liquidity
E. vulnerability
Ans : A
88. SINUOUS :
A. wet
B. vacant
C. numerous
D. direct
E. round
Ans : D
89. PLUMB :
A. reversed
B. horizontal
C. light
D. lofty
E. thin
Ans : B
F.
G. summing up
H. noxious fumes
I. fragrant aroma
J. benevolent
Ans : B
90. OPPORTUNIST
A. Man of principle
B. fatalist
C. fledgling
D. colleague
E. foe.
Ans : D
91. SEGMENT:
A. inflate
B. affix
C. keep still
D. make whole
E. cleanse
Ans : D
92. OSSIFY :
A. reassemble fragments
B. overlook problems
C. create consensus
D. placate critics
E. transcend conventions
Ans : E
93. RAVEL :
A. increase in value
B. omit
C. remain silent
D. measure
E. knit
Ans : E
94. CALUMINATE :
A. vindicate
B. supplant
C. rejuvenate
D. follow
E. familiarize
Ans : A
95. TURPITUDE :
A. pragmatism
B. probity
C. judiciousness
D. animation
E. determinedness
Ans : B
96. INVECTIVE :
A. willing compliance
B. normality
C. restoration
D. fertility
E. laudatory words
Ans : E
97. PILLORY :
A. lament
B. foster
C. exalt
D. enjoy
E. forgive
Ans : C
98. UNTOWARD :
A. experienced
B. inevitable
C. industrious
D. straight forward
E. favourable
Ans : E

Quantitative Comparisons Sample Questions
Directions:
In this section you will be given two quantities, one in column A and one in column B. You are to determine a relationship between the two quantities and mark.
A. If the quantity in column A is greater than the quantity in column B.
B. If the quantity in column B is greater than the quantity in column A.
C. If the quantities are equal.
D. If the comparison cannot be determined from the information that is given.
1. Quantity A: (-6)4
Quantity B: (-6)5
A. if the quantity A is greater;
B. if the quantity B is greater;
C. if the two quantities are equal;
D. if the relationship cannot be determined from the information given.
Ans : A
2. Quantity A: Time to travel 95 miles at 50 miles per hour
Quantity B: Time to travel 125 miles at 60 miles per hour
A. Quantity A is greater
B. Quantity A equals Quantity B
C. Quantity B is greater
D. Relationship Indeterminate
Ans : C
3. Quantity A: (9/13)2
Quantity B: (9/13)1/2
A. Quantity A equals Quantity B
B. Relationship Indeterminate
C. Quantity B is greater
D. Quantity A is greater
Ans : C
4. Quantity A: 4 / 100
Quantity B: 0.012 / 3
A. Quantity B is greater
B. Quantity A equals Quantity B
C. Quantity A is greater
D. Relationship Indeterminate
Ans : C
5. x = 2y + 3
y = -2
Quantity A: x
Quantity B: -1
A. if the quantity in Column A is greater
B. if the quantity in Column B is greater
C. if the two quantities are equal
D. if the relationship cannot be determined from the information given
Ans : C
6. x + 2y > 8
Quantity A: 2x + 4y
Quantity B: 20
A. if the quantity in Column A is greater
B. if the quantity in Column B is greater
C. if the two quantities are equal
D. if the relationship cannot be determined from the information given.
Ans : D
7. Quantity A: The number of months in 7 years
Quantity B: The number of days in 12 weeks
A. if the quantity in Column A is greater
B. if the quantity in Column B is greater
C. if the two quantities are equal
D. if the relationship cannot be determined from the information given
Ans : C
8. Quantity A: 1-1/27
Quantity B: 8/9 + 1/81
A. if the quantity in is greater
B. if the quantity in is greater
C. if the two quantities are equal
D. if the relationship cannot be determined from the information given.
Ans : A
9. r/>s/>0/>
Quantity A: rs/r
Quantity B: rs/s
A. if the quantity A is greater
B. if the quantity B is greater
C. if the two quantities are equal
D. if the relationship cannot be determined from the information given.
Ans : B
10. Quantity A: 0.83
Quantity B: 0.81/3
A. Quantity B is greater
B. Relationship Indeterminate
C. Quantity A is greater
D. Quantity A equals Quantity B
11. Ans : A t is a positive integer.
4/7 = t/s
Quantity A: s
Quantity B:7
A. if the quantity in Column A is greater;
B. if the quantity in Column B is greater;
C. if the two quantities are equal;
D. if the relationship cannot be determined from the information given
Ans : D
12. Quantity A: (0.82)2(0.82)3
Quantity B:(0.82)6
A. if the quantity in Column A is greater;
B. if the quantity in Column B is greater;
C. if the two quantities are equal;
D. if the relationship cannot be determined from the information given.
Ans : A
13. For all real numbers a, let a* = 1 - a.
Quantity A: ((-1)*)*
Quantity B: 2*
A. if the quantity in Column A is greater;
B. if the quantity in Column B is greater;
C. if the two quantities are equal;
D. if the relationship cannot be determined from the information given.
Ans : C
14. Quantity A: (x - 1)(x)(x + 1)
Quantity B:(x)(x)(x)
A. if the quantity in Column A is greater;
B. if the quantity in Column B is greater;
C. if the two quantities are equal;
D. if the relationship cannot be determined from the information given.
Ans : D
15. Quantity A: (3 x 4 x 17) / (121 x 100)
Quantity B: (4 x 5 x 19) / (1000 x 121)
A. Quantity A is greater
B. Quantity A equals Quantity B
C. Relationship Indeterminate
D. Quantity B is greater
A
16. Consider a triangle PQR.
Quantity A: length of PQ + length of QR
Quantity B: length of PR
A. Quantity A is greater
B. Quantity B is greater
C. Relationship Indeterminate
D. Quantity A equals Quantity B
Ans : A
17. Quantity A: (27 - 13) (296 + 534)
Quantity B: (27 + 13) (534 + 296)
A. Quantity B is greater
B. Quantity A equals Quantity B
C. Relationship Indeterminate
D. Quantity A is greater
Ans : D
18. Quantity A: A = 1.1
Quantity B: 12.11/2
A. Relationship Indeterminate
B. Quantity B is greater
C. Quantity A equals Quantity B
D. Quantity A is greater
Ans : B
19. 100 < y < 200 and 100 < z < 210
Quantity A: y
Quantity B: z
A. Quantity A is greater
B. Quantity A equals Quantity B
C. Quantity B is greater
D. Relationship Indeterminate
Ans : D
20. y2 + z2 = 34 and yz = 15
Quantity A: y2 + 2yz + z2
Quantity B: (y + z)2
A. Quantity B is greater
B. Relationship Indeterminate
C. Quantity A is greater
D. Quantity A equals Quantity B
Ans : D

21. Consider a rectangle. The length of its shorter side is 8, and the length of its diagonal is 16.
Quantity A: 30o
Quantity B: measure of angle formed by diagonal and shorter side
A. Relationship Indeterminate
B. Quantity A equals Quantity B
C. Quantity A is greater
D. Quantity B is greater
Ans : D
22. Quantity A: (y + 5)2
Quantity B: (y - 5)2
A. Quantity B is greater
B. Relationship Indeterminate
C. Quantity A equals Quantity B
D. Quantity A is greater
Ans : B
23. Quantity A: (1/25)1/2 + (1/144)1/2
Quantity B: [(1/25) + (1/144)]1/2
A. Relationship Indeterminate
B. Quantity A is greater
C. Quantity B is greater
D. Quantity A equals Quantity B
Ans : A
24. y2 + z2 = 34 and yz = 15
Quantity A: y2 + 2yz + z2
Quantity B: (y + z)2
A. Quantity A is greater
B. Relationship Indeterminate
C. Quantity A equals Quantity B
D. Quantity B is greater
Ans : C
25. 100 < y < 200 and 100 < z < 210
Quantity A: y
Quantity B: z
A. Quantity A is greater
B. Quantity A equals Quantity B
C. Quantity B is greater
D. Relationship Indeterminate
Ans : D
26. Quantity A: (y + 5)2
Quantity B: (y - 5)2
A. Quantity A equals Quantity B
B. Quantity A is greater
C. Relationship Indeterminate
D. Quantity B is greater
Ans : C
27. Consider a rectangle. The length of its shorter side is 8, and the length of its diagonal is 16.
Quantity A: 30o
Quantity B: measure of angle formed by diagonal and shorter side
A. Quantity A is greater
B. Quantity A equals Quantity B
C. Quantity B is greater
D. Relationship Indeterminate
Ans : C
28. The sum of three consecutive even numbers is 18.
Quantity A: Their average
Quantity B: 6
A. Relationship Indeterminate
B. Quantity A is greater
C. Quantity A equals Quantity B
D. Quantity B is greater
Ans : C
29. x - y > 10
Quantity A: y - x
Quantity B: 12
A. Quantity B is greater
B. Quantity A is greater
C. Quantity A equals Quantity B
D. Relationship Indeterminate
Ans : A
30. x = 0, y > 0
Quantity A: xy
Quantity B: yx
A. Quantity A equals Quantity B
B. Quantity A is greater
C. Quantity B is greater
D. Relationship Indeterminate
Ans : C
Quantitative Comparisons Sample Questions
31.
Diagram is illustrative and is not drawn to scale.
Quantity A: Measure of angle 3 - Measure of angle 2
Quantity B: Measure of angle 5 - Measure of angle 6
A. Relationship Indeterminate
B. Quantity A equals Quantity B
C. Quantity B is greater
D. Quantity A is greater
Ans : B
32. Quantity A: 29
Quantity B: 92
A. Quantity B is greater
B. Quantity A is greater
C. Relationship Indeterminate
D. Quantity A equals Quantity B
Ans : B
33. 0 < -x < 10
11 < -y < 20
Quantity A: x
Quantity B: y
A. Relationship Indeterminate
B. Quantity A equals Quantity B
C. Quantity B is greater
D. Quantity A is greater
Ans : D
34.
Diagram is illustrative and is not drawn to scale.
Given angles 1 and 2 are equal,
length of side AB = x, length of side BC = y, length of side AC = z.
Quantity A: x + y
Quantity B: y + z
A. Quantity A is greater
B. Quantity A equals Quantity B
C. Quantity B is greater
D. Relationship Indeterminate
Ans : B
35.
Diagram is illustrative and is not drawn to scale.
In triangle ABC, AB = AC and measure of angle 1 = 100o.
Quantity A: Measure of angle 2 + Measure of angle 3
Quantity B: 90o
A. Quantity B is greater
B. Relationship Indeterminate
C. Quantity A is greater
D. Quantity A equals Quantity B
Ans : A
36. x and y are both positive and x/y > 5
Quantity A: 0.2x
Quantity B: y
A. Quantity A is greater
B. Quantity B is greater
C. Relationship Indeterminate
D. Quantity A equals Quantity B
Ans : A
37.
Diagram is illustrative and is not drawn to scale.
Given AB = AC and angle BAC = 60o
Quantity A: Length of side AB
Quantity B: Length of side BC
A. Quantity A equals Quantity B
B. Quantity B is greater
C. Relationship Indeterminate
D. Quantity A is greater
Ans : A
38. y2 = 36
Quantity A: y
Quantity B: 6
A. Relationship Indeterminate
B. Quantity A is greater
C. Quantity B is greater
D. Quantity A equals Quantity B
Ans : A
39.
Diagram is illustrative and is not drawn to scale.
Quantity A: Measure of angle 1 + Measure of angle 2 + Measure of angle 4
Quantity B: 180o
A. Relationship Indeterminate
B. Quantity A is greater
C. Quantity B is greater
D. Quantity A equals Quantity B
Ans : D
40.
Diagram is illustrative and is not drawn to scale.
In triangle ABC, angle A = 60o and AB = AC.
Quantity A: Measure of angle 1 + Measure of angle 2
Quantity B: 120o
A. Relationship Indeterminate
B. Quantity A is greater
C. Quantity A equals Quantity B
D. Quantity B is greater
Ans : C
41.
Diagram is illustrative and is not drawn to scale.
Quantity A: Measure of angle 2 + Measure of angle 3
Quantity B: 180o
A. Quantity B is greater
B. Quantity A is greater
C. Quantity A equals Quantity B
D. Relationship Indeterminate
Ans : C
42.
Diagram is illustrative and is not drawn to scale.
AB is the diameter of the circle.
Quantity A: Measure of angle 1
Quantity B: Measure of angle 2
A. Relationship Indeterminate
B. Quantity A is greater
C. Quantity B is greater
D. Quantity A equals Quantity B
Ans : D
43.
Diagram is illustrative and is not drawn to scale.
Quantity A: Measure of angle 1 + Measure of angle 3
Quantity B: Measure of angle 2 + Measure of angle 4
A. Relationship Indeterminate
B. Quantity A is greater
C. Quantity A equals Quantity B
D. Quantity B is greater
Ans : A
44.
Diagram is illustrative and is not drawn to scale.
In triangle ABC, AB = AC and measure of angle 1 = 100o.
Quantity A: Measure of angle 2 + Measure of angle 3
Quantity B: 90o
A. Quantity B is greater
B. Quantity A equals Quantity B
C. Relationship Indeterminate
D. Quantity A is greater
Ans : A
45.
Diagram is illustrative and is not drawn to scale.
Given angles 1 and 2 are equal,
length of side AB = x, length of side BC = y, length of side AC = z.
Quantity A: x + y
Quantity B: y + z
A. Quantity B is greater
B. Quantity A equals Quantity B
C. Quantity A is greater
D. Relationship Indeterminate
Ans : B
46. x and y are both positive and x/y > 5
Quantity A: 0.2x
Quantity B: y
A. Quantity B is greater
B. Relationship Indeterminate
C. Quantity A equals Quantity B
D. Quantity A is greater
Ans : D
47. yz < 0
Quantity A: (y - z)2
Quantity B: y2 + z2
A. Quantity A is greater
B. Quantity B is greater
C. Quantity A equals Quantity B
D. Relationship Indeterminate
Ans : A
48. For any positive integer n,
n! is the product of all positive integers less than or equal to n.
Quantity A: 20! / 17!
Quantity B: 80! / 78!
A. Quantity A is greater
B. Quantity B is greater
C. Quantity A equals Quantity B
D. Relationship Indeterminate
Ans : A
49. 2 < z < 4
Quantity A: π2z3
Quantity B: π3z2
A. Quantity A is greater
B. Quantity B is greater
C. Quantity A equals Quantity B
D. Relationship Indeterminate
Ans : D
50. Amy, Beth and Charlie divided a pizza amongst themselves.
Amy took 30% of the pizza and ate (3/4) of what she took.
Beth took 20% of the pizza.
Charlie ate (2/5) of what he took.
Quantity A: The amount Amy ate
Quantity B: The amount Charlie ate
A. Quantity A is greater
B. Quantity B is greater
C. Quantity A equals Quantity B
D. Relationship Indeterminate
Ans : A
51. p > 0 > q
Quantity A: p + q
Quantity B: pq
A. The quantity in Column A is greater.
B. The quantity in Column B is greater.
C. The quantities are equal.
D. The relationship cannot be determined from the information given.
Ans : D
52. Quantity A: The average (arithmetic mean) of x and y
Quantity B: The average (arithmetic mean) of x - 1 and y + 1
A. The quantity in Column A is greater.
B. The quantity in Column B is greater.
C. The quantities are equal.
D. The relationship cannot be determined from the information given.
Ans : C
53. The integer (x - 1) is a prime number between 40 and 50.
Quantity A: The sum of all different prime factors of x
Quantity B: 14
A. The quantity in Column A is greater.
B. The quantity in Column B is greater.
C. The quantities are equal.
D. The relationship cannot be determined from the information given.
Ans : B
54. A < C
B > D > 0
Quantity A: A - B
Quantity B: C - D
A. The quantity in Column A is greater.
B. The quantity in Column B is greater.
C. The quantities are equal.
D. The relationship cannot be determined from the information given.
Ans : B
55. In a particular jellybean jar, the number of red jellybeans exceeds the number of white ones by a ratio of 3:2. If two red jellybeans were removed, the ratio of red to white jellybeans would be 1:1.
Quantity A: The number of white jellybeans in the jar
Quantity B: 4
A. The quantity in Column A is greater.
B. The quantity in Column B is greater.
C. The quantities are equal.
D. The relationship cannot be determined from the information given.
Ans : C
Quantitative Section : Quantitative Sample Questions
Directions:In this section you will be given two quantities, one in column A and one in column B. You are to determine a relationship between the two quantities and mark.
A. If the quantity in column A is catater than the quantity in column B.
B. If the quantity in column B is catater than the quantity in column A.
C. If the quantities are equal.
D. If the comparison cannot be determined from the information that is given.
1. A rectangle is 14 cm long and 10 cm wide. If the length is reduced by x cms and its width is increased also by x cms so as to make it a square then its area changes by :
A. 4
B. 144
C. 12
D. 2
E. None of the above.
Ans : A
2. A motorcycle stunts man belonging to a fair, rides over the vertical walls of a circular well at an average speed of 54 kph for 5 minutes. If the radius of the well is 5 meters then the distance traveled is:
A. 2.5 kms
B. 3.5 kms
C. 4.5 kms
D. 5.5 kms
E. None of the above
Ans : C
3. If 1 cm on a map corresponds to an actual distance of 40 kms. And the distance on the map between Bombay and Calcutta is 37.5 cms., the actual distance between them is :
A. 375 kms
B. 3750 kms
C. 1500 kms
D. 1375 kms
E. None of the above
Ans : C
4. A box contains 90 mts each of 100 gms and 100 bolts each of 150 gms. If the entire box weighs 35.5 kg., then the weight of the empty box is :
A. 10 kg
B. 10.5 kg
C. 11 kg
D. 11.5 kg
E. None of the above
Ans : D
5. If the radius of a circle is increased by 20% then the area is increased by :
A. 44%
B. 120%
C. 144%
D. 40%
E. None of the above
Ans : A
6. Tom, Dick and Harry went for lunch to a restaurant. Tom had $100 with him, Dick had $60 and Harry had $409. They got a bill for $104 and decided to give a tip of $16. They further decided to share the total expenses in the ratio of the amounts of money each carried. The amount of money which Tom paid more than what Harry paid is
A. 120
B. 200
C. 60
D. 24
E. 36
Ans : E
7. A plot of land is in the shape of a trapezium whose dimensions are given in the figure below :

Hence the perimeter of the field is
A. 50 m
B. 64 m
C. 72 m
D. 84 m
E. None of the above
Ans : c
8. Four concentric ( having the same center ) circles with radii, x, 2x, 3x and 4x are drawn to form two rings A and B as shown in the figure.

Ratio of the area of inner ring A to the area of outer ring B is
A. 1 : 2
B. 1 : 4
C. 2 : 3
D. 3 : 7
E. None of the above
Ans : D
9. If 3/p = 6 and 3/q = 15 then p - q = ?
A. 1/3
B. 2/5
C. 3/10
D. 5/6
E. None of the above
Ans : C
10. A father is three times as old as his son. After fifteen years the father will be twice as old as his son's age at that time. Hence the father's present age is
A. 36
B. 42
C. 45
D. 48
E. None of the above
Ans : C
11. (1/4)3 + (3/4)3 + 3(1/4)(3/4)(1/4 + 3/4) =?
A. 1/64
B. 27/64
C. 49/64
D. 0
E. 1
Ans : E
12. If the area of two circles are in the ratio 169 : 196 then the ratio of their radii is
A. 10 : 11
B. 11 : 12
C. 12 : 13
D. 13 : 14
E. None of the above
Ans : D
13. A semi-circle is surmounted on the side of a square. The ratio of the area of the semi-circle to the area of the square is

A. 1 : 2
B. 2 : p
C. p : 8
D. 8 : p
E. None of the above
Ans : C
14. Which of the following is the greatest ?
A. 40% of 30
B. 3/5 of 25
C. 6.5% of 200
D. Five more than the square of 3
E. 1/2-4
Ans : E
15. Two identical taps fill 2/5 of a tank in 20 minutes. When one of the taps goes dry in how many minutes will the remaining one tap fill the rest of the tank ?
A. 5 minutes
B. 10 minutes
C. 15 minutes
D. 20 minutes
E. None of the above
Ans : C
16. If the value of XYZ Company stock drops from $25 per share to $21 per share, what is the percent of the decrease?
A. 4
B. 8
C. 12
D. 16
E. 20
Ans : D
17. If a building b feet high casts a shadow f feet long, then, at the same time of day, a tree t feet high will cast a shadow how many feet long?
A. ft/b
B. fb/t
C. b/ft
D. tb/f
E. t/fb
Ans : A
18. If x, y, and z are consecutive negative integers, and if x > y > z, which of the following must be a positive odd integer?
A. xyz
B. (x - y) (y - z)
C. x - yz
D. x(y + z)
E. x + y + z
Ans : B
19. At a certain ice cream parlor, customers can choose among five different ice cream flavors and can choose either a sugar cone or a waffle cone. Considering both ice cream flavor and cone type, how many distinct triple-scoop cones with three different ice cream flavors are available?
A. 12
B. 16
C. 20
D. 24
E. 30
Ans : C
20. What is the greatest value of a positive integer n such that 3n is a factor of 1815?
A. 15
B. 18
C. 30
D. 33
21. 45 If .2t = 2.2 - .6s and .5s = .2t + 1.1, then s =
A. 1
B. 3
C. 10
D. 11
E. 30
Ans : B
22. Five years ago, Beth's age was three times that of Amy. Ten years ago, Beth's age was one half that of Chelsea. If C repre- sents Chelsea's current age, which of the following represents Amy's current age?
A. c/6 + 5
B. 2c
C. (c-10)/3
D. 3c-5
E. 5c/3 - 10
Ans : A
23. A portion of $7200 is invested at a 4% annual return, while the remainder is invested at a 5% annual return. If the annual income from both portions is the same, what is the total income from the two investments?
A. $160
B. $320
C. $400
D. $720
E. $1,600
Ans : B
24. An empty swimming pool can be filled to capacity through an inlet pipe in 3 hours, and it can be completely drained by a drainpipe in 6 hours. If both pipes are fully open at the same time, in how many hours will the empty pool be filled to capacity?
A. 4
B. 4.5
C. 5
D. 5.5
E. 6
Ans : E
25. If r = (3p + q)/2 and s = p - q, for which of the following values of p would r2 = s2?
A. 1q/5
B. 10 - 3q/2
C. q - 1
D. 3q
E. 9q/2 - 9
Ans : A
26. At 10 a.m. two trains started traveling toward each other from stations 287 miles apart. They passed each other at 1:30 p.m. the same day. If the average speed of the faster train exceeded the average speed of the slower train by 6 miles per hour, which of the following represents the speed of the faster train, in miles per hour?
A. 38
B. 40
C. 44
D. 48
E. 50
Ans : C
27. On the xy-coordinate plane, points A and B both lie on the circumference of a circle whose center is O, and the length of AB equals the circle's diameter. If the (x,y) coordinates of O are (2,1) and the (x,y) coordinates of B are (4,6), what are the (x,y) coordinates of A?
A. (3, 3/2)
B. (1, 2/2)
C. (0, -4)
D. (2/2, 1)
E. (-1, -2/2)
Ans : C
28. If a rectangle's length and width are both doubled, by what percent is the rectangle's area increased?
A. 50
B. 100
C. 200
D. 300
E. 400
Ans : D
29. A rectangular tank 10" by 8" by 4" is filled with water. If all of the water is to be transferred to cube-shaped tanks, each one 3 inches on a side, how many of these smaller tanks are needed?
A. 9
B. 12
C. 16
D. 21
E. 39
Ans : B
30. Point Q lies at the center of the square base (ABCD) of the pyramid pictured above. The pyramid's height (PQ) measures exactly one half the length of each edge of its base, and point E lies exactly halfway between C and D along one edge of the base. What is the ratio of the surface area of any of the pyramid's four triangular faces to the surface area of the shaded triangle?
A. 3 :√2
B. √5:1
C. 4√3:3
D. 2√2:1
E. 8:√5
Ans : D
31. The average wages of a worker during a fortnight comprising 15 consecutive working days was Rs.90 per day. During the first 7 days, his average wages was Rs.87/day and the average wages during the last 7 days was Rs.92 /day. What was his wage on the 8th day?
A. 83
B. 92
C. 90
D. 97
Ans : D
32. The average of 5 quantities is 6. The average of 3 of them is 8. What is the average of the remaining two numbers?
A. 6.5
B. 4
C. 3
D. 3.5
Ans : C
33. The average temperature on Wednesday, Thursday and Friday was 250. The average temperature on Thursday, Friday and Saturday was 240. If the temperature on Saturday was 270, what was the temperature on Wednesday?
A. 240
B. 210
C. 270
D. 300
Ans : D
34. The average age of a group of 12 students is 20years. If 4 more students join the group, the average age increases by 1 year. The average age of the new students is
A. 24
B. 26
C. 23
D. 22
Ans : A
35. When a student weighing 45 kgs left a class, the average weight of the remaining 59 students increased by 200g. What is the average weight of the remaining 59 students?
A. 57 kgs
B. 56.8 kgs
C. 58.2 kgs
D. 52.2 kgs
Ans : A
36. Three math classes: X, Y, and Z, take an algebra test.
The average score in class X is 83.
The average score in class Y is 76.
The average score in class Z is 85.
The average score of all students in classes X and Y together is 79.
The average score of all students in classes Y and Z together is 81.
What is the average for all the three classes?
A. 81
B. 81.5
C. 82
D. 84.5
Ans : B
37. The average weight of a class of 24 students is 36 years. When the weight of the teacher is also included, the average weight increases by 1kg. What is the weight of the teacher?
A. 60 kgs
B. 61 kgs
C. 37 kgs
D. None of these
Ans : B
38. The average of 5 quantities is 10 and the average of 3 of them is 9. What is the average of the remaining 2?
A. 11
B. 12
C. 11.5
D. 12.5
Ans : C
39. The average age of a family of 5 members is 20 years. If the age of the youngest member be 10 years then what was the average age of the family at the time of the birth of the youngest member?
A. 13.5
B. 14
C. 15
D. 12.5
Ans : D
40. A student finds the average of 10 positive integers. Each integer contains two digits. By mistake, the boy interchanges the digits of one number say ba for ab. Due to this, the average becomes 1.8 less than the previous one. What was the difference of the two digits a and b?
A. 8
B. 6
C. 2
D. 4
Ans : C
41. Average cost of 5 apples and 4 mangoes is Rs. 36. The average cost of 7 apples and 8 mangoes is Rs. 48. Find the total cost of 24 apples and 24 mangoes.
A. 1044
B. 2088
C. 720
D. 324
Ans : B
41. A father left a will of Rs.35 lakhs between his two daughters aged 8.5 and 16 such that they may get equal amounts when each of them reach the age of 21 years. The original amount of Rs.35 lakhs has been instructed to be invested at 10% p.a. simple interest. How much did the elder daughter get at the time of the will?
A. Rs. 17.5 lakhs
B. Rs. 21 lakhs
C. Rs. 15 lakhs
D. Rs. 20 lakhs
Ans : B
42. What will Rs.1500 amount to in three years if it is invested in 20% p.a. compound interest, interest being compounded annually?
A. 2400
B. 2592
C. 2678
D. 2540
Ans : B
43. If a sum of money grows to 144/121 times when invested for two years in a scheme where interest is compounded annually, how long will the same sum of money take to treble if invested at the same rate of interest in a scheme where interest is computed using simple interest method?
A. 9 years
B. 22 years
C. 18 years
D. 33 years
Ans : B
44. The population of a town was 3600 three years back. It is 4800 right now. What will be the population three years down the line, if the rate of growth of population has been constant over the years and has been compounding annually?
A. 6000
B. 6400
C. 7200
D. 9600
Ans : B
45. A man invests Rs.5000 for 3 years at 5% p.a. compound interest reckoned yearly. Income tax at the rate of 20% on the interest earned is deducted at the end of each year. Find the amount at the end of the third year.
A. 5624.32
B. 5630.50
C. 5788.125
D. 5627.20
Ans : A
46. The difference between the compound interest and the simple interest on a certain sum at 12% p.a. for two years is Rs.90. What will be the value of the amount at the end of 3 years?
A. 9000
B. 6250
C. 8530.80
D. 8780.80
Ans : D
47. Vijay invested Rs.50,000 partly at 10% and partly at 15%. His total income after a year was Rs.7000. How much did he invest at the rate of 10%?
A. Rs.40,000
B. Rs.40,000
C. Rs.12,000
D. Rs.20,000
Ans : B
48. A sum of money invested for a certain number of years at 8% p.a. simple interest grows to Rs.180. The same sum of money invested for the same number of years at 4% p.a. simple interest grows to Rs.120 only. For how many years was the sum invested?
A. 25 years
B. 40 years
C. 33 years and 4 months
D. Cannot be determined
Ans : A
49. How long will it take for a sum of money to grow from Rs.1250 to Rs.10,000, if it is invested at 12.5% p.a simple interest?
A. 8 years
B. 64 years
C. 72 years
D. 56 years
Ans : D
50. Rs. 5887 is divided between Shyam and Ram, such that Shyam's share at the end of 9 years is equal to Ram's share at the end of 11 years, compounded annually at the rate of 5%. Find the share of Shyam.
A. 2088
B. 2000
C. 3087
D. None of these
Ans : C

51. Find the coordinates of the point which divides the line joining (5, -2) and (9, 6) internally in the ratio 1 : 3.
A. (6, 0)
B. (6, 3)
C. (0, 6)
D. (3, 6)
Ans : A
52. Find the number of triangles in an octagon.
A. 326
B. 120
C. 56
D. Cannot be determined
Ans : C
53. Find the equation of a line whose intercepts are twice of the line 3x - 2y - 12 = 0
A. 3x - 2y = 24
B. 2x - 3y = 12
C. 2x - 3y = 24
D. None of these
Ans : A
54. Find the area of the sector covered by the hour hand after it has moved through 3 hours and the length of the hour hand is 7cm.
A. 77 sq.cm
B. 38.5 sq.cm
C. 35 sq.cm
D. 70 sq.cm
Ans : B
55. Find the area of the triangle whose vertices are (-6, -2), (-4, -6), (-2, 5).
A. 36
B. 18
C. 15
D. 30
Ans : C
56. A stairway 10ft high is such that each step accounts for half a foot upward and one-foot forward. What distance will an ant travel if it starts from ground level to reach the top of the stairway?
A. 30 ft
B. 33 ft
C. 10 ft
D. 29 ft
Ans : D
57. Each interior angle of a regular polygon is 120 degrees greater than each exterior angle. How many sides are there in the polygon?
A. 6
B. 8
C. 12
D. 3
Ans : C
58. What is the area of the largest triangle that can be fitted into a rectangle of length 'l' units and width 'w' units?
A. lw/3
B. (2lw)/3
C. (3lw)/4
D. (lw)/2
Ans : D
59. Which of the following is inCorrect?
A. An incentre is a point where the angle bisectors meet.
B. The median of any side of a triangle bisects the side at right angle.
C. The point at which the three altitudes of a triangle meet is the orthocentre
D. The point at which the three perpendicular bisectors meet is the centre of the circumcircle.
Ans : B
60. A and B are two points with the co-ordinates (-2, 0) and (0, 5). What is the length of the diagonal AC if AB form one of the sides of the square ABCD?
A. units
B. units
C. units
D. units
Ans : B
61. What is the measure of the circum radius of a triangle whose sides are 9, 40 and 41?
A. 6
B. 4
C. 24.5
D. 20.5
Ans : D
62. If the sum of the interior angles of a regular polygon measures up to 1440 degrees, how many sides does the polygon have?
A. 10 sides
B. 8 sides
C. 12 sides
D. 9 sides
Ans : A
63. If ABC is a right angle triangle with angle A = 900 and 2s = a + b + c, where a > b > c where notations have their usual meanings, then which one of the following is Correct?
A. (s - b) (s - c) > s (s - a)
B. (s - a) (s - c) > s (s - b)
C. (s - a) (s - b) < s (s - c)
D. 4s (s - a) (s - b) (s - c) = bc
Ans : C
64. What is the measure of in radius of the triangle whose sides are 24, 7 and 25?
A. 12.5
B. 3
C. 6
D. None of these
Ans : B
65. What is the circum radius of a triangle whose sides are 7, 24 and 25 respectively?
A. 18
B. 12.5
C. 12
D. 14
Ans : B
Quantitative Ability : Menstruation
66. A regular hexagon is inscribed in a circle of radius r cms. What is the perimeter of the regular hexagon?
A. 3r
B. 6r
C. r
D. 9r
Ans : B
67. A 4 cm cube is cut into 1 cm cubes. What is the percentage increase in the surface area after such cutting?
A. 4%
B. 300%
C. 75%
D. 400%
Ans : B
68. If the diagonal and the area of a rectangle are 25 m and 168 m2, what is the length of the rectangle?
A. 17 m
B. 31 m
C. 12 m
D. 24 m
Ans : D
69. The surface area of the three coterminous faces of a cuboid are 6, 15, 10 sq.cm respectively. Find the volume of the cuboid.
A. 30
B. 20
C. 40
D. 35
Ans : A
70. If each interior angle of a regular polygon is 150 degrees, then it is
A. Octagon
B. Decagon
C. Dodecagon
D. Tetrahedron
Ans : C
Menstruation
71. A 5 cm cube is cut into as many 1 cm cubes as possible. What is the ratio of the surface area of the larger cube to that of the sum of the surface areas of the smaller cubes?
A. 1 : 6
B. 1 : 5
C. 1 : 25
D. 1 : 125
Ans : B
72. If the sides of a triangle measure 72, 75 and 21, what is the measure of its in radius?
A. 37.5
B. 24
C. 9
D. 15
Ans : C
73. The circumference of the front wheel of a cart is 30 ft long and that of the back wheel is 36 ft long. What is the distance travelled by the cart, when the front wheel has done five more revolutions than the rear wheel?
A. 20 ft
B. 25 ft
C. 750 ft
D. 900 ft
Ans : D
74. The area of a square field is 24200 sq m. How long will a lady take to cross the field diagonally at the rate of 6.6 km/hr?
A. 3 minutes
B. 2 minutes
C. 2.4 minutes
D. 2 minutes 40 seconds
Ans : B
Quantitative Ability : Trignometry
75. a and b are the lengths of the base and height of a right angled triangle whose hypotenuse is h. If the values of a and b are positive integers, which of the following cannot be a value of the square of the hypotenuse?
A. 13
B. 23
C. 37
D. 41
Ans : B
76. The angle of elevation of the top of a tower 30 m high, from two points on the level ground on its opposite sides are 45 degrees and 60 degrees. What is the distance between the two points?
A. 30
B. 51.96
C. 47.32
D. 81.96
Ans : C
77. What is the value of cot 15o + cot 75o + cot 135o - cosec 30o?
A. 3
B. Infinity
C. 1
D. None of these
Ans : C
Ratio And Proportion
78. Rs.432 is divided amongst three workers A, B and C such that 8 times A's share is equal to 12 times B's share which is equal to 6 times C's share. How much did A get?
A. Rs.192
B. Rs.133
C. Rs.144
D. Rs.128
Ans : C
79. If 20 men or 24 women or 40 boys can do a job in 12 days working for 8 hours a day, how many men working with 6 women and 2 boys take to do a job four times as big working for 5 hours a day for 12 days?
A. 8 men
B. 12 men
C. 2 men
D. 24 men
Ans : C
80. Two cogged wheels of which one has 32 cogs and other 54 cogs, work into each other. If the latter turns 80 times in three quarters of a minute, how often does the other turn in 8 seconds?
A. 48
B. 135
C. 24
D. None of these
Ans : C
Ratio And Proportion
81. The monthly incomes of A and B are in the ratio 4 : 5, their expenses are in the ratio 5 : 6. If 'A' saves Rs.25 per month and 'B' saves Rs.50 per month, what are their respective incomes?
A. Rs.400 and Rs.500
B. Rs.240 and Rs.300
C. Rs.320 and Rs.400
D. Rs.440 and Rs.550
Ans : A
82. The proportion of milk and water in 3 samples is 2:1, 3:2 and 5:3. A mixture comprising of equal quantities of all 3 samples is made. The proportion of milk and water in the mixture is
A. 2:1
B. 5:1
C. 99:61
D. 227:133
Ans : D
83. A group of workers can do a piece of work in 24 days. However as 7 of them were absent it took 30 days to complete the work. How many people actually worked on the job to complete it?
A. 35
B. 30
C. 28
D. 42
Ans : C
84. A, B and C play cricket. A's runs are to B's runs and B's runs are to C's as 3:2. They get altogether 342 runs. How many runs did A make?
A. 162
B. 108
C. 72
D. None of these
Ans : A
85. The monthly salaries of two persons are in the ratio of 4:7. If each receives an increase of Rs.25 in the salary, the ratio is altered to 3: 5. Find their respective salaries.
A. 120 and 210
B. 80 and 140
C. 180 and 300
D. 200 and 350
Ans : D
86. A fort has provisions for 60 days. If after 15 days 500 men strengthen them and the food lasts 40 days longer, how many men are there in the fort?
A. 3500
B. 4000
C. 6000
D. None of these
Ans : B
87. The ratio of marks obtained by vinod and Basu is 6:5. If the combined average of their percentage is 68.75 and their sum of the marks is 275, find the total marks for which exam was conducted.
A. 150
B. 200
C. 400
D. None of these.
Ans : B
88. The present ages of A and B are as 6 : 4. Five years ago their ages were in the ratio 5 : 3. Find their present ages.
A. 42, 28
B. 36, 24
C. 30, 20
D. 25, 15
Ans : C
89. A, B and C enter into a partnership by investing Rs.3600, Rs.4400 and Rs.2800. A is a working partner and gets a fourth of the profit for his services and the remaining profit is divided amongst the three in the rate of their investments. What is the amount of profit that B gets if A gets a total of Rs. 8000?
A. 4888.88
B. 9333.33
C. 4000
D. 3666.66
Ans : A
90. A, B and C, each of them working alone can complete a job in 6, 8 and 12 days respectively. If all three of them work together to complete a job and earn Rs.2340, what ill be C's share of the earnings?
A. Rs.520
B. Rs.1080
C. Rs.1170
91. Rs.630 A 20 litre mixture of milk and water contains milk and water in the ratio 3 : 2. 10 litres of the mixture is removed and replaced with pure milk and the operation is repeated once more. At the end of the two removal and replacement, what is the ratio of milk and water in the resultant mixture?
A. 17 : 3
B. 9 : 1
C. 3 : 17
D. 5 : 3
Ans : B
92. In what ratio must a person mix three kinds of tea costing Rs.60/kg, Rs.75/kg and Rs.100 /kg so that the resultant mixture when sold at Rs.96/kg yields a profit of 20%?
A. 1 : 2 : 4
B. 3 : 7 : 6
C. 1 : 4 : 2
D. None of these
Ans : C
93. A merchant mixes three varieties of rice costing Rs.20/kg, Rs.24/kg and Rs.30/kg and sells the mixture at a profit of 20% at Rs.30 / kg. How many kgs of the second variety will be in the mixture if 2 kgs of the third variety is there in the mixture?
A. 1 kg
B. 5 kgs
C. 3 kgs
D. 6 kgs
Ans : B
94. How many litres of water should be added to a 30 litre mixture of milk and water containing milk and water in the ratio of 7 : 3 such that the resultant mixture has 40% water in it?
A. 7 litres
B. 10 litres
C. 5 litres
D. None of these
Ans : C
95. How many kgs of Basmati rice costing Rs.42/kg should a shopkeeper mix with 25 kgs of ordinary rice costing Rs.24 per kg so that he makes a profit of 25% on selling the mixture at Rs.40/kg?
A. 20 kgs
B. 12.5 kgs
C. 16 kgs
D. 200 kgs
Ans : A
96. How many litres of a 12 litre mixture containing milk and water in the ratio of 2 : 3 be replaced with pure milk so that the resultant mixture contains milk and water in equal proportion?
A. 4 litres
B. 2 litres
C. 1 litre
D. 1.5 litres
Ans : B
97. A sample of x litres from a container having a 60 litre mixture of milk and water containing milk and water in the ratio of 2 : 3 is replaced with pure milk so that the container will have milk and water in equal proportions. What is the value of x?
A. 6 litres
B. 10 litres
C. 30 litres
D. None of these
Ans : B
98. A zookeeper counted the heads of the animals in a zoo and found it to be 80. When he counted the legs of the animals he found it to be 260. If the zoo had either pigeons or horses, how many horses were there in the zoo?
A. 40
B. 30
C. 50
D. 60
Ans : C
99. From a cask of milk containing 30 litres, 6 litres are drawn out and the cask is filled up with water. If the same process is repeated a second, then a third time, what will be the number of litres of milk left in the cask?
A. 0.512 liters
B. 12 liters
C. 14.38 liters
D. 15.36 liters
Ans : D
100. In a km race, A gives B a start of 20 seconds and beats him by 40m. However, when he gives B a start of 25 seconds they finish in a dead heat. What is A's speed in m/sec?
A. 12.5 m/sec
B. 20 m/sec
C. 8 m/sec
D. 10 m/sec
Ans : D

101. In a kilometre race, A can give B a start of 100 m or 15 seconds. How long does A take to complete the race?
A. 150 seconds
B. 165 seconds
C. 135 seconds
D. 66.67 seconds
Ans : C
102. A gives B a start of 10 metres in a 100 metre race and still beats him by 1.25 seconds. How long does B take to complete the 100 metre race if A runs at the rate of 10 m/sec?
A. 8 seconds
B. 10 seconds
C. 16.67 seconds
D. 12.5 seconds
Ans : D
103. A predator is chasing its prey. The predator takes 4 leaps for every 6 leaps of the prey and the predator covers as much distance in 2 leaps as 3 leaps of the prey. Will the predator succeed in getting its food?
A. Yes
B. In the 6th leap
C. Never
D. Cannot determine
Ans : D
104. A skating champion moves along the circumference of a circle of radius 21 meters in 44 seconds. How many seconds will it take her to move along the perimeter of a hexagon of side 42 meters?
A. 56
B. 84
C. 64
D. 48
Ans : B
105. A runs 13/5 times as fast as B. If A gives a start of 240m, how far must the post be so that A and B might reach at the same time.
A. 390 m
B. 330 m
C. 600 m
D. 720 m
Ans : A
106. A gives B a start of 30 seconds in a km race and still beats him by 20 m. However, when he gives B a start of 35 seconds, they finish the race in a dead heat. How long does A take to run the km?
A. 250 seconds
B. 285 seconds
C. 220 seconds
D. 215 seconds
Ans : D
107. A can give B 20 points, A can give C 32 points and B can give C 15 points. How many points make the game?
A. 150
B. 200
C. 100
D. 170
Ans : D
108. A can give B a start of 50 metres or 10 seconds in a kilometer race. How long does A take to complete the race?
A. 200 seconds
B. 140 seconds
C. 220 seconds
D. 190 seconds
Ans : D
109. Three runners A, B and C run a race, with runner A finishing 12 meters ahead of runner B and 18 meters ahead of runner C, while runner B finishes 8 meters ahead of runner C. Each runner travels the entire distance at a constant speed.
What was the length of the race?
A. 36 meters
B. 48 meters
C. 60 meters
D. 72 meters
Ans : B
110. P can give Q a start of 20 seconds in a kilometer race. P can give R a start of 200 meters in the same kilometer race. And Q can give R a start of 20 seconds in the same kilometer race. How long does P take to run the kilometer?
A. 200 seconds
B. 240 seconds
C. 160 seconds
D. 140 seconds
Ans : C
Number System
111. Two numbers when divided by a certain divisor leave remainders of 431 and 379 respectively. When the sum of these two numbers is divided by the same divisor, the remainder is 211. What is the divisor?
A. 599
B. 1021
C. 263
D. Cannot be determined
Ans : A
112. How many zeros contained in 100!?
A. 100
B. 24
C. 97
D. Cannot be determined
Ans : B
113. Which is greater of the two
A. 2300
B. 3200
C. Both are equal
D. Cannot be determined
Ans : B
114. What is the value of M and N respectively? If M39048458N is divisible by 8 & 11; Where M & N are single digit integers?
A. 7, 8
B. 8, 6
C. 6, 4
D. 5, 4
Ans : C
115. When 26854 and 27584 are divided by a certain two digit prime number, the remainder obtained is 47. Which of the following choices is a possible value of the divisor?
A. 61
B. 71
C. 73
D. 89
Ans : C
116. Find the G.C.D of 12x2y3z2, 18x3y2z4, and 24xy4z3
A. 6xy2z2
B. 6x3y4z3
C. 24xy2z2
D. 18x2y2z3
Ans : A
117. Find the G.C.D of 12x2y3z2, 18x3y2z4, and 24xy4z3
A. 6xy2z2
B. 6x3y4z3
C. 24xy2z2
D. 18x2y2z3
Ans : A
118. The 7th digit of (202)3is
A. 2
B. 4
C. 8
D. 6
Ans : C
119. A railway half ticket costs half the full fare and the reservation charge is the same on half ticket as on full ticket. One reserved first class ticket from Chennai to Trivandrum costs Rs. 216 and one full and one half reserved first class tickets cost Rs. 327. What is the basic first class full fare and what is the reservation charge?
A. Rs. 105 and Rs. 6
B. Rs. 216 and Rs. 12
C. Rs. 210 and Rs. 12
D. Rs. 210 and Rs. 6
Ans :D
120. Find the range of real values of x satisfying the inequalities 3x - 2 > 7 and 4x - 13 > 15.
A. x > 3
B. x > 7
C. x < 7
D. x < 3
Ans :B
121. How many different factors are there for the number 48, excluding 1 and 48?
A. 12
B. 4
C. 8
D. None of these
Ans :C
122. What is the remainder when 9 + 92 + 93 + .... + 98 is divided by 6?
A. 3
B. 2
C. 0
D. 5
Ans :C
123. The sum of the first 100 numbers, 1 to 100 is divisible by
A. 2, 4 and 8
B. 2 and 4
C. 2 only
D. None of these
Ans :C
124. The sum of the first 100 numbers, 1 to 100 is divisible by
A. 2, 4 and 8
B. 2 and 4
C. 2 only
D. None of these
Ans :C
125. For what value of 'n' will the remainder of 351n and 352n be the same when divided by 7?
A. 2
B. 3
C. 6
D. 4
Ans :B
126. A person starts multiplying consecutive positive integers from 20. How many numbers should he multiply before the will have result that will end with 3 zeroes?
A. 11
B. 10
C. 6
D. 5
Ans :C
127. What is the minimum number of square marbles required to tile a floor of length 5 metres 78 cm and width 3 metres 74 cm?
A. 176
B. 187
C. 54043
D. 748
Ans :B
128. What number should be subtracted from x3 + 4x2 - 7x + 12 if it is to be perfectly divisible by x + 3?
A. 42
B. 39
C. 13
D. None of these
Ans :A
129. Let x, y and z be distinct integers. x and y are odd and positive, and z is even and positive. Which one of the following statements cannot be true?
A. (x-z)2y is even
B. (x-z)y2 is odd
C. (x-z)y is odd
D. (x-y)2z is even
Ans :A
130. Anita had to do a multiplication. Instead of taking 35 as one of the multipliers, she took 53. As a result, the product went up by 540. What is the new product?
A. 1050
B. 540
C. 1440
D. 1590
Ans :D
131. Let n be the number of different 5 digit numbers, divisible by 4 with the digits 1, 2, 3, 4, 5 and 6, no digit being repeated in the numbers. What is the value of n?
A. 144
B. 168
C. 192
D. None of these
Ans :C
132. Find the greatest number of five digits, which is exactly divisible by 7, 10, 15, 21 and 28.
A. 99840
B. 99900
C. 99960
D. 99990
Ans :C
133. When 242 is divided by a certain divisor the remainder obtained is 8. When 698 is divided by the same divisor the remainder obtained is 9. However, when the sum of the two numbers 242 and 698 is divided by the divisor, the remainder obtained is 4. What is the value of the divisor?
A. 11
B. 17
C. 13
D. 23
Ans :C
134. A number when divided by a divisor leaves a remainder of 24. When twice the original number is divided by the same divisor, the remainder is 11. What is the value of the divisor?
A. 13
B. 59
C. 35
D. 37
Ans : D
135. Given A = 265 and B = (264+263+262+...+20)
A. B is 264 larger than A
B. A and B are equal
C. B is larger than A by 1
D. A is larger than B by 1
Ans :D
136. The sum of third and ninth term of an A.P is 8. Find the sum of the first 11 terms of the progression.
A. 44
B. 22
C. 19
D. None of these
Ans :A
137. If (x + 2)2 = 9 and (y + 3)2 = 25, then the maximum value of x / y is
A. 1 / 2
B. 5 / 2
C. 5 / 8
D. 1 / 8
Ans :C
138. If p and q are the roots of the equation x2 - bx + c = 0, then what is the equation if the roots are (pq + p + q) and (pq - p - q)?
A. x2 - 2cx + (c2 - b2) = 0
B. x2 - 2bx + (b2 + c2) = 0
C. Bcx2 - 2(b+c)x + c2 = 0
D. x2 + 2bx - (c2 - b2) = 0
Correct answer- A
139. A piece of equipment cost a certain factory Rs. 600,000. If it depreciates in value, 15% the first year, 13.5% the next year, 12% the third year, and so on, what will be its value at the end of 10 years, all percentages applying to the original cost?
A. 2,00,000
B. 1,05,000
C. 4,05,000
D. 6,50,000
Correct answer- B
140. Solve the inequality 33x-2 > 1
A. x > 1
B. x > 3
C. x > 2/3
D. x > 1/3
Ans :C
141. The largest number amongst the following that will perfectly divide 101100 - 1 is
A. 100
B. 10000
C. 100100
D. 100000
Ans : 2
142. What is the highest power of 7 in 5000!? (5000! means factorial 5000)
A. 4998
B. 714
C. 832
D. 816
Ans :C
143. What is the total number of different divisors including 1 and the number that can divide the number 6400?
A. 24
B. 27
C. 54
D. 68
Ans :B
144. How many four digit numbers exist which can be formed by using the digits 2, 3, 5 and 7 once only such that they are divisible by 25?
A. 4! - 3!
B. 4
C. 8
D. 6
Ans :B
145. A certain number when successfully divided by 8 and 11 leaves remainders of 3 and 7 respectively. What will be remainder when the number is divided by the product of 8 and 11, viz 88?
A. 3
B. 21
C. 59
D. 68
Ans :C
146. 'a' and 'b' are the lengths of the base and height of a right angled triangle whose hypotenuse is 'h'. If the values of 'a' and 'b' are positive integers, which of the following cannot be a value of the square of the hypotenuse?
A. 13
B. 23
C. 37
D. 41
Ans :B
147. What is the reminder when 91 + 92 + 93 + ...... + 99 is divided by 6?
A. 0
B. 3
C. 4
D. None of these
Ans :B
148. How many times will the digit '0' appear between 1 and 10,000?
A. 4000
B. 4003
C. 2893
D. 3892
Ans :C
149. What is the total number of different divisors of the number 7200?
A. 20
B. 4
C. 54
D. 32
Ans :C
150. What is the least number that should be multiplied to 100! to make it perfectly divisible by 350?
A. 144
B. 72
C. 108
D. 216
Ans :B
151. There are 2 brothers among a group of 20 persons. In how many ways can the group be arranged around a circle so that there is exactly one person between the two brothers?
A. 2 * 19!
B. 18! * 18
C. 19! * 18
D. 2 * 18!
Ans : D
152. How many words can be formed by re-arranging the letters of the word ASCENT such that A and T occupy the first and last position respectively?
A. 5!
B. 4!
C. 6! - 2!
D. 6! / 2!
Ans : B
153. There are 12 yes or no questions. How many ways can these be answered?
A. 1024
B. 2048
C. 4096
D. 144
Ans : C
154. How many ways can 4 prizes be given away to 3 boys, if each boy is eligible for all the prizes?
A. 256
B. 12
C. 81
D. None of these
Ans : C
155. A team of 8 students goes on an excursion, in two cars, of which one can seat 5 and the other only 4. In how many ways can they travel?
A. 9
B. 26
C. 126
D. 3920
Ans : C
156. How many numbers are there between 100 and 1000 such that atleast one of their digits is 6?
A. 648
B. 258
C. 654
D. 252
Ans : D
157. How many ways can 10 letters be posted in 5 post boxes, if each of the post boxes can take more than 10 letters?
A. 510
B. 105
C. 10P5
D. 10C5
Ans : A
158. In how many ways can the letters of the word EDUCATION be rearranged so that the relative position of the vowels and consonants remain the same as in the word EDUCATION?
A. 9!/4
B. 9!/(4!*5!)
C. 4!*5!
D. None of these
159. Ans : C How many words of 4 consonants and 3 vowels can be made from 12 consonants and 4 vowels, if all the letters are different?
A. 16C7 * 7!
B. 12C4 * 4C3 * 7!
C. 12C3 * 4C4
D. 12C4 * 4C3
Ans : B
160. In how many ways can 5 letters be posted in 3 post boxes, if any number of letters can be posted in all of the three post boxes?
A. 5C3
B. 5P3
C. 53
D. 35
Ans : D
161. How many number of times will the digit '7' be written when listing the integers from 1 to 1000?
A. 271
B. 300
C. 252
D. 304
Ans : B
162. There are 6 boxes numbered 1, 2,...6. Each box is to be filled up either with a red or a green ball in such a way that at least 1 box contains a green ball and the boxes containing green balls are consecutively numbered. The total number of ways in which this can be done is
A. 5
B. 21
C. 33
D. 60
Ans : B
163. What is the value of 1*1! + 2*2! + 3!*3! + ............ n*n!, where n! means n factorial or n(n-A(n-2)...1
A. n(n-A(n-A!))
B. (n+A!)/(n(n-A))
C. (n+A! - n!)
D. (n + A! - 1!)
Ans : D
164. There are 5 Rock songs, 6 Carnatic songs and 3 Indi pop songs. How many different albums can be formed using the above repertoire if the albums should contain at least 1 Rock song and 1 Carnatic song?
A. 15624
B. 16384
C. 6144
D. 240
Ans : A
165. In how many ways can the letters of the word MANAGEMENT be rearranged so that the two As do not appear together?
A. 10! - 2!
B. 9! - 2!
C. 10! - 9!
D. None of these
Ans : D
166. How many five digit numbers can be formed using the digits 0, 1, 2, 3, 4 and 5 which are divisible by 3, without repeating the digits?
A. 15
B. 96
C. 216
D. 120
Ans : C
167. How many words can be formed by re-arranging the letters of the word PROBLEMS such that P and S occupy the first and last position respectively?
A. 8! / 2!
B. 8! - 2!
C. 6!
D. 8! - 2*7!
Ans : C
168. Four dice are rolled simultaneously. What is the number of possible outcomes in which at least one of the die shows 6?
A. 6! / 4!
B. 625
C. 671
D. 1296
Ans : C

169. In how many ways can 15 people be seated around two round tables with seating capacities of 7 and 8 people?
A. 15!/(8!)
B. 7!*8!
C. (15C8)*6!*7!
D. 2*(15C7)*6!*7!
Ans : C
170. If the letters of the word CHASM are rearranged to form 5 letter words such that none of the word repeat and the results arranged in ascending order as in a dictionary what is the rank of the word CHASM?
A. 24
B. 31
C. 32
D. 30
Ans : C
Percentage
171. A trader makes a profit equal to the selling price of 75 articles when he sold 100 of the articles. What % profit did he make in the transaction?
A. 33.33%
B. 75%
C. 300%
D. 150%
Ans : C
172. A merchant buys two articles for Rs.600. He sells one of them at a profit of 22% and the other at a loss of 8% and makes no profit or loss in the end. What is the selling price of the article that he sold at a loss?
A. Rs. 404.80
B. Rs. 440
C. Rs. 536.80
D. Rs. 160
Ans : A
173. A trader professes to sell his goods at a loss of 8% but weights 900 grams in place of a kg weight. Find his real loss or gain per cent.
A. 2% loss
B. 2.22% gain
C. 2% gain
D. None of these
Ans : B
174. Rajiv sold an article for Rs.56 which cost him Rs.x. If he had gained x% on his outlay, what was his cost?
A. Rs. 40
B. Rs. 45
C. Rs. 36
D. Rs. 28
Ans : A
175. A trader buys goods at a 19% Aount on the label price. If he wants to make a profit of 20% after allowing a Aount of 10%, by what % should his marked price be greater than the original label price?
A. +8%
B. -3.8%
C. +33.33%
D. None of these
Ans : A
176. If apples are bought at the rate of 30 for a rupee. How many apples must be sold for a rupee so as to gain 20%?
A. 28
B. 25
C. 20
D. 22
Ans : B
177. One year payment to the servant is Rs. 200 plus one shirt. The servant leaves after 9 months and recieves Rs. 120 and a shirt. Then find the price of the shirt.
A. Rs. 80
B. Rs. 100
C. Rs. 120
D. Cannot be determined
Ans : C
178. Two merchants sell, each an article for Rs.1000. If Merchant A computes his profit on cost price, while Merchant B computes his profit on selling price, they end up making profits of 25% respectively. By how much is the profit made by Merchant B greater than that of Merchant A?
A. Rs.66.67
B. Rs. 50
C. Rs.125
D. Rs.200
Ans : B
179. A merchant marks his goods in such a way that the profit on sale of 50 articles is equal to the selling price of 25 articles. What is his profit margin?
A. 25%
B. 50%
C. 100%
D. 66.67%
Ans : C
180. A merchant marks his goods up by 75% above his cost price. What is the maximum % Aount that he can offer so that he ends up selling at no profit or loss?
A. 75%
B. 46.67%
C. 300%
D. 42.85%
Ans : D
181. Rajesh traveled from city A to city B covering as much distance in the second part as he did in the first part of this journey. His speed during the second part was twice as that of the speed during the first part of the journey. What is his average speed of journey during the entire travel?
A. His average speed is the harmonic mean of the individual speeds for the two parts.
B. His average speed is the arithmetic mean of the individual speeds for the two parts.
C. His average speed is the geometric mean of the individual speeds for the two parts.
D. Cannot be determined.
Ans : B
182. Two boys begin together to write out a booklet containing 535 lines. The first boy starts with the first line, writing at the rate of 100 lines an hour; and the second starts with the last line then writes line 534 and so on, backward proceeding at the rate of 50 lines an hour. At what line will they meet?
A. 356
B. 277
C. 357
D. 267
Ans : C
183. A man and a woman 81 miles apart from each other, start travelling towrds each other at the same time. If the man covers 5 miles per hour to the women's 4 miles per hour, how far will the woman have travelled when they meet?
A. 27
B. 36
C. 45
D. None of these.
Ans : B
184. The speed of a motor boat itself is 20 km/h and the rate of flow of the river is 4 km/h. Moving with the stream the boat went 120 km. What distance will the boat cover during the same time going against the stream?
A. 80 km
B. 180 km
C. 60 km
D. 100 km
Ans : A
185. Two friends A and B run around a circular track of length 510 metres, starting from the same point, simultaneously and in the same direction. A who runs faster laps B in the middle of the 5th round. If A and B were to run a 3 km race long race, how much start, in terms of distance, should A give B so that they finish the race in a dead heat?
A. 545.45 metres
B. 666.67 metres
C. 857.14 metres
D. Cannot be determined
Ans : B
186. I travel the first part of my journey at 40 kmph and the second part at 60 kmph and cover the total distance of 240 km to my destination in 5 hours. How long did the first part of my journey last?
A. 4 hours
B. 2 hours
C. 3 hours
D. 2 hours 24 minutes
Ans : C
187. By walking at 3/4th of his usual speed, a man reaches office 20 minutes later than usual. What is his usual time?
A. 30 min
B. 60 min
C. 70 min
D. 50 min
Ans : B
188. A passenger train covers the distance between stations X and Y, 50 minutes faster than a goods train. Find this distance if the average speed of the passenger train is 60 kmph and that of goods train is 20 kmph.
A. 20 kms
B. 25 kms
C. 45 kms
D. 40 kms
Ans : B
Yana and Gupta leave points x and y towards y and x respectively simultaneously and travel in the same route. After Speed, Time and Distance
191. Train A traveling at 60 km/hr leaves Mumbai for Delhi at 6 P.M. Train B traveling at 90 km/hr also leaves Mumbai for Delhi at 9 P.M. Train C leaves Delhi for Mumbai at 9 P.M. If all three trains meet at the same time between Mumbai and Delhi, what is the speed of Train C if the distance between Delhi and Mumbai is 1260 kms?
A. 60 km/hr
B. 90 km/hr
C. 120 km/hr
D. 135 km/hr
Ans : C
192. Two trains, 200 and 160 meters long take a minute to cross each other while traveling in the same direction and take only 10 seconds when they cross in opposite directions. What are the speeds at which the trains are traveling?
A. 21 m/s; 15 m/s
B. 30 m/s; 24 m/s
C. 18 m/s; 27 m/s
D. 15 m/s; 24 m/s
Ans : A
193. An express train traveling at 72 km/hr speed crosses a goods train traveling at 45 km/hr speed in the opposite direction in half a minute. Alternatively, if the express train were to overtake the goods train, how long will it take to accomplish the task. Assume that the trains continue to travel at the same respective speeds as mentioned in case 1.
A. Cannot be determined
B. 30 seconds
C. 150 seconds
D. 130 seconds
Ans : D
194. A train travels at an average speed of 90 km/hr without any stoppages. However, its average speed decrease to 60km/hr on account of stoppages. On an average, how many minutes per hour does the train stop?
A. 12 minutes
B. 18 minutes
C. 24 minutes
D. 20 minutes
Ans : D
195. Two trains A and B start simultaneously from stations X and Y towards each other respectively. After meeting at a point between X and Y, train A reaches station Y in 9 hours and train B reaches station X in 4 hours from the time they have met each other. If the speed of train A is 36 km/hr, what is the speed of train B?
A. 24 km/hr
B. 54 km/hr
C. 81 km/hr
D. 16 km/hr
Ans : B
196. A man moves from A to B at the rate of 4 km/hr. Had he moved at the rate of 3.67 km/hr, he would have taken 3 hours more to reach the destination. What is the distance between A and B?
A. 33 kms
B. 132 kms
C. 36 kms
D. 144 kms
Ans : B
197. A ship develops a leak 12 km from the shore. Despite the leak, the ship is able to move towards the shore at a speed of 8 km/hr. However, the ship can stay afloat only for 20 minutes. If a rescue vessel were to leave from the shore towards the ship, and it takes 4 minutes to evacuate the crew and passengers of the ship, what should be the minimum speed of the rescue vessel in order to be able to successfully rescue the people aboard the ship?
A. 53 km/hr
B. 37 km/hr
C. 28 km/hr
D. 44 km/hr
Ans : B
198. A man driving his bike at 24 kmph reaches his office 5 minutes late. Had he driven 25% faster on an average he would have reached 4 minutes earlier than the scheduled time. How far is his office?
A. 24 km
B. 72 km
C. 18 km
D. Data Insufficient
Ans : C
199. When an object is dropped, the number of feet N that it falls is given by the formula N = ½gt2 where t is the time in seconds from the time it was dropped and g is 32.2. If it takes 5 seconds for the object to reach the ground, how many feet does it fall during the last 2 seconds?
A. 64.4
B. 96.6
C. 161.0
D. 257.6
Ans : D
200. If the wheel of a bicycle makes 560 revolutions in travelling 1.1 km, what is its radius?
A. 31.25 cm
B. 37.75 cm
C. 35.15 cm
D. 11.25 cm
Ans : A
189. meeting each other on the way, Yana takes 4 hours to reach her destination, while Gupta takes 9 hours to reach his destination. If the speed of Yana is 48 km/hr, what is the speed of Gupta?
A. 72 kmph
B. 32 mph
C. 20 mph
D. None of these
Ans : C
201. Ram covers a part of the journey at 20 kmph and the balance Two workers A and B manufactured a batch of identical parts. A worked for 2 hours and B worked for 5 hours and they did half the job. Then they worked together for another 3 hours and they had to do (1/20)th of the job. How much time does B take to complete the job, if he worked alone?
A. 24 hours
B. 12 hours
C. 15 hours
D. 30 hours
Ans : C
202. Pipe A can fill a tank in 'a' hours. On account of a leak at the bottom of the tank it takes thrice as long to fill the tank. How long will the leak at the bottom of the tank take to empty a full tank, when pipe A is kept closed?
A. (3/2)a hours
B. (2/3)a
C. (4/3)a
D. (3/4)a
Ans : A
203. A and B working together can finish a job in T days. If A works alone and completes the job, he will take T + 5 days. If B works alone and completes the same job, he will take T + 45 days. What is T?
A. 25
B. 60
C. 15
D. None of these
Ans : C
204. A man can do a piece of work in 60 hours. If he takes his son with him and both work together then the work is finished in 40 hours. How long will the son take to do the same job, if he worked alone on the job?
A. 20 hours
B. 120 hours
C. 24 hours
D. None of these
Ans : B
205. A, B and C can do a work in 5 days, 10 days and 15 days respectively. They started together to do the work but after 2 days A and B left. C did the remaining work (in days)
A. 1
B. 3
C. 5
D. 4
Ans : D
206. X alone can do a piece of work in 15 days and Y alone can do it in 10 days. X and Y undertook to do it for Rs. 720. With the help of Z they finished it in 5 days. How much is paid to Z?
A. Rs. 360
B. Rs. 120
C. Rs. 240
D. Rs. 300
Ans : B
207. Ram starts working on a job and works on it for 12 days and completes 40% of the work. To help him complete the work, he employs Ravi and together they work for another 12 days and the work gets completed. How much more efficient is Ram than Ravi?
A. 50%
B. 200%
C. 60%
D. 100%
Ans : D
208. A red light flashes 3 times per minute and a green light flashes 5 times in two minutes at regular intervals. If both lights start flashing at the same time, how many times do they flash together in each hour?
A. 30
B. 24
C. 20
D. 60
Ans : A
209. A and B can do a piece of work in 21 and 24 days respectively. They started the work together and after some days A leaves the work and B completes the remaining work in 9 days. After how many days did A leave?
A. 5
B. 7
C. 8
D. 6
Ans : B
210. Ram, who is half as efficient as Krish, will take 24 days to complete a work if he worked alone. If Ram and Krish worked together, how long will they take to complete the work?
A. 16 days
B. 12 days
C. 8 days
D. 18 days
Ans : C

Probability
211. A number is selected at random from first thirty natural numbers. What is the chance that it is a multiple of either 3 or 13?
A. 17/30
B. 2/5
C. 11/30
D. 4/15
Ans : B
212. What is the probability of getting at least one six in a single throw of three unbiased dice?
A. 1 / 6
B. 125 / 216
C. 1 / 36
D. 91 / 216
Ans : D
213. What is the probability that a two digit number selected at random will be a multiple of '3' and not a multiple of '5'?
A. 2 / 15
B. 4 / 15
C. 1 / 15
D. 4 / 90
Ans : B
214. A man bets on number 16 on a roulette wheel 14 times and losses each time. On the 15th span he does a quick calculation and finds out that the number 12 had appeared twice in the 14 spans and is therefore, unable to decide whether to bet on 16 or 12 in the 15th span. Which will give him the best chance and what are the odds of winning on the bet that he takes? (Roulette has numbers 1 to 36)
A. 16; 22 : 14
B. 12; 72 : 1
C. 12; 7 : 1
D. Either; 35 : 1
Ans : D
215. Two squares are chosen at random on a chessboard. What is the probability that they have a side in common?
A. 1 / 18
B. 64 / 4032
C. 63 / 64
D. 1 / 9
Ans : A
216. When two dice are thrown simultaneously, what is the probability that the sum of the two numbers that turn up is less than 11?
A. 5 / 6
B. 11 / 12
C. 1 / 6
D. 1 / 12
Ans : B
217. When 4 dice are thrown, what is the probability that the same number appears on each of them?
A. 1/36
B. 1/18
C. 1/216
D. 1/5
Ans : C
218. An experiment succeeds twice as often as it fails. What is the probability that in the next 5 trials there will be four successes?
A. 0
B. (2/3)^4
C. 5*((2/3)^4)*(1/3)
D. ((2/3)^4)*(1/3)
Ans : C
219. An anti aircraft gun can fire four shots at a time. If the probabilities of the first, second, third and the last shot hitting the enemy aircraft are 0.7, 0.6, 0.5 and 0.4, what is the probability that four shots aimed at an enemy aircraft will bring the aircraft down?
A. 0.084
B. 0.916
C. 0.036
D. 0.964
Ans : A
220. A can complete a project in 20 days and B can complete the same project in 30 days. If A and B start working on the project together and A quits 10 days before the project is completed, in how many days will the project be completed?
A. 18 days
B. 27 days
C. 26.67 days
D. 16 days
Ans : A
Miscellaneous
221. Which amongst the following investments has the lowest return?
a. 7% Rs.100 shares at Rs.120
b. 8% Rs.10 shares at 13.50
c. 9% Rs.50 shares at Rs.54
A. a
B. b
C. c
D. a and c
Ans : A
222. The classic problem of a monkey and the greased flag pole . The height of a certain flag pole is 30 feet. Grease is applied to the pole. A monkey attempts to climb the pole. It climbs 3 feet every second but slips down 2 ft in the next second. When will the monkey reach the top of the flag pole?
A. 56 secs
B. 27 secs
C. 60 secs
D. 55 secs
Ans : D
223. How many squares can be formed using the checkered 1 * 1 squares in a normal chessboard?
A. 64
B. 204
C. 1296
D. 65
Ans : B
224. Solve for real 'x' if :
A. x > 9
B. 0 < x < 9
C. x < 0
D. None of these
Ans : B
225. If log 2 = 0. 3010, then find how many digits are contained in the number 256.
A. 15
B. 16
C. 17
D. Cannot be determined
Ans : C
226. A gentleman buys every year Bank's cash certificates of value exceeding the last year's purchase by Rs. 300. After 20 years, he finds that the total value of the certificates purchased by him is Rs. 83,000. Find the value of the certificates purchased by him in the 13th year.
A. Rs. 4900
B. Rs. 6900
C. Rs. 1300
D. None of these.
Ans : A
227. From the following choices what is the equation of a line whose x intercept is half as that of the line 3x + 4y = 12 and y intercept is twice as that of the same line.
A. 3x + 8y = 24
B. 8x + 3y = 24
C. 16x + 3y = 24
D. 3x + y = 6
Ans : D
228. Pipe A can fill a tank completely in 5 hours. However, on account of a leak at the bottom of the tank, it takes 3 more hours to fill the tank. How long will the leak take to empty a full tank, when pipe A is shut?
A. 13 hours 20 minutes
B. 7.5 hours
C. 14 hours 40 minutes
D. None of these
Ans : A
229. The sum of the first 50 terms common to the series 15,19,23 ... and 14,19,24 ... is
A. 25450
B. 24550
C. 50900
D. Cannot be determined
Ans : A
230. For what values of 'x' will the function be defined in the real domain?
A. -10 < x < 4
B. 4 < x < 10
C. x does not lie between the closed interval 10 and 4
D. x does not lie between the open interval 4 and 10
Ans : D
231. Find the sum of all the integers which are multiples of 7 and lie between 200 and 400.
A. 8729
B. 8700
C. 8428
D. None of these
Ans : A
232. How many digits will the number 3200 have if the value of log 3 = 0.4771?
A. 95
B. 94
C. 96
D. None of these
Ans : C
233. A and B enter in to a partnership and A invests Rs. 10,000 in the partnership. At the end of 4 months he withdraws Rs.2000. At the end of another 5 months, he withdraws another Rs.3000. If B receives Rs.9600 as his share of the total profit of Rs.19,100 for the year, how much did B invest in the company?
A. 12,000
B. 96,000
C. 8000
D. 6000
Ans : C
234. The sum of the first and the 9th of an arithmetic progression is 24. What is the sum of the first nine terms of the progression?
A. 216
B. 108
C. 54
D. None of these
Ans : B
235. What is the equation of the line that is parallel to the line 3x + 7y = 10 and passes through the point (4, 8)
A. 7x - 3y = 46
B. 3x + 7y = 44
C. 9x + 21y - 184 = 0
D. 3x + 7y = 68
Ans : D
236. Ram and Shyam take a vacation at their grandparents' house. During the vacation, they do any activity together. They either played tennis in the evening or practiced Yoga in the morning, ensuring that they do not undertake both the activities on any single day. There were some days when they did nothing. Out of the days that they stayed at their grandparents' house, they involved in one of the two activities on 22 days. However, their grandmother while sending an end of vacation report to their parents stated that they did not do anything on 24 mornings and they did nothing on 12 evenings. How long was their vacation?
A. 36 days
B. 14 days
C. 29 days
D. Cannot be determined.
Ans : C
237. Ram buys Rs.100 shares at Rs.112. If he sells the shares after a year at Rs.132 after receiving a dividend of 8% just before selling, what is the net yield on his investment?
A. 17.85%
B. 6.06%
C. 32%
D. 25%
Ans : D
238. The sum of 20 terms of the series 12 + 22, 32 + 42, 52 + 62 is:
A. 210
B. 519
C. 190
D. None of these.
Ans : A
239. A tank is fitted with 8 pipes, some of them that fill the tank and others that are waste pipe meant to empty the tank. Each of the pipes that fill the tank can fill it in 8 hours, while each of those that empty the tank can empty it in 6 hours. If all the pipes are kept open when the tank is full, it will take exactly 6 hours for the tank to empty. How many of these are fill pipes?
A. 2
B. 4
C. 6
D. 5
Ans : B
240. If A and B work together, they will complete a job in 7.5 days. However, if A works alone and completes half the job and then B takes over and completes the remaining half alone, they will be able to complete the job in 20 days. How long will B alone take to do the job if A is more efficient than B?
A. 20 days
B. 40 days
C. 30 days
D. 24 days
Ans : C
Profit and Loss
241. A trader makes a profit equal to the selling price of 75 articles when he sold 100 of the articles. What % profit did he make in the transaction?
A. 33.33%
B. 75%
C. 300%
D. 150%
Ans : C
242. A merchant buys two articles for Rs.600. He sells one of them at a profit of 22% and the other at a loss of 8% and makes no profit or loss in the end. What is the selling price of the article that he sold at a loss?
A. Rs. 404.80
B. Rs. 440
C. Rs. 536.80
D. Rs. 160
Ans : A
243. A trader professes to sell his goods at a loss of 8% but weights 900 grams in place of a kg weight. Find his real loss or gain per cent.
A. 2% loss
B. 2.22% gain
C. 2% gain
D. None of these
Ans : B
244. Rajiv sold an article for Rs.56 which cost him Rs.x. If he had gained x% on his outlay, what was his cost?
A. Rs. 40
B. Rs. 45
C. Rs. 36
D. Rs. 28
Ans : A
245. A trader buys goods at a 19% Aount on the label price. If he wants to make a profit of 20% after allowing a Aount of 10%, by what % should his marked price be greater than the original label price?
A. +8%
B. -3.8%
C. +33.33%
D. None of these
Ans : A
246. If apples are bought at the rate of 30 for a rupee. How many apples must be sold for a rupee so as to gain 20%?
A. 28
B. 25
C. 20
D. 22
Ans : B
247. One year payment to the servant is Rs. 200 plus one shirt. The servant leaves after 9 months and recieves Rs. 120 and a shirt. Then find the price of the shirt.
A. Rs. 80
B. Rs. 100
C. Rs. 120
D. Cannot be determined
Ans : C
248. Two merchants sell, each an article for Rs.1000. If Merchant A computes his profit on cost price, while Merchant B computes his profit on selling price, they end up making profits of 25% respectively. By how much is the profit made by Merchant B greater than that of Merchant A?
A. Rs.66.67
B. Rs. 50
C. Rs.125
D. Rs.200
Ans : B
249. A merchant marks his goods in such a way that the profit on sale of 50 articles is equal to the selling price of 25 articles. What is his profit margin?
A. 25%
B. 50%
C. 100%
D. 66.67%
Ans : C
250. A merchant marks his goods up by 75% above his cost price. What is the maximum % Aount that he can offer so that he ends up selling at no profit or loss?
A. 75%
B. 46.67%
C. 300%
D. 42.85% Ans : D

Quantitative Section : Data Interpretation Sample Questions
Questions 1 - 5 refers to the following table:
PROFILE OF CONGRESS IN YEAR X
(total membership: 535)
House of
Representatives Senate
________________________________________ ________________________________________Party ________________________________________
292 Democratic 62
143 Republican 38
435 TOTAL 100
Sex
418 Male 100
17 Female 0
Age
27 Youngest 34
77 Oldest 80
48 Average
(arithmetic mean) 54
Religion
255 Protestant 69
107 Catholic 12
18 Jewish 5
4 Mormon 3
51 Other 11
House of
Representatives Senate
________________________________________ ________________________________________Profession ________________________________________
215 Lawyer 63
81 Business Executive
or Banker 15
45 Educator 6
14 Farmer or Rancher 6
22 Career Government 0
24 Official 4
2 Journalist or
Communications 0
1 Executive 1
0 Physician 2
6 Veterinarian
Geologist 0
25 Worker or Skilled
Tradesperson
Other 3
Ethnic Group
17 Black American 1
2 Asian American 3
4 Hispanic American 0
1. In the Senate, if 25 male members were replaced by 25 female members, the ratio of male members to female members would be
A. 4 to 1
B. 3 to 1
C. 3 to 2
D. 2 to 1
E. 1 to 1
Ans : B
2. Approximately what percent of the members of Congress are lawyers?
A. 63%
B. 58%
C. 56%
D. 52%
E. 49%
Ans : D
3. If 5 senators are Catholic Democrats, how many senators are neither Catholic nor Democratic?
A. 79
B. 74
C. 69
D. 31
E. 21
Ans : D
4. If all lawyers and all women in the House of Representatives vote for the passage of a bill, how many more votes will be needed for a majority?
A. 435
B. 220
C. 3
D. 0
E. It cannot be determined from the information given.
Ans : E
5. Which of the following can be inferred from the information given in the chart?
I. More than 80 percent of the men in Congress are members of the House of Representatives.
II. The percent of members who are categorized as farmers or ranchers is greater for the House of Representatives than for the Senate.
III. The median age in the Senate is 57.
D. I only
E. II only
F. III only
G. I and II
H. I and III
Ans : A
Questions 6 - 7 refers to the following table:
PERCENT CHANGE IN DOLLAR AMOUNT OF SALES
IN CERTAIN RETAIL STORES FROM 1977 TO 1979
Percent Change
Store From 1977
to 1978 From 1978
to 1979
P +10 -10
Q -20 +9
R +5 +12
S -7 -15
T +17 -8
________________________________________
6. In 1979, for which of the stores was the dollar amount of sales greater than that of any of the others shown?
A. P
B. Q
C. R
D. S
E. It cannot be determined from the information given.
Ans : E
7. In store T, the dollar amount of sales for 1978 was approximately what percent of the dollar amount of sales for 1979?
A. 86%
B. 92%
C. 109%
D. 117%
E. 122%
Ans : C
Questions 8 - 9 refers to the following Figure:

8. Of every dollar received by the federal government, how much (in cents) is from coporate sources?
A. 32
B. 70
C. 30
D. 35
E. 29
Answer : 1
9. what percentage of the federal revenue is derived from borrowings?
A. 0.2%
B. 0.02%
C. 2.7%
D. 1.2%
E. 2.5%
Answer : 3
Questions 10 - 11 refers to the following table:
DIRECTIONS: The following question are based on the bellow table, which shows per capita Mean Expenditure, Per capita Food expenditure, Number of Households and Per capita cereal consumption, in both quantity and value, for different expenditure classes of rural India. The sampled 41597 households are divided into 12 expenditure classes, starting from less than Rs.65 per month per capita and ending at more than Rs.385 per capita per month.

10. According to the results of this sample survey, what is the proportion of total expenditure on food to total expenditure for all the sampled households taken together?
A. 58%
B. 36.7%
C. 63.3%
D. 71%
E. Cannot be determined
Answer : 3
11. What is the difference, approximately, between the gross expenditure of the sampled households in the Rs.95-110 expenditure class and in the Rs.180-215 expenditure class?
A. 372000
B. 448000
C. 496000
D. 93.8
E. 52.3
Answer : A
Questions 12 - 13 refers to the following Graph:
GRAPH SHOWS EXPENDITURE ON ARMS BY DIFFERENT COUNTRIES (VALUE IN DOLLARS '000 MILLIONS)

12. The amount spent by country C in 1983 is what percentage more than the amount spent by Countries A and B together in 1977? (Find approximately)
A. 50%
B. 179%
C. 75%
D. 13%
E. 70%
Answer : C
13. Which of the following statements must be true?
i. Country A spends minimum amount of its budget on arms.
ii. Throughout, Country C has spent the maximum amount on arms during the years shown.
iii. An examination of the information for the last 3 years reveals that generally all 3 countries are reducing their expenditure on arms.
D. i only.
E. i and ii only
F. i and iii only
G. ii and iii only
H. None of the statements above.
Answer : E
Analytical Reasoning Sample Questions
Directions :All CAT sample analytical resoning questions are based on a passage or set of conditions. While answering a few of the questions, you would find it useful to draw a rough diagram. To answer any CAT sample analytical reasoning question choose the answer you think is most appropriate among the given options.
Questions 1- 3
Three men (Tom, Peter and Jack) and three women (Eliza, Anne and Karen) are spending a few months at a hillside. They are to stay in a row of nine cottages, each one living in his or her own cottage. There are no others staying in the same row of houses.
1. Anne, Tom and Jack do not want to stay in any cottage, which is at the end of the row.
2. Eliza and Anne are unwilling to stay besides any occupied cottage..
3. Karen is next to Peter and Jack.
4. Between Anne and Jack's cottage there is just one vacant house.
5. None of the girls occupy adjacent cottages.
6. The house occupied by Tom is next to an end cottage.
1. Which of the above statements can be said to have been derived from two other statements ?
A. Statement 1
B. Statement 2
C. Statement 3
D. Statement 5
E. Statement 6
Ans : D
2. How many of them occupy cottages next to a vacant cottage ?
A. 2
B. 3
C. 4
D. 5
E. 6
Ans : C
3. Which among these statement(s) are true ?
I. Anne is between Eliza and Jack.
II. At the most four persons can have occupied cottages on either side of them. .
III. Tom stays besides Peter.
D. I only
E. II only
F. I and III only
G. II and III only
H. I, II and III
Ans : C
Questions 4 - 7
An employee has been assigned the task of allotting offices to six of the staff members. The offices are numbered 1 - 6. The offices are arranged in a row and they are separated from each other by six foot high dividers. Hence voices, sounds and cigarette smoke flow easily from one office to another.
Miss Robert's needs to use the telephone quite often throughout the day. Mr. Mike and Mr. Brown need adjacent offices as they need to consult each other often while working. Miss. Hardy, is a senior employee and has to be allotted the office number 5, having the biggest window. .
Mr. Donald requires silence in the offices next to his. Mr. Tim, Mr. Mike and Mr. Donald are all smokers. Miss Hardy finds tobacco smoke allergic and consecutively the offices next to hers to be occupied by non-smokers.
Unless specifically stated all the employees maintain an atmosphere of silence during office hours.
4. The ideal candidate to occupy the office furthest from Mr. Brown would be
A. Miss Hardy
B. Mr. Mike
C. Mr. Tim
D. Mr. Donald
E. Mr. Robert
Ans : D
5. The three employees who are smokers should be seated in the offices.
A. 1, 2 and 4
B. 2, 3 and 6
C. 1, 2 and 3
D. 1, 2 and 3
E. 1, 2 and 6
Ans : D
6. The ideal office for Mr. Mike would be.
A. 2
B. 6
C. 1
D. 3
E. 4
Ans : D
7. In the event of what occurrence, within a period of one month since the assignment of the offices, would a request for a change in office be put forth by one or more employees ?
A. Mr. Donald quitting smoking.
B. The installation of a noisy teletype machine by Miss Hardy in her office.
C. Mr. Robert's needing silence in the office (s) next to her own. .
D. Mr. Brown suffering from laryngitis.
E. Mr. Tim taking over the duties formerly taken care of by Miss. Robert. .
Ans : E
Analytical Reasoning Sample Questions
Questions 8 - 10
In an experiment conducted at a laboratory, 160 white mice were injected with Serum D. 160 other white mice were injected with a harmless sugar solution .In two weeks time 39% of the white mice, who were injected with Serum D contracted the highly contagious and often fatal disease, jungle fever. Hence, it can be concluded that jungle fever is caused by some elements similar to the elements in Serum D.
8. The above discussion would be weakened most severely in case it is shown that
A. People contracting jungle fever are usually the victims of the bite of the South American Lesser Hooded Viper.
B. One among the 160 white mice had already contracted jungle fever prior to the laboratory experiment.
C. The natural habitats of white mice does not contain any of the elements found in Serum D.
D. The scientists administered the injections being ignorant of the contents of the solutions used.
E. The 160 white mice used in the laboratory experiment were kept isolated from each other.
Ans : B
9. The above argument would be highly empowered in case it were shown that:
A. Some of the elements in Serum D are extracted from the root of a certain poisonous jungle wildflower.
B. Within a period of two weeks about 40% of the white mice, who were injected with a harmless sugar solution also contracted jungle fever.
C. Almost all the white mice died within a period of two days after the first symptoms appeared.
D. Normally the rate of jungle fever among white mice is less than 0.01%.
E. Invariably the blood of the victims of jungle fever victims contains a high level of a certain toxic substance also found in serum D.
Ans : E
10. Distribution of leaflets and delivering speeches on government property should be outlawed. Radicals and fanatics have no right to use public property when peddling their unsavory views.
The argument above is based on the postulate
A. The general public has a special concern in the free exchange of different political views.
B. Radicals and fanatics prefer the use of public property while propagating their viewpoint.
C. Every person who hands out leaflets and delivers speeches is a radical or fanatic.
D. Legal constraints which are applicable to one group need not be equally applicable to all.
E. Any political activity, which hinders the proper functioning of the government should not be protected by the law.
Ans : C
Questions 11 - 12
Successfully adjusting to one's environment leads to happiness. War at a universal level war destroys the weaker people, who are the most unable to adjust to their environment. Thus, war at the universal level puts weaklings out of their misery and allows more space for their predators to enjoy life in a better manner. As those actions have to be performed, which maximize the level of happiness of the greatest number, war at a universal level should take place.
11. What response would the author of the above discussion come up with, in the case of the objection that the weaklings far exceed strong people?
I. He would respond with the statement that the person making the objection is a weakling.
II. He would respond by saying that weaklings will be miserable no matter what happens.
III. He would respond with the statement that the strong would be frustrated if the weaklings are destroyed.
D. I only
E. II only
F. III only
G. I and II only
H. II and III only
Ans : E
12. The author's discussion would be greatly if he agreed to which of the following?
. Technology could change the environment.
I. War at the universal level would be an integral part of the environment.
II. It is possible for the strong to survive without suppressing the weak.
C. I only
D. II only
E. III only
F. I and III only
G. I, II and III only
Ans : A
Questions 13 - 15
Come back with us to the real America leaving behind the turmoil of civilization. The real America is still inhabited by the eagle, the buffalo, the mountain lion and elk; it is still spacious, sprawling and majestic. Experience the freedom and serenity still to be found in
13. Choose the best option to complete the above statement:
A. the natural beauty of our land
B. the fascinating urban centers
C. the wild terrain of Africa
D. one's own subconscious
E. the great sprawling cities of the Southwest
Ans : A
14. The above paragraph is most likely to appear in which of the following?
A. A Hunter's Guide to The United States
B. Exploring the Great Outdoors
C. The Quiet Beauty of Alaska
D. How the Eagle Became Extinct
E. Returning to America
Ans : D
15. When I am elected, I will work towards effecting those changes for which I have been fighting all these years. We will work together to do away with the bureaucratic bogs which have existed ever since my opponent took office. Everyone of you knows what I stand for; I invite my opponent to ...
For completion of the above statement choose the best option:
A. hand in his resignation graciously
B. make his stance clear
C. stop lying to the public
D. get our city more federal aid
E. extend his support to me
Ans : B
16. We can never make our beliefs regarding the world certain. Even scientific theory of a most rigorous and well-confirmed nature is likely to change over a decade or even tomorrow. If we refuse to even try to understand, then it is like resigning from the human race. Undoubtedly life of an unexamined kind is worth living in other respects--as it is no mean thing to be a vegetable or an animal. It is also true that a man wishes to see this speculative domain beyond his next dinner.
From the above passage it is clear that the author believes that
A. men would not do well to speculate
B. progress in the scientific field is impossible
C. one should live life with the dictum 'what will be will be'
D. men should ignore their animal needs
E. men are different from animals as far as their reasoning abilities are concerned.
Ans : E
Questions 17-21
Two or more essences out of a stock of five essences-- L, M, N, O, and P are used in making all perfumes by a manufacturer. He has learned that for a blend of essences to be agreeable it should comply with all the rules listed below.
A perfume containing L, should also contain the essence N, and the quantity of N should be twice as that of L.
A perfume containing M, must also have O as one of its components and they should be in equal proportion.
A single perfume should never contain N as well as O.
O and P should not be used together.
A perfume containing the essence P should contain P in such a proportion that the total amount of P present should be greater than the total amount of the other essence or essences used.
17. Among the following which is an agreeable formula for a perfume?
A. One part L, one part P
B. Two parts M, two parts L
C. Three parts N, three parts L
D. Four parts O, four parts M
E. Five parts P, five parts M
Ans : D
18. Adding more amount of essence N will make which of the following perfumes agreeable?
A. One part L, one part N, five parts P
B. Two parts M, two parts N, two parts P
C. One part M, one part N, one part P
D. Two parts M, one part N, four parts P
E. Two parts N, one part O, three parts P
Ans : A
19. Among the following, the addition of which combination would make an unagreeable perfume containing two parts N and one part P agreeable?
(A) One part L (B) One part M (C) Two parts N (D) One part O (E) Two parts P
Ans : E
20. Among the following which combination cannot be used together in an agreeable perfume containing two or more essences?
A. L and M
B. L and N
C. L and P
D. M and O
E. P and N
Ans : A
21. Among the below mentioned formulas, which can be made agreeable by the eliminating some or all of one essence ?
A. One part L, one part M, one part N, four parts P
B. One part L, two parts N, one part O, four parts P
C. One part L, one part M, one part O, one part P
D. Two parts L, two parts N, one part O, two parts P
E. Two parts M, one part N, two parts O, three parts P
Ans : B
22. Everything that a person does, which is dictated by reason of ignorance is not voluntary. Involuntary actions are those which produce pain and repentance. Incase a man has done something in his ignorance and he does not feel vexed due to his action, he has not acted voluntarily as he was not aware of what he was doing, nor yet involuntarily since he is not pained.
After reading this passage we can arrive at the conclusion that:
A. A person is not a voluntary agent, if he acts by reason of ignorance and repents. .
B. If an action is done by reason of ignorance and is not voluntary , then it was repented.
C. A man is an involuntary agent, if he acts by reason of ignorance.
D. Some actions are either involuntary or not voluntary.
E. If a man is not a voluntary agent, then he acted by reason of ignorance and repents.
Ans : A
23. Everything that God knows necessarily is, because even what we ourselves know necessarily is; and, of course, our knowledge is not as certain as God's knowledge is. But no future contingent thing necessarily is..
Among the following statements, which naturally follows from the above:
1. There are no future contingent things.
2. It is not true that God has knowledge of only necessary things.
3. God has knowledge of no contingent future things.
4. It is not possible for us to know God.
5. God has knowledge of everything. .
Ans : C
Questions 24 - 25
Some lawyers are of the view that the observation of the intrinsic qualities of pornography in any composition depends on literary criticism and hence it is a matter of opinion. It is rather odd, though, that in a legal connection, serious critics themselves quite often behave as if they believed criticism to be a matter of opinion. Why be a critic - and teach in universities - in case criticism involves nothing but uttering capricious and arbitrary opinions ?
24. In the above argument the author is trying to establish that
A. whether a composition can be called pornographic or not is a matter of opinion. .
B. it is not a matter of opinion whether a work is pornographic.
C. observance of the qualities of pornography is not dependent on literary criticism.
D. critics seem hypocritical.
E. critics should not teach at universities.
Ans : D
25. The above discussion would be weakened if it is pointed out that:
A. literary critics are of the opinion that nothing is pornographic.
B. lawyers believe that the observance of the qualities of pornography is a matter of opinion, as literary critics are not in agreement in this regard.
C. literary critics are not legal authorities.
D. literary critics should not concern themselves with deciding what is pornographic.
E. literary critics in the teaching profession at the university level are init only for the money.
Ans : B
Questions 26 - 31
Nine individuals - Z, Y, X, W, V, U, T, S and R - are the only candidates, who can serve on three committees-- A, B and C, and each candidate should serve on exactly one of the committees.
Committee A should consist of exactly one member more than committee B.
It is possible that there are no members of committee C.
Among Z, Y and X none can serve on committee A.
Among W, V and U none can serve on committee G.
Among T, S and R none can serve on committee C.
26. In case T and Z are the individuals serving on committee B, how many of the nine individuals should serve on committee C?
A. 3
B. 4
C. 5
D. 6
E. 7
Ans : B
27. Of the nine individuals, the largest number that can serve together on committee C is
A. 9
B. 8
C. 7
D. 6
E. 5
Ans : D
28. In case R is the only individual serving on committee B, which among the following should serve on committee A?
A. W and S
B. V and U
C. V and T
D. U and S
E. T and S
Ans : E
29. In case any of the nine individuals serves on committee C, which among the following should be the candidate to serve on committee A?
A. Z
B. Y
C. W
D. T
E. S
Ans : C
30. In case T, S and X are the only individuals serving on committee B, the total membership of committee C should be:
A. Z and Y
B. Z and W
C. Y and V
D. Y and U
E. X and V
Ans : A
31. Among the following combinations which could constitute the membership of committee C?
A. Y and T
B. X and U
C. Y, X and W
D. W, V and U
E. Z, X, U and R
Ans : B
Questions 32 - 34
(M, N, O and P are all different individuals)
I. M is the daughter of N.
II. N is the son of O
III. O is the father of P.
32. Among the following statements, which is true ?
A. O is the uncle of M.
B. P and N are brothers
C. M is the daughter of P.
D. If B is the daughter of N, then M and B are sisters.
E. If C is the granddaughter of O, then C and M are sisters.
Ans : D
33. Which among the following statements is contradictory to the above premises?
A. P is the father of M.
B. O has three children.
C. M has one brother.
D. M is the granddaughter of O.
E. Another party C, could be the mother of M.
Ans : A
34. If B is the son of N and B has one brother, D, then
I. M is the sister of D.
II. D and N are brothers.
III. O is the grandfather of D.
D. I only
E. II only
F. III only
G. I and III only
H. I and II only
Ans : D
Questions 35 - 41
The only people to attend a conference were four ship captains and the first mates of three of those captains. The captains were L, M, N and O; the first mates were A, D and G. Each person in turn delivered a report to the assembly as follows:
Each of the first mates delivered their report exactly after his or her captain. The first captain to speak was M, and captain N spoke after him.
35. Among the following which is not an appropriate order of delivered reports?
A. M, A, N, G, O, L, D
B. M, D, N, G, L, O, A
C. M, N, A, L, D, O, G
D. M, N, A, O, D, L, G
E. M, N, G, D, O, L, A
Ans : E
36. In case L speaks after A, and A is the third of the first mates to speak, then among the following statements which would be untrue?
A. O spoke immediately after G.
B. The order of the first four speakers was M, G, N, D.
C. O's first mate was present.
D. A was the fourth speaker after M.
E. The captains spoke in the order M, N, O, L.
Ans : D
37. Among the following statements which statement must be true?
A. In case the second speaker was a captain, the seventh speaker was a first mate.
B. In case the second speaker was a first mate, the seventh speaker was a captain.
C. In case the third speaker was a first mate, the seventh speaker was a captain.
D. In case the third speaker was a captain, the seventh speaker was a first mate.
E. In case the seventh speaker was a first mate, the first and third speakers were captains.
Ans : A
38. In case A spoke immediately after L and immediately before O, and O was not the last speaker, L spoke
A. second
B. third
C. fourth
D. fifth
E. sixth
Ans : C
39. In case G is M's first mate, D could be the person who spoke immediately
A. prior to T
B. prior to L
C. prior to V
D. after T
E. after V
Ans : D
40. In case A is the third of the first mates to speak, and L is the captain whose first mate is not present, which among the following statements must be true?
A. A spoke sometime before L.
B. D spoke sometime before O.
C. L spoke sometime before O.
D. O spoke sometime before L.
E. O spoke sometime before N.
Ans : B
41. Among the following statements, which would make M, D, N, G, L, O, A the only possible sequence of speakers?
A. D is M's first mate; G is N's first mate; A is O's first mate.
B. D is M's first mate; G is N's first mate; A was the second to speak after L.
C. The order of the first four speakers was M, D, N, G.
D. The order of the last three speakers was L, O, A.
E. The order in which the captains spoke was M, N, L, O.
Ans : B
Questions 42 - 45
1. The microbe A causes the contagious disease A.
2. The first symptoms appear after a period of two days since the microbe A enters the body.
3. The microbe A is found in some flies and bees.
4. A fly bit Jack on Monday, February 6.
5. Nick worked with Jack the next day, Tuesday, February 7.
There were no other possibilities of exposure to Disease A.
42. In case Jack showed symptoms of Disease A, which of the following statements would be true?
I. Jack contracted the Disease A from Nick.
II. Jack first noticed symptoms of Disease A on February 8.
III. The fly that Jack was bitten by was not a carrier of the microbe A.
D. I only
E. II only
F. III only
G. I and II only
H. I and III only
Ans : B
43. In case Nick displayed symptoms of the disease A, which among the following would be true?
. I only
A. II only
B. III only
C. II and III only
D. I, II and III
Ans : D
44. In case Jack displayed symptoms of Disease A, which would be true?
. Jack was also bitten by a fly on February 5.
I. Jack was bitten by a mosquito which carried the microbe A.
II. Nick contracted Disease A from Jack.
3. I only
4. II only
5. III only
6. I and II only
7. II and III only
Ans : D
45. In case Nick displayed the symptoms of Disease A which would be true?
. Nick was bitten by a bee on February 6.
I. Nick ate food which contained the microbe A.
II. Nick also worked with Jack on February 6.
3. I only
4. II only
5. III only
6. I and II only
7. I, II and III
Ans : A
Questions 46 - 47
The principal evil in today's society is selfishness. Everywhere we see people, who are concerned only with themselves. Personal advancement is the only motivating force in the world today. This does not mean that individuals are not willing to help one another; on the contrary, _________. But, these are only short-term occurrences which ultimately serve our long-term goal of personal gain.
46. To fill in the blank in the above passage, select one of the options from the below mentioned options:
1. we are always trying to undermine others’ endeavors.
2. my uncle Jeremy used to help me with my homework.
3. no one can be trusted, not even close friends.
4. our yearning for power prevents us from understanding our existential purpose.
5. there are many occasions when we graciously offer our assistance.
Ans : E
47. Which among the following options would most strongly contradict the author's attitude towards society?
1. The greatest strength of society is altruism.
2. The forces of good will ultimately triumph over evil.
3. Our short-term actions may ostensibly contradict our long-term goals.
4. We must all learn the art of selfishness.
5. Morality is the bedrock of a growing community.
Ans : A
48. It can be proved by an example that our words are devoid of meaning as they cannot be distinguished from their opposites. People think that they are aware of the difference between the meanings of 'bald' and 'having hair' Let us suppose that an average twenty-one year old has X strands of hair on his or her head. We say that such a person is not bald but has hair. But surely one hair less would make no difference, and a person with X - 1 hairs on his or her head would be said to have hair. Suppose we go on like this, decreasing one hair at a time, the result would be the same. Then what difference would there be between someone who has one hair and someone who has none? We call them both bald. Thus, we cannot make a distinction between the terms 'bald' and 'having hair.'
Among the following statements, which statement best counters the argument above?
A. The word 'bald' can be translated into other languages.
B. A word can have more than one meaning.
C. A word such as 'monkey' can be applied to several animals that differ in some respects.
D. Words can lack precision without being meaningless.
E. People cannot think clearly without using words.
Ans : D
49. Virus M helps in controlling the population of gnats; they manage to do this by killing the moth's larvae. Though the virus is always present in the larvae, it is only every six or seven years that the virus seriously decimates the numbers of larvae, greatly reducing the population of the gnats. Scientists are of the opinion that the gnats, usually latent, are activated only when the larvae experience biological stress.
In case the above mentioned scientists are correct, it can be inferred that the decimation of gnat populations by the virus M could be most probably activated by the following conditions?
A. A shift by the gnats from drought areas to a normal area occupied by them
B. The resultant stress from defoliation of trees attacked by the gnats for the second consecutive year.
C. Attacks on the larvae by all kinds by parasitic wasps and flies.
D. Starvation of the gnat larvae due to over population.
E. Spraying of gnat infested areas with laboratory - raised Virus M.
Ans : D
50. In a particular code, the digits from 0 to 9 inclusive are each represented by a different letter of the alphabet, the letter always representing the same digit. In case the following sum
B O P B
+ S K B
--------
C V B Q
holds true when it is expressed in digits, which of the following cannot be properly inferred:
A. B cannot be 0.
B. B must be less than 5.
C. Q must be even.
D. O + S must be greater than 8.
E. C must be greater than B by 1.
Ans : B
Questions 51 - 56
There are three on-off switches on a control panel A, B, and C. They have to be changed from an initial setting to a second setting according to the following conditions : In case only switch A is the switch on in the initial setting , then turn on switch B.
In case switches A and B are the only switches on in the initial setting, then turn on switch C. In case all the three switches are on initially setting, then turn off the switch C. For any other initial setting, turn on all switches that are off and turn off all switches, if any, that are on.
51. In case in the initial setting is the switches A and B are on and the switch C is off, then what could be the second setting?
A. A on, B on, C on.
B. A on, B off, C on.
C. A on, B off, C off.
D. A off, B on, C off.
E. A off, B off, C on.
Ans : A
52. In case switch B is the only switch on in the initial setting, what must be the second setting?
A. A on, B on, C on.
B. A on, B on, C off.
C. A on, B off, C on.
D. A off, B off, C on.
E. A off, B off, C off.
Ans : C
53. In case all the three switches are on in the second setting, which among the following could have been the initial setting ?
A. A on, B on, C on.
B. A on, B on, C off.
C. A on, B off, C on.
D. A on, B off, C off.
E. A off, B on, C off
Ans : B
54. In case switch A is off in the second setting, which among the following could have been the initial setting ?
A. A on, B on, C on.
B. A on, B on, C off.
C. A on, B off, C on.
D. A on, B off, C off.
E. A off, B on, C off.
Ans : C
55. In case only switch B is on in the second setting, which among the following could have been the initial setting ?
A. A on, B on, C on.
B. A on, B off, C on.
C. A off, B on, C off.
D. A off, B off, C on.
E. A off, B off, C off.
Ans : B
56. Which among the following initial settings leads to a second setting, where only one switch is off ?
A. A on, B on, C off.
B. A on, B off, C on.
C. A off, B on, C on.
D. A off, B on, C off.
E. A off, B off, C off.
Ans : D

Questions 57 - 59
A bus has exactly six stops on its route. The bus first stops at stop one and then at stops two, three, four, five, and six respectively. After the bus leaves stop six, the bus turns and returns to stop one and repeats the cycle. The stops are at six building that are, in alphabetical order L, M, N, O, P, and Q.
P is the third stop.
M is the sixth stop.
The stop O is the stop immediately before Q.
N is the stop immediately before L.
57. In case N is the fourth stop, which among the following must be the stop immediately before P ?
A. O
B. Q
C. N
D. L
E. M
Ans : B
58. In case L is the second stop, which among the following must be the stop immediately before M ?
A. N
B. L
C. P
D. O
E. Q
Ans : E
59. In case a passenger gets on the bus at O, rides past one of the stops, and gets off at P, which of the following must be true ?
A. O is stop one.
B. Q is stop three.
C. P is stop four.
D. N is stop five.
E. L is stop six.
Ans : A
60. A survey recently conducted revealed that marriage is fattening. The survey found that on an average, women gained 23 pounds and men gained 18 pounds during 13 years of marriage. The answer to which among the following questions would be the most appropriate in evaluating the reasoning presented in the survey ?
A. Why is the time period of the survey 13 years, rather than 12 or 14 ?
B. Did any of the men surveyed gain less than 18 pounds during the period they were married ?
C. How much weight is gained or lost in 13 years by a single people of comparable age to those studied in the survey ?
D. When the survey was conducted were the women as active as the men?
E. Will the gains seen in the study be retained over the lifetimes of the surveyed persons?
Ans : C
61. Classical works occupy a unique and peculiar position. They are imperishable as cultural landmarks, but the views expressed in some of the works are often superseded by newer views. But the classics survive just like ancient castles, as imposing features of the landscape, yet unsuited for habitation unless remodeled.
The principal point put forth in the above passage is that classical works.
A. Demand our respect and admiration even if their ideas are no longer current.
B. Prove to be obstacles in the path of new ideas as they are unduly respected.
C. Should be preserved and treasured as scholarly opinion keeps changing and they will come into fashion again.
D. Lose their distinctive features as time passes.
E. Are not given due consideration by the younger generation.
Ans : A
62. Experts in the field of forestry are now questioning the policy of attempting to extinguish all forest fires instead of allowing them to run their course and die out on their own. The study of forest fires indicates that in the long run, forest fires may in fact, be beneficial to the ecology as a whole.
Among the following, which statement would logically follow from one of the ’observations’ referred to in the passage above ?
A. Most of the forest fires are causes due to natural reasons like lightning rather than as a result of human carelessness.
B. Every year the number of people dying as they attempt to fight forest fires is greater than the number of people who would have died or suffered injury if the fires were allowed to run their own course.
C. Accumulation of highly flammable dead brushwood and timber, which could lead to massive uncontrollable fires is prevented by frequent small forest fires.
D. The animal and plant life destroyed by forest fires seldom represents endangered species.
E. Studies indicate that forest fires regularly occurred in the times prior to human occupation of forest areas.
Ans : C
63. There is a theory that the dinosaurs inhabiting the earth were destroyed when a huge heavenly body hit the earth. The impact of the heavenly body when it struck the earth threw a mass of pulverized debris into the atmosphere. This dust created a fog, which blocked the sunlight depleting plant life. This resulted in the perishing of the plant-eating dinosaurs; the meat-eating dinosaurs, who preyed on the plant-eaters inturn starved to death.
Which among the following statements, would best support the above mentioned theory, if true?
A. Enough dust has been sent into the atmosphere by modern volcanic explosions to change the color of sunsets around the world for several years.
B. Craters formed by heavenly bodies are plentiful in several areas of the South west where many dinosaur fossils have also been found.
C. Studies conducted recently on the bone structure of dinosaurs show that they were very agile and not ponderous awkward animals as popularly believed.
D. The sedimentary rock strata of the earth formed from atmospheric dust at the time the dinosaurs disappeared contains minerals and other trace elements in proportions characteristic of the heavenly bodies.
E. Many meat-and plant-eating species of mammals who coexisted with the dinosaurs continue to exist today, although with a slightly modified appearance.
Ans : D

Questions 64 - 70
Five educational films A, B, C, D, & E are to be shown to a group of students. The films are to be shown in a particular order, which conforms to the following conditions:
A must be shown earlier than C.
B must be shown earlier than D.
E should be the fifth film shown.
64. Which among the following is an acceptable order for showing the educational films ?
A. A, C, B, D, E
B. A, C, D, E, B
C. B, D, C, A, E
D. B, D, E, A, C
E. E, B, C, A, D
Ans : D
65. In case C is shown earlier than E, which among the following will hold true ?
A. A is the first film shown.
B. B is the second film shown.
C. C is the third film shown.
D. D is the fifth film shown.
E. E is the fourth film shown.
Ans : D
66. In case D is to be shown earlier than A, then for which among the following is there exactly one position from first through fifth in which it can be scheduled to be shown ?
A. A
B. B
C. C
D. D
E. E
Ans : C
67. Which among the following is a pair of films that CANNOT both be shown earlier than E ?
A. A and B
B. A and D
C. B and C
D. B and D
E. C and D
Ans : E
68. In case D and E are shown as far apart from each other as possible, which among the following would be true ?
A. A is shown earlier than B.
B. B is shown earlier than C.
C. C is shown earlier than E.
D. D is shown earlier than A.
E. E is shown earlier than B.
Ans : E
69. In case B, D and E are to be shown one after the other in the given order, the two positions from first to fifth in which A could possibly be shown are
A. first and second.
B. first and fourth.
C. second and third.
D. third and fifth.
E. fourth and fifth.
Ans : B
70. In case exactly one film is shown between A and C, and exactly one film is shown between B and D, which among the following will hold true ?
A. B is the film shown between A and C.
B. C is the film shown between B and D.
C. E is the film shown between A and C.
D. D is the last film shown.
E. E is the first film shown.
Ans : E

71. Authorities concerned with mass transport in metropolitan cities are struggling with deficits. Commuters complain about delays and breakdowns, cuts in service, and fares higher than they are accustomed to paying. For all these reasons and because the price of fuel is still not prohibitive, the number of commuters using public transportation has fallen, adding to the deficits.
Which among the following statements about the relationship between the number of commuters using public transportation and the price of fuel is best supported by the above passage?
A. With the rise in the price of fuel, there is a rise in the number of commuters using public transportation.
B. Even if the price of fuel rises, the number of commuters using public transportation will continue to decline.
C. The number of commuters using public transportation will rise, if the price of fuel rises to a prohibitive level.
D. Most of the commuters using public transportation do not use fuel, therefore fluctuations in the price of fuel is unlikely to affect the number of commuters using public transportation.
E. The price of fuel is always low enough to make private transportation cheaper than public transportation; therefore fluctuations in the price of fuel is unlikely to affect the number of commuters using public transportation.
Ans : C
72. Pharmaceutical firms are now engaged in the production of analogues of endorphins, peptides, which are thought to transmit messages among brain cells, which bring about pain relief. The pharmaceutical firms claim that the analogues, when injected into the bloodstream, will lead to effective and long-lasting pain relief by increasing the action of peptides already present in the brain.
The claims put forth by the pharmaceutical firms would be considerably weakened if it were true that
A. Compared to the other types of neurotransmitters present in the brain, endorphins remain active for a longer time period.
B. Certain peptides have been traced in parts besides the brain, like the alimentary canal and the skin.
C. It is easier and cheaper to produce analogues of peptides in the laboratory than the peptides themselves.
D. Analogues of peptides, which are found naturally in the body are often filtered from the blood before the blood circulates in the brain.
E. Endorphins interact chemically both with other naturally occurring peptides and with the brain's other neurotransmitters.
Ans : D
73. Quite often it happens that a particular crisis or opportunity induces people to find a practical use for things, which originally had no serious purpose. To exemplify this principle, consider dolls and mannequins, programmed to move and built for the delight of the wealthy in the eighteenth century; these were the forerunners of the modern computer. Similarly, it is almost certain that pets were the first domesticated animals. Domestication of animals seems to have arisen as an amusement long before it had any practical application.
Which among the following, provides another example supporting the above mentioned principle, if true?
A. America was discovered as a by-product of the search for ginger, cloves, pepper, and cinnamon.
B. Children often imitate adults in their games.
C. Till the commercial and scientific potential of the spyglass was recognized and its power of magnification was improved it remained a mere source of diversion.
D. Some culture horses were used only for pleasure and not for work, even though in those cultures people had to labor hard in the absence of laboring animals.
E. People who made moving dolls and mannequins in the eighteenth century were also clockmakers.
Ans : C
74. Ever since the communications-entertainment firms have taken over publishing concerns, the management's new methods have increased the financial profits of commercial publishing. This is done at the price of narrowing the range of available books to the public and by catering to the vulgar tastes of the new buyers of books. There business has boomed ; but in the process the losers are the majority of authors or aspiring writers, and all discriminating readers.
In case the above statement is true, which among the following will hold true?
A. Profitable business ventures are relatively new in the publishing world.
B. The readership which commercial publishing caters to today is different from the readership served in the past.
C. The profits resulting from the methods introduced by communications-entertainment management will encourage literary writers to work against all the odds.
D. The limited range of books available to the public is directed toward a more discriminating audience.
E. The public is not aware of the trend in the publishing industry, which tends to specialize in publishing blockbuster books.
Ans : B
75. It is popularly believed that teachers are more or less indifferent about the microcomputer technology. This assumption is false, or at least dated. A survey recently conducted indicated that 80 percent of the 7,000 surveyed teachers revealed a high level of interest in microcomputers.
Among the following statements which would most damage the above argument if proved to be true?
A. There was no attempt made in the survey to ascertain whether the surveyed teachers had any previous exposure to microcomputers.
B. Teachers interested in microcomputer technology were more likely to complete and return the questionnaires than others.
C. Irrespective of their subject area, their expertise and their teaching experience questionnaires were received by the teachers.
D. After the survey results were tabulated there have been many developments in the applications of microcomputer technology.
E. A company manufacturing and selling microcomputers conducted the survey.
Ans : B
Questions 76 - 79
Six scientists A, B, C, D, E, and F are to present a paper each at a one-day conference. Three of them will present their papers in the morning session before the lunch break whereas the other three will be presented in the afternoon session. The lectures have to be scheduled in such a way that they comply with the following restrictions:
B's should present his paper immediately before C's presentation; their presentations cannot be separated by the lunch break.
D must be either the first or the last scientist to present his paper.
76. In case C is to be the fifth scientist to present his paper, then B must be
A. first
B. second
C. third
D. fourth
E. sixth
Ans : D
77. B could be placed for any of the following places in the order of presenters EXCEPT
A. first
B. second
C. third
D. fourth
E. fifth
Ans : C
78. In case F is to present his paper immediately after D presents his paper, C could be scheduled for which of the following places in the order of presenters?
A. First
B. Second
C. Third
D. Fourth
E. Fifth
Ans : E
79. In case F and E are the fifth and sixth presenters respectively then which of the following must be true?
A. A is first in the order of presenters.
B. A is third in the order of presenters.
C. A is fourth in the order of presenters.
D. B is first in the order of presenters.
E. C is fourth in the order of presenters.
Ans : C
Questions 80 - 82
In a small inn, one or more of the chefs have to perform duty during dinnereveryday. The chefs are Nicholas, Antonio, and Jeremy.
None of them can be assigned to dinner duty two or more days in a row.
80. In case Antonio and Jeremy share the dinner duty thrice over a five-day period, which among the following would be true?
A. Nicholas is on dinner duty alone on the first of the five days.
B. Nicholas is on dinner duty alone on the second of the five days.
C. Nicholas is on dinner duty alone on the third of the five days.
D. Antonio and Jeremy share dinner duty on the second of the five days.
E. Antonio and Jeremy share dinner duty on the fourth of the five days.
Ans : C
81. In case Nicholas and Antonio share dinner duty on Monday of some week, and if Antonio and Jeremy share dinner duty on Thursday of the same week, which of the following would be true for that week?
A. Antonio is on dinner duty alone on Tuesday.
B. Jeremy is on dinner duty alone on Wednesday.
C. Nicholas and Jeremy share dinner duty on Wednesday.
D. Nicholas is on duty alone on Tuesday, and Antonio is on breakfast duty alone on Wednesday.
E. Jeremy is on breakfast duty alone on Tuesday, and Nicholas is on breakfast duty alone on Wednesday.
Ans : E
82. Which among the following could be true of some four-day period?
A. On three of the four days, pairs of chefs were on dinner duty.
B. On one of the four days, all three chefs were on dinner duty.
C. Antonio drew twice as many dinner assignments as did Jeremy.
D. Nicholas drew three times as many dinner assignments as did Antonio.
E. Both Nicholas and Antonio drew three times as many dinner assignments as did Jeremy.
Ans : C
83. Which among the following statements is the most probable taking into consideration the source and the degree of seriousness with which they are uttered?
A. Examiner: "The exam will be taken under the honor system: the professors have the honor, and the students have the system."
B. Banker: "The low percentage of Jews in this bank has nothing to do with discrimination. Jews don't seem to be interested in banking."
C. Book cover: "This book is sure to become the last word in its field."
D. Politician: "My opponent has an impeccable political and personal record."
E. Artist: "Einstein's theory of relativity falsifies life."
Ans : D
84. Tom: It is likely that Greece will be humiliated in the Cyprus affair. In case she fights Turkey, she will be defeated in battle; whereas if she doesn't fight, letting Turkey win, she will be humiliated, as she would then seem impotent.
Sarah: In that case Greece could always call the Soviet Union to her aid.
Sarah attacks Tom's argument by:
A. attacking Tom on a personal level.
B. attacking Tom's method of reasoning
C. siding with Greece
D. advocating passive resistance
E. citing an alternative which was overlooked by Tom.
Ans : E
85. In case Sue sits between Pete and Harry, then Harry sits between Sue and Mike. Harry won't be there unless Sue sits next to Mike. Hence, Sue will not sit between Pete and Harry.
Apart from the above mentioned statements, what additional premises are assumed by the author of this argument?
Mike sits next to Sue if no one sits between them.
If Sue sits between Pete and Harry, then Sue sits between Harry and Pete.
If Harry isn't there, then he doesn't sit next to Mike.
A. I and II only
B. I and III only
C. II and III only
D. I, II and III
E. None of the above
Ans : B
Questions 86 - 89
An increasing number of people prefer to retain their own individuality and their own identity and consequently this has lead to a decline in the marriage rate.
86. Which among the following assumptions are used in the above premises?
I. When a person is married, he or she loses his or her own identity and is no longer accountable to himself or herself.
II. Married persons do not find contentment as opposed to unmarried people.
III. There has been a steady increase in the divorce rate.
D. I only
E. II only
F. III only
G. I and II only
H. I, II, and III
Ans : A
87. Among the following statements, which would weaken the above argument?
. The stability resulting from marriage offsets the negative aspects of the dual responsibility of husband and wife.
A. Most people are not mature enough to be married.
B. Among most married couples the wife wants to have children.
C. There are a differing set of values honored by men and women.
D. It is advantageous to be single form a tax point of view.
Ans : A
88. Which among the following would strengthen the above argument?
. Very few people prefer to bring up a family.
A. Emotionally divorce is not an easy procedure.
B. 700 couples from 1000 surveyed couples complained that they were losing their identity.
C. Married people have to make a considerable effort to make the marriage last.
D. The financial complications arising from a divorce are becoming decreasingly complicated.
Ans : C
89. Emma: Financially the private university is ruined. In case it raises its tuition fees, the number of paying students will come down and if they refrain from raising the fees the university will go bankrupt.
Richard: The pay of the teachers and professors could be cut.
Richard responds to Emma’s argument by
. drawing attention to an earlier overlooked alternative.
A. attacking her method of reasoning.
B. resigning himself to the demise of the private university.
C. attacking Emma on a personal level.
D. taking the side of the university as against that of the students.
Ans : A
Questions 90 - 92
Mrs. Green wishes to renovate her cottage. She hires the services of a plumber, a carpenter, a painter, an electrician, and an interior decorator. The renovation is to completed in a period of one working week i.e. Monday to Friday. Every worker will be taking one complete day to do his job. Mrs. Green will allow just one person to work per day.
1. The painter can do his work only after the plumber and the carpenter have completed their jobs.
2. The interior decorator has to complete his job before that of the electrician.
3. The carpenter cannot work on Monday or Tuesday.
90. In case the painter works on Thursday, which among the following alternatives is possible?
A. The electrician works on Tuesday.
B. The electrician works on Friday.
C. The interior decorator does his work after the painter.
D. The plumber and the painter work on consecutive days.
E. Mrs. Green cannot fit all of the workers into schedule.
Ans : B
91. In case the painter works on Friday, which among the following statements must be untrue?
A. The carpenter may work on Wednesday.
B. The carpenter and the electrician may work on consecutive days.
C. In case the carpenter works on Thursday, the electrician has to work on the previous day i.e. Wednesday.
D. The plumber may work before the electrician does.
E. The electrician may work on Tuesday.
Ans : C
92. Which arrangement among the following is possible?
A. The electrician will work on Tuesday and the interior decorator on Friday.
B. The painter will work on Wednesday and the plumber on Thursday.
C. The carpenter will work on Tuesday and the painter on Friday.
D. The painter will work on Monday and the carpenter on Thursday.
E. The carpenter will work on Wednesday and the plumber on Thursday.
Ans : E

93. According to the laws pertaining to the use of recyclable containers, beverages are sold only in reusable containers. The people who object to such laws point out that the collection of reusable bottles requires gasoline, and washing them requires water; but, it takes less water to wash these containers than it does to make throwaway bottles. The expenditure saved due to the saved water exceeds the cost of the gasoline used to transport empty bottles.
The above passage directly answers which of the below mentioned objections to recyclable beverage containers?
A. It is likely that consumers will continue to discard even the recyclable containers.
B. Consumers will find it an inconvenience to return bottles.
C. The extra expense involved in recycling would raise the prices of beverages.
D. Recycling would be detrimental to the glass and plastic industries, which produce containers.
E. Recycling of containers only partially answers the problem of disposal of solid waste.
Ans : C
94. A recently conducted study reveals that the prospects for success in later schooling for 3-5 old children were improved by a particular educational plan. From this it follows that the introduction of similar education plans for all 3-5 year old children will lead to an improvement in their chances for success in later schooling.
Which among the following would weaken the above argument if true?
A. Parents of preschool children in the United States get attracted to fads in the educational field, without having a clear idea of the type of early educational plans which would benefit their children.
B. The cognitive abilities of children are constantly changing between ages three and five
C. The people who conducted the research included quite a few children who had earlier been exposed to another educational enrichment program.
D. Many parents are of the view that formal training at an early age takes up the time, which the children could better spend exploring their worlds independently.
E. To establish such educational programs at a national level would require extraordinary public expense.
Ans : C
Questions 95 - 97
Three girls Joan, Rita, and Kim and two boys Tim and Steve are the only dancers in a dance program, which consists of six numbers in this order: One a duet; two a duet; three a solo; four a duet; five a solo; and six a duet.
None of the dancers is in two consecutive numbers or in more than two numbers.
The first number in which Tim appears is the one that comes before the first number in which Kim appears.
The second number in which Tim appears is one that comes after the second number in which Kim appears.
95. Which among the following is a complete and accurate list of those numbers that could be the last one in which Kim performs?
A. Three
B. Four
C. Five
D. Three, Four
E. Four, Five
Ans : E
96. Rita must perform only in duets if
A. Kim is in number two
B. Kim is in number five
C. Tim is in number one
D. Tim is in number two
E. Tim is in number six
Ans : D
97. In case Steve is in number five, number four must consist of
A. two women
B. two men
C. Tim and a woman
D. Rita and a man
E. Kim and a man
Ans : A
Logical Reasoning Sample Questions
Directions : Each CAT sample logical reasoning question in this part of the assessment starts with a reading passage containing the information to be used to choose between correct and incorrect logical conclusions. These conclusions are based on the information in the passage. After this reading passage, you are given a lead-in phrase that tells you to choose from among five different responses. These possible responses are generated by correctly or incorrectly applying logical thought to the information in the passage at the beginning of the question. They can be thought of as different ways of completing a sentence that begins with the lead-in phrase.
Each reading passage is based on actual Bureau of Labor Statistics documents but is not necessarily a completely accurate representation of BLS work. It is important that you accept every fact in the reading passage as true, when you evaluate the response choices offered. You should use only the information in the passage as the basis for accepting or rejecting any response choices. Be careful not to allow any "facts" that are not clearly stated in the reading passage, or any outside knowledge you may have of the "facts", to influence your thinking.
Following are some CAT sample logical reasoning questions.
1. Testifying before the Senate committee investigating charges that cigarette manufacturers had manipulated nicotine levels in cigarettes in order to addict consumers to their products, tobacco executives argued that cigarette smoking is not addictive. The primary reason they gave in support of this claim was that cigarette smoking was not regulated by the Federal Drug Administration.
For the tobacco executives' argument to be logically correct, which of the following must be assumed?
A. Substances that are not addictive are not regulated by ...........the Federal Drug Administration.
B. The tobacco executives lied when they claimed that ...........cigarette smoking was not addictive.
C. Some addictive substances are not regulated by the ...........Federal Drug Administration.
D. There is no scientific proof that cigarette smoking is ...........addictive.
E. Substances that are not regulated by the Federal Drug ...........Administration are not addictive.
Ans : E
2. People should be held accountable for their own behavior, and if holding people accountable for their own behavior entails capital punishment, then so be it. However, no person should be held accountable for behavior over which he or she had no control.
Which of the following is the most logical conclusion of the argument above?
A. People should not be held accountable for the ...........behavior of other people.
B. People have control over their own behavior.
C. People cannot control the behavior of other people.
D. Behavior that cannot be controlled should not be ...........punished.
E. People have control over behavior that is subject ...........to capital punishment.
Ans : B
3. There is clear evidence that the mandated use of safety seats by children under age four has resulted in fewer child fatalities over the past five years. Compared to the five-year period prior to the passage of laws requiring the use of safety seats, fatalities of children under age four have decreased by 30 percent.
Which one of the following, if true, most substantially strengthens the argument above?
A. The number of serious automobile accidents involving ...........children under age four has remained steady over the ...........past five years.
B. Automobile accidents involving children have decreased ...........sharply over the past five years.
C. The use of air bags in automobiles has increased by ...........30 percent over the past five years.
D. Most fatal automobile accidents involving children under ...........age four occur in the driveway of their home.
E. The number of teenage drivers has increased by 30 ...........percent over the past five years.
Ans : A
4. Lycopene, glutathione, and glutamine are powerful antioxidants that neutralize the free radicals that are produced in the body as a result of routine bodily processes. An excess of these free radicals in your system causes rapid aging because they accelerate the rate of cellular damage. Aging is simply the result of this damage. Thus, to slow down aging it is necessary to supplement your diet with these antioxidants on a daily basis.
Which of the following, if true, most seriously undermines the author's contention?
A. Most persons aren't concerned with the effects of aging ...........until it is too late to do anything.
B. Exercise associated with normal daily activities effectively ...........neutralizes and dissipates the free radicals that are ...........produced as a result of routine bodily processes.
C. The cost of antioxidants is exorbitantly high and well ...........beyond the budget of most consumers.
D. Only overweight people who do not exercise on a daily ...........basis are likely to have an excess of free radicals in their ...........systems.
E. Smoking cigarettes is one of the main causes of cellular ...........damage in humans.
Ans : B
5. Is it wrong for doctors to lie about their patients' illnesses? Aren't doctors just like any other people we hire to do a job for us? Surely, we would not tolerate not being told the truth about the condition of our automobile from the mechanic we hired to fix it, or the condition of our roof from the carpenter we employed to repair it. Just as these workers would be guilty of violating their good faith contracts with us if they were to do this, doctors who lie to their patients about their illnesses violate these contracts as well, and this is clearly wrong.
The conclusion of the argument is best expressed by which of the following?
A. Doctors who lie to their patients about their illnesses ...........violate their good faith contracts with their patients.
B. Doctors often lie to their patients about their illnesses.
C. Doctors are just hired workers like mechanics and ...........carpenters.
D. It is wrong for doctors to lie about their patients' ...........illnesses.
E. Doctors, like mechanics and carpenters, enter into good ...........faith contracts with us when we hire them.
Ans : D
6. As any economist knows, healthy people pose less of an economic burden to society than unhealthy people. Not surprisingly, then, every dollar our state government spends on prenatal care for undocumented immigrants will save taxpayers of this state three dollars.
Which of the following, if true, would best explain why the statistics cited above are not surprising?
A. The state's taxpayers pay for prenatal care of all ...........immigrants.
B. Babies born in this state to undocumented immigrant ...........parents are entitled to infant care benefits from ...........the state.
C. State benefits for prenatal care serve to promote ...........undocumented immigration.
D. Babies whose mothers did not receive prenatal care ...........are just as healthy as other babies.
E. Pregnant women who do not receive prenatal care are ...........more likely to experience health problems than ...........other pregnant women.
Ans : E
7. Beautiful beaches attract people, no doubt about it. Just look at this city's beautiful beaches, which are among the most overcrowded beaches in the state.
Which of the following exhibits a pattern of reasoning most similar to the one exhibited in the argument above?
A. Moose and bear usually appear at the same drinking ...........hole at the same time of day. Therefore, moose and ...........bear must grow thirsty at about the same time.
B. Children who are scolded severely tend to misbehave ...........more often than other children. Hence if a child is ...........not scolded severely that child is less likely to ...........misbehave.
C. This software program helps increase the work ...........efficiency of its users. As a result, these users ...........have more free time for other activities.
D. During warm weather my dog suffers from fleas ...........more so than during cooler weather. Therefore, ...........fleas must thrive in a warm environment.
E. Pesticides are known to cause anemia in some people. ...........However, most anemic people live in regions where ...........pesticides are not commonly used.
Ans : D
8. Our school district should not spend its money on the new Verbal Advantage reading program. After all, our students get all the reading practice they need by studying history and science.
The argument above depends on which the following assumptions?
A. The Verbal Advantage program would not help the ...........students learn history and science.
B. Other reading programs are just as effective but ...........less expensive than the Verbal Advantage program.
C. The Verbal Advantage program involves only reading ...........practice.
D. Teaching students history and science is more ...........important than teaching them reading skills.
E. The students can already read well enough to ...........study history and science
Ans : C
9. A study of native born residents in Newland found that two-thirds of the children developed considerable levels of nearsightedness after starting school, while their illiterate parents and grandparents, who had no opportunity for formal schooling, showed no signs of this disability.
If the above statements are true, which of the following conclusions is most strongly supported by them?
A. Only people who have the opportunity for formal ...........schooling develop nearsightedness.
B. People who are illiterate do not suffer from ...........nearsightedness.
C. The nearsightedness in the children is caused by the ...........visual stress required by reading and other class work.
D. Only literate people are nearsighted.
E. One-third of the children are illiterate.
Ans : C
10. Newspaper publishers earn their profits primarily from advertising revenue, and potential advertisers are more likely to advertise in newspapers with a wide circulation—a large number of subscribers and other readers—than with other newspapers. But the circulation of the newspaper that is currently the most profitable one in this city has steadily declined during the last two years, while the circulation of one of its competitors has steadily increased.
Any of the following, if true, would help explain the apparent discrepancy between the two statements above EXCEPT:
A. Advertisers generally switch from the most widely ...........circulated newspaper to another one only when the ...........other one becomes the most widely circulated ...........newspaper instead.
B. Advertising rates charged by the most profitable ...........newspaper in the city are significantly higher than ...........those charged by its competitors.
C. The most profitable newspaper in the city receives ...........revenue from its subscribers as well from advertisers.
D. The circulation of the most profitable newspaper ...........in the city is still greater than than of any of its ...........competitors.
E. The number of newspapers competing viably with the ...........most profitable newspaper in the city has increased ...........during the last two years.
Ans : E
11. Although most of the fastest growing jobs in today's economy will require a college degree, many of the new jobs being created-from home health aide to desktop publisher-require knowledge other than that gained from earning a degree. For workers in those jobs, good basic skills in reading, communication, and mathematics play an important role in getting a job and developing a career.
From the information given above it can be validly concluded that, in today's economy,
A. skills in reading, communication, and mathematics play an important role in developing a career as a desktop publisher
B. the majority of the new jobs being created require knowledge other than that gained from earning a college degree
C. a job as a home health aide will rely more on communication skills than on basic skills in reading and mathematics
D. if a job is one of the fastest growing jobs, it will require a college degree
E. desktop publisher jobs and home health aide jobs are not among the fastest growing jobs
Ans : A
12. According to the National Agricultural Aviation Society (NAAS), without the use of crop protection products to control insects, weeds, and diseases, crop yields per acre will drop by more than 50 percent. The first aerial application of insecticide occurred in 1921, and it was a huge success. By contrast, in today's economy all aircraft that are classified as aerial applicators do more than just apply insecticide; today, they also spread seed and apply fertilizer.
From the information given above it CANNOT be validly concluded that
A. in today's economy, if an aerial applicator is used, then it will be able to spread seed and to apply fertilizer
B. according to the NAAS, if crop yields per acre never drop by more than 50 percent, then crop protection products have been used to control insects, weeds, and diseases
C. in today's economy, any aircraft that cannot be used to apply fertilizer cannot be classified as an aerial applicator
D. in 1921, if an aircraft was used for the application of insecticide, then it was not also used to spread seed
E. according to the NAAS, if crop yields per acre drop by more than 50 percent, then crop protection products have not been used to control insects, weeds, and diseases.
Ans : E
13. No national productivity measures are available for underground industries that may exist but remain unreported. On the other hand, at least some industries that are run entirely by self-employed industrialists are included in national productivity measures.
From the information given above, it can be validly concluded that
A. there are at least some industries run entirely by self-employed industrialists that are underground industries
B. no industries that are run entirely by self-employed industrialists operate underground
C. there are at least some industries other than those run entirely by self-employed industrialists that are underground industries
D. there are at least some industries run entirely by self-employed industrialists that are not underground industries
E. there are at least some underground industries for which national productivity measures are available
Ans : D
14. Lou observes that if flight 409 is canceled, then the manager could not possibly arrive in time for the meeting. But the flight was not canceled. Therefore, Lou concludes, the manager will certainly be on time. Evelyn replies that even if Lou's premises are true, his argument is fallacious. And therefore, she adds, the manager will not arrive on time after all.
Which of the following is the strongest thing that we can properly say about this discussion?
A. Evelyn is mistaken in thinking Lou's argument to be fallacious, and so her own conclusion is unwarranted.
B. Evelyn is right about Lou's argument, but nevertheless her own conclusion is unwarranted.
C. Since Evelyn is right about Lou's argument, her own conclusion is well supported.
D. Since Evelyn is mistaken about Lou's argument, her own conclusion must be false.
E. Evelyn is right about Lou's argument, but nevertheless her own conclusion is false.
Ans : B
15. Sally has never received a violation from the Federal Aviation Administration during her 16-year flying career. Sally must be a great pilot.
Which of the following can be said about the reasoning above?
A. The definitions of the terms create ambiguity.
B. The argument uses circular reasoning.
C. The argument works by analogy.
D. The argument is built upon hidden assumptions.
E. This is an example of an argument that is directed against the source of the claim rather than the claim itself.
Ans : D
16. The Japanese economic model created strong domestic industries through subsidies from its Ministry of Trade and by closing off competitive foreign firms to its domestic market. This strategy promised to help economic growth by incubating domestic industries. New Japanese industries could count on a known local demand and would be protected from competition by tariffs and other barriers. The program could reduce the amount of imports and therefore improve the nation's balance of trade.
Which of the following, based on the passage above, is a weakness in this economic strategy?
A. A protectionist policy will create animosity among other nations.
B. Fast growth of small industries will create a class of millionaires and increase the inequality of income.
C. Subsidies and import constraints keep domestic prices high and impose a burden on consumers.
D. Quotas are more regressive than tariffs.
E. The demand for the products made by the incubated industries would not be known.
Ans : C
17. Historically, famines have generally been followed by periods of rising wages, because when a labor force is diminished, workers are more valuable in accordance with the law of supply and demand. The Irish potato famine of the 1840s is an exception; it resulted in the death or emigration of half of Ireland's population, but there was no significant rise in the average wages in Ireland in the following decade.
Which one of the following, if true, would LEAST contribute to an explanation of the exception to the generalization?
A. Improved medical care reduced the mortality rate among able-bodied adults in the decade following the famine to below prefamine levels.
B. Eviction policies of the landowners in Ireland were designed to force emigration of the elderly and infirm, who could not work, and to retain a high percentage of able-bodied workers.
C. Advances in technology increased the efficiency of industry and agriculture, and so allowed maintenance of economic output with less demand for labor.
D. The birth rate increased during the decade following the famine, and this compensated for much of the loss of population that was due to the famine.
E. England, which had political control of Ireland, legislated artificially low wages to provide English-owned industry and agriculture in Ireland with cheap labor.
Ans : D
18. Cars are safer than planes. Fifty percent of plane accidents result in death, while only one percent of car accidents result in death.
Which of the following, if true, would most seriously weaken the argument above?
A. Planes are inspected more often than cars.
B. The number of car accidents is several hundred thousand times higher than the number of plane accidents.
C. Pilots never fly under the influence of alcohol, while car drivers often do.
D. Plane accidents are usually the fault of air traffic controllers, not pilots.
E. Planes carry more passengers than cars do.
Ans : B
19. The body of anyone infected by virus X will, after a week, produce antibodies to fight the virus; the antibodies will increase in number for the next year or so. There is now a test that reliably indicates how many antibodies are present in a person's body. If positive, this test can be used during the first year of infection to estimate to within a month how long that person has had the virus.
Which one of the following conclusions is best supported by the statements above?
A. Antibodies increase in number only until they have defeated the virus.
B. Without the test for antibodies, there is no way of establishing whether a person has virus X.
C. Antibodies are produced only for viral infections that cannot be fought by any other body defenses.
D. If a person remains infected by virus X indefinitely, there is no limit to the number of antibodies that can be present in the person's body.
E. Anyone infected by virus X will for a time fail to exhibit infection if tested by the antibody test.
Ans : E
20. Ever since I arrived at the college last week, I've been shocked by the poor behavior of the students. The student population is completely lacking in proper social skills.
Which of the following, if true, would weaken the above conclusion?
A. Students who are away from their parents often exhibit rude behavior.
B. The college numbers over 50,000 students.
C. The narrator is a student and has interacted with many students.
D. Social skills should not be expected of college students.
E. The narrator was reluctant to stay at the college.
Ans : B
21. A study of a math program implemented in several pre-schools indicates that children who received the specialized Math Plus math education between the ages three and five had significantly higher math scores in 3rd and 4th grade than their classmates who did not receive this instruction. The proponents of the math argue that the introduction of this program for all children age three to five will significantly improve their chances for success in school.
Which of the following, if true, would most seriously weaken the above argument?
A. Most parents send their children to preschool for social development and do not have a clear idea about what types of education they want for their children.
B. Cognitive abilities of 3- to 5-year-old children are constantly changing.
C. The children in the pre-schools that were studied had previously been exposed to another math enrichment program.
D. Children are not really interested in enrichment programs in preschool.
E. The cost factor needs to be specified and established before a large scale program can be undertaken.
Ans : C
22. The symptoms of mental disorders are behavioral, cognitive, or emotional problems. Some patients with mental disorders can be effectively treated with psychotherapy, but it is now known that in some patients' mental disorders result from chemical imbalances affecting the brain. Thus, these patients can be effectively treated only with medication that will reduce or correct the imbalance.
The argument depends on assuming which one of the following?
A. Treatment by psychotherapy can produce no effective reduction in or correction of chemical imbalances that cause mental disorders.
B. Treatment with medication always shows faster results for patients with mental disorders than does treatment with psychotherapy
C. Most mental disorders are not the result of chemical imbalances affecting the brain.
D. Medication is always more effective in treating patients with mental disorders than is psychotherapy.
E. Treatment with psychotherapy has no effect on mental disorders other than a reduction of the symptoms.
Ans : A
23. Dear Editor: I feel obliged to comment on the unfair review you published last week written by Robert Duxbury. Your readers should know that Mr. Duxbury recently published his own book that covered the same topic as my book, which you asked him to review. It is regrettable that Mr. Duxbury should feel the need to belittle a competing work in the hope of elevating his own book.
The author of the letter above makes her point by employing which method of argument?
A. Attacking the motives of the author of the unfavorable review.
B. Attacking the book on the same topic written by the author of the review.
C. Contrasting her own book with that written by the author of the review.
D. Questioning the judgment of the author of the unfavorable review.
E. Stating that her book should not have been reviewed by the author of a competing work.
Ans : A
24. The government of Zunimagua has refused to schedule free elections, release political prisoners, or restore freedom of speech; therefore, no more financial aid from the United States should be provided to Zunimagua.
Which of the following is an assumption made in the argument above?
A. Withdrawal of U.S. aid from Zunimagua will force a change in the policies of its government.
B. The people of Zunimagua would be better off if their present despotic government were overthrown.
C. The government of Zunimagua is dependent on continued U.S. aid for its existence.
D. U.S. aid should be given only to countries willing to adopt policies in line with U.S. interests and goals.
E. U.S. aid should be withdrawn from any country that refuses to operate its government along democratic lines.
Ans : E
25. Many people argue that the death penalty deters murder. However, the notorious killer Ned Grandy deliberately moved to a state that imposes the death penalty just before embarking on a series of ferocious murders. Thus, it seems clear that the existence of the death penalty does not serve as a deterrent to murder.
The argument above may best be characterized as:
A. an appeal to emotion.
B. a flawed analogy.
C. a general conclusion based on a specific example.
D. circular reasoning.
E. an application of a general principle to a specific example.
Ans : C
26. Steve and JoAnne are both members of a certain club, though they are not speaking to each other and refuse to work with each other. Cecily, the club president, is appointing members to the fundraising committee, but she has resolved that she will not appoint anyone without his or her explicit consent. Steve tells Cecily, "I will not consent to appointment on that committee unless I know whether JoAnne is to be a member of it." And JoAnne says, "I will not consent to be a member of that committee unless I know whether Steve will be appointed to it."
If all three of these people stick by these resolutions, then:
A. Neither of them can be appointed to the committee.
B. The situation described in the scenario cannot arise, because it is inherently incoherent.
C. They must either both be appointed or both be left out.
D. The committee may finally have one of them, both of them, or neither of them as members.
E. Either one of them can be appointed, but not both.
Ans : E
27. Russia's aggressive fishing in the prime fishing grounds of the Northern Pacific has led to a sharp decline in the populations of many fish and a general increase in the retail price of fish. This same pattern has occurred with far too many of our scarce vital natural resources, resulting in high prices for many products. It is likely then, that fish prices will continue to rise in the near future.
In making the argument above, the author relies on all of the following assumptions EXCEPT:
A. The scarcity of fish is a determining factor in its price.
B. The decline in the number of fish available will result in higher prices for fish in stores.
C. There will not be any substantial decrease in other costs involved in the fishing process that could keep the price of fish from increasing.
D. Fish populations will not recover in the near future.
E. Fishing practices can substantially influence the demand for fish.
Ans : E
28. During the past year, Boz Corporation, a cigarette manufacturer, has engaged in a "corporate image" advertising campaign. One executive now urges that the advertising be extended for another year because profits have increased by 29% over the previous year. Another executive, however, is skeptical. She observes that the increases are typical for the industry over the past year, although none of their competitors have used corporate image advertising.
The most accurate way of summarizing the second executive's point would be:
A. She argues that the effect may not really be due to its supposed cause because there has not been a sufficient lapse of time between the cause and the effect.
B. She argues that the assignment of a cause for this effect is premature, because there is as yet no well-established theory of such interactions.
C. She argues that corporate image advertising is unprofitable, since it has evidently benefited competitors as much as the corporation that paid for it.
D. She knows that effective advertising requires a constant influx of new ideas and approaches, and she argues that one year of corporate image advertising is enough for awhile.
E. She argues that the effect may not be due to its alleged cause since the same effect is found elsewhere without that cause.
Ans : E
29. The senate candidate expressed outrage that few judges have any background in technology, yet they try to resolve cases involving high tech companies. He stated that not one federal judge has a degree or any experience in computer technology.
A promising response to this concern, arguing that things are not as bad as they might seem, could involve which of the following claims?
A. Most of the public policy questions in this area are really about the morality and the value of scientific and technological developments. They do not require much technical understanding beyond that of a layperson.
B. Computer scientists, by and large, have little interest in politics and public policy. It would be difficult to find scientists with the degree of commitment required for a serious contribution to the judicial system.
C. There is a lack of people who are qualified in both technical and legal areas of expertise.
D. There is very little opportunity for, and indeed little need for, technical expertise in the judicial branch. There is therefore almost no way for a technical specialist to rise through the ranks to a top-level position in government.
E. The rewards of a life as a judge, in terms of both money and prestige, are not high enough to attract top-flight technical experts to this area.
Ans : A
30. There has been a sharp increase in the subscription prices of many professional and scholarly journals in the past seven years. Many publishers ascribe the necessity for these increases to the easy availability of photocopying facilities, which enable people simply to copy the articles they want rather than buying the journal.
Which of the following, if it is true, would make this explanation more plausible?
A. The great majority of professional and scholarly journals have a massive backlog of papers awaiting publication.
B. Over the past five years there has been a substantial decline in the number of individual subscriptions to professional and scholarly journals, while library subscriptions have remained fairly stable.
C. In the five years immediately preceding the price surge, there was a substantial decline in the number of individual subscriptions to professional and scholarly journals, while library subscriptions remained fairly stable.
D. Many libraries have recently begun cutting back on subscriptions to professional and scholarly journals.
E. In almost every field, several new professional and scholarly journals have begun publication in the past few years.
Ans : C
31. Smoking in bed has long been the main cause of home fires. Despite a significant decline in cigarette smoking in the last two decades, there has been no comparable decline in the number of people killed in home fires.
Each one of the following statements, if true over the last two decades, helps to resolve the apparent discrepancy above EXCEPT:
A. Compared to other types of home fires, home fires caused by smoking in bed usually cause relatively little damage before they are extinguished.
B. Home fires caused by smoking in bed often break out after the home's occupants have fallen asleep.
C. Smokers who smoke in bed tend to be heavy smokers who are less likely to quit smoking than are smokers who do not smoke in bed.
D. An increasing number of people have been killed in home fires that started in the kitchen.
E. Population densities have increased, with the result that one home fire can cause more deaths than in previous decades.
Ans : B
32. Mrs. Mason is gifted with psychic powers that enable her to foretell future events. In the past, Mrs. Mason has predicted such actual events as the election of President Clinton, the stock market crash of 1987, and the St. Louis Cardinals' 1982 World Series victory. These are just a few of Mrs. Mason's accurate predictions.
The answer to which of the following questions would be most useful in evaluating the strength of the argument above?
A. What percentage of Mrs. Mason's predictions has come true?
B. Could the election of President Reagan have been predicted without the help of psychic powers?
C. What is the actual mechanism by which Mrs. Mason's psychic powers are supposed to operate?
D. How long before the events in question did Mrs. Mason make her accurate predictions?
E. Do most scientists accept the idea that the power to predict the future through psychic means really exists?
Ans : A
33. An ingredient in coffee, known as RTC, has been found to inactivate common cold viruses in experiments. In previous experiments, researchers found that inactivated common cold viruses can convert healthy cells into cancer cells. It can be concluded that the use of coffee can cause cancer.
Which one of the following, if true, most seriously weakens the argument?
A. Several teams of scientists performed the various experiments, and all of the teams had similar results.
B. The carcinogenic effect of RTC could be neutralized by the other ingredients found in coffee.
C. When RTC kills common cold viruses it weakens the immune system, and it might thus diminish the body's ability to fight other viruses, including viruses linked to cancers.
D. If chemists modify the structure of RTC, RTC can be safely incorporated into medications to prevent the common cold.
E. To lessen the undesirable side effects of chemotherapy, the use of coffee has been recommended for cancer patients who are free of the common cold virus.
Ans : B
34. Jack Bygrave is an executive at a major South African diamond company that produces 2% of the world's total annual diamond production. The CFO is anxious to maximize revenues and increase sales. Bygrave, however, believes that increased production would only drive down the world price of diamonds and lower revenues.
Which of the following represents the logical flaw in Bygrave's reasoning?
A. Jack connects the price of unrefined diamonds and the price of jewelry-quality diamonds.
B. He assumes that production goals are similar to financial goals.
C. He assumes that the supply produced by a single company can significantly alter the aggregate supply for the market.
D. He assumes that seasonal and long term supply are proportional.
E. He correlates long-term and short-term demand.
Ans : C
35. The crux of creativity resides in the ability to manufacture variations on a theme. If we look at the history of science, for instance, we see that every idea is built upon a thousand related ideas. Careful analysis leads us to understand that what we choose to call a new theme or a new discovery is itself always and without exception some sort of variation, on a deep level, of previous themes.
If all of the statements in the passage are true, each of the following must also be true EXCEPT:
A. A lack of ability to manufacture a variation on a previous theme connotes a lack of creativity
B. No scientific idea is entirely independent of all other ideas.
C. Careful analysis of a specific variation can reveal previous themes of which it is a variation.
D. All great scientific discoverers have been able to manufacture a variation on a theme.
E. Some new scientific discoveries do not represent, on a deep level, a variation on previous themes.
Ans : E
36. Studies of fatal automobile accidents reveal that, in the majority of cases in which one occupant of an automobile is killed while another survives, it is the passenger, not the driver, who is killed. It is ironic that the innocent passenger should suffer for the driver's carelessness, while the driver often suffers only minor injuries or none at all.
Which of the following is an assumption underlying the reasoning in the passage above?
A. In most fatal automobile accidents, the driver of a car in which an occupant is killed is at fault.
B. Drivers of automobiles are rarely killed in auto accidents.
C. Most deaths in fatal automobile accidents are suffered by occupants of cars rather than by pedestrians.
D. Auto safety experts should increase their efforts to provide protection for those in the passenger seats of automobiles.
E. Automobile passengers sometimes play a contributing role in causing auto accidents.
Ans : A
37. The editors of Business Today magazine conducted a poll of its readers regarding the proposed increase in the rate of income tax paid on profits from the sale of stocks. More than 60% of the readers opposed the proposed tax. The editors announced that the majority of Americans opposed any increase in the tax on profits from stock sales.
Which one of the following statements, if true, would most weaken the editor's conclusion?
A. Some readers of Business Today magazine are citizens of countries other than the United States.
B. Decisions concerning the income tax laws are made by the Congress rather than directly by the people.
C. Most of those who earn profits from stock sales are wealthy and can afford to pay higher taxes.
D. The viewpoints of the vast majority of the readers of Business Today magazine differ from the views of most Americans.
E. Not all readers of Business Today magazine responded to the editors' poll.
Ans : D
38. Several movie versions of Charles Dickens' Tale of Two Cities have been made. The original movie version made in 1939 is the best because it is closest in spirit to the original novel.
An underlying assumption of the argument above is that a movie based on a novel should:
A. reflect the director's original interpretation of the main themes of the novel.
B. accurately depict the time and place in which the novel is set.
C. feature actors and actresses who closely resemble the characters in the novel both in body and spirit.
D. faithfully render the details of the plot from the narrator's point of view.
E. capture the true meaning and intention of the novel.
Ans : E
General Awareness Sample Questions
Following are some CAT sample general awareness questions.
1. Which company's famously advertised vision statement is 'The Network is the Computer'?
A. Cisco Systems
B. Lucent Technologies
C. Sun Microsystems
D. Nortel Networks
Answer : C
2. The software company I-flex Solutions was originally a division of which famous financial services company?
A. Citicorp
B. ICICI
C. HSBC
D. ABN Amro Bank
Answer : A
3. Which former advertising personality has recently been named the Undersecretary of State for 'public diplomacy and public affairs' of the US in an exercise to rebrand the US following the September 11 terrorist strikes?
A. Charlotte Beers
B. Jay Chiat
C. Martin Sorrell
D. George Lois
Answer : A
4. Which premium international range of luggage was recently launched in India by BlowPlast?
A. Samsonite
B. Strolley
C. American Tourister
D. Delsey
Answer : D
5. This year saw the launch of Yahoo!, Amazon.com and the famous launch of the Orange mobile phone service in the UK. Which year was this?
A. 1992
B. 1994
C. 1995
D. 1993
Answer : B
6. Henry Ford revolutionised the car market with the first mass- produced car, the Ford Model T. In which year was it launched?
A. 1924
B. 1912
C. 1908
D. 1897
Answer : C
7. Which company owns the beer brands Haywards 2000, Hi-Five and Lal Toofan?
A. United Breweries
B. Millennium Alcobev
C. Shaw Wallace
D. Mohan Meakins
Answer : C
8. Which company owns the footwear brand - Stryde?
A. Woodland
B. Bata
C. Tata International
D. Hindustan Lever
Answer : C
9. It was the brand that made David Ogilvy famous. Which shirt brand's ads had the famous man with an eye patch in the 1950s, which catapulted David Ogilvy to fame?
A. Van Heusen
B. Arrow Shirts
C. Hathaway
D. Dockers
Answer : C
10. Which TV channel has been in the news for its coverage of the Bin Laden crisis, especially for having carried all broadcasts of Osama Bin Laden to audiences in West Asia?
A. Star Asia
B. BBC Asia
C. Al Jazeera
D. Khaleed Times
Answer : C
11. In the case of a test tube baby
A. Fertilization takes place inside the test tube
B. Development of the baby takes place inside the test tube
C. Fertilization takes place outside the mother's body
D. Unfertilized egg develops inside the test tube
Answer : C
12. Who elects the President of India?
A. Lok Sabha
B. Rajya Sabha
C. People of India
D. Parliament and State Assemblies
Answer : D
13. The Fundamental Duties were incorporated in the Constitution of India by the
A. First Amendment
B. Tenth Amendment
C. Thirty-second Amendment
D. Forty-second Amendment
Answer : D
14. The speed of light will be minimum while passing through
A. glass
B. vacuum
C. air
D. water
Answer : A
15. The time taken by the Sun to revolve around the center of our galaxy is
A. 50 mn years
B. 100 mn years
C. 250 mn years
D. 365 mn years
Answer : C
16. The oldest monarchy in the world is that of
A. Nepal
B. U K
C. Spain
D. Japan
Answer : D
17. The Vice-President is elected by an electoral college consisting of
A. only elected members of both Housed of Parliament
B. only Rajya Sabha members
C. all members of Parliament and State Legislative Assemblies
D. all members of Lok Sabha and Rajya Sabha
Answer : D
18. In which one of the following states of India is it legal for a Hindu male and illegal for a Muslim male to have more than one living wife?
A. Nagaland
B. Goa
C. Himachal Pradesh
D. Arunachal Pradesh
Answer : B
19. Where did Buddha die?
A. Lumbini
B. Kusinagara
C. Pavapuri
D. Magadha
Answer : B
20. What was the main difference between the Indus Valley Civilization and Vedic Civilization?
A. Indus Valley Civilization was urban, while the Vedic Civilization was rural.
B. "Pipal" tree was worshiped in Indus Valley Civilization, while "Burgad" tree was worshiped in Vedic Civilization.
C. The main emphasis in Indus Valley Civilization was on trade while in the Vedic Age was on religion.
D. Indus Valley Civilization believed in non-violence while Vedic Civilization had no hard and fast rules about violence.
Answer : A
21. All of the following are constituents of RNA molecule except
A. Adenine
B. Guanine
C. Thymine
D. Uracil
Answer : C
22. The father of modern biology is
A. Aristotle
B. Darwin
C. Mendel
D. Vesalius
Answer : A
23. Which of the following seeds will normalize blood sugar level?
A. Coriander
B. Mustard
C. Cumin
D. Fenugreek
Answer : D
24. The point at which solid, liquid and gaseous forms of a substance co-exist is called
A. sublimation point
B. distillation point
C. triple point
D. melting point
Answer : C
25. Which among the following substances is used as a lubricant?
A. Quartz
B. Silica
C. Graphite
D. Nickel
Answer : C
26. The weight of an object will be minimum when it is placed at
A. The North Pole
B. The South Pole
C. The Equator
D. The center of the Earth
Answer : D
27. Earth quake waves travel fastest in
A. Soil
B. Molten rock
C. Water
D. Flexible rock
Answer : C
28. The angle between the geographical meridian and magnetic meridian is called
A. Angle of dip
B. Angle of declination
C. Angle of inclination
D. None of the above
Answer : B
29. What is the currency of Mexico?
A. Lira
B. Peso
C. Mexican dollars
D. Krones
Answer : B
30. The Essar group of companies has been promoted by
A. Ruias
B. Ambanis
C. Goenkas
D. Kanorias
Answer : A
31. One of the following liquor brands is not owned by United Breweries
A. Kalyani Black Label
B. Blue Riband
C. McDowells
D. Bagpiper
Answer : D
32. Amtrex air conditioners has a technical collaboration with
A. Sanyo
B. Mitsubishi
C. Hitachi
D. Carrier
Answer : C
33. Marlboro Cigarette is owned by
A. ITC
B. Godfrey-Philips
C. Philip Morris
D. British American Tobacco
Answer : C
34. Who is the CEO of Microsoft?
A. Bill Gates
B. Paul Allen
C. Larry Ellison
D. Steve Ballnes
Answer : D
35. One of the following is not an Insurance Company
A. ICICI Prudential
B. HDFC Natwest
C. OM Kotak Mahindra
D. Birla Sun Life
Answer : B
36. What is the currency of Portugal?
A. Escudo
B. Guilder
C. Mark
D. Schilling
Answer : A
37. One of the following personalities is not associated with the Indian auto sector
A. Suresh Krishna
B. Venu Srinivasan
C. Anand Mahindra
D. Venugopal Dhoot
Answer : D
38. One of the following companies is not in the area of air-conditioning & refrigeration
A. BPL
B. Thermax
C. Lloyd
D. Carrier Aircon
Answer : C
39. How many countries are a part of the European union?
A. 18
B. 15
C. 21
D. 12
Answer : B
40. Who is the attorney general of United States of America?
A. Ashcroft
B. Powell
C. Rumsfeld
D. Cheney
Answer : A
41. When was the generic domain name (Top Level Domain - TLD) .com introduced?
A. 1989
B. 1985
C. 1994
D. 1991
Answer : B
42. Who is the prime minister of Israel?
A. Benajamin Netanyahu
B. Ariel Sharon
C. Shaul Mofaz
D. Goldamyer
Answer : B
43. What was the significant about the purchase of a kilo of lychees on the French island of Reunion, located in the Indian Ocean?
A. They were the first lot of lychees exported from India.
B. It was the first official purchase using the new currency Euro.
C. It was done by to commemorate the release of the new France with Princess Diana's image on it.
D. None of these
Answer : B
44. Which brand had the highest number of Web searches in 2001?
A. Google
B. Marlboro
C. Intel
D. Play Station
Answer : D
45. What is common to all of the following names - Eduardo Camano, Adolfo Rodriguez Saa, Ramon Puerta and Fernando de la Rua?
A. They were all former Presidents of Argentina.
B. They are the dreaded drug lords of South America
C. They own together 70% of the world's Silver mines.
D. None of these
Answer : A
46. The video games X box is a product of
A. Sega
B. Sony
C. Intel
D. Microsoft
Answer : D
47. It was acknowledged as the second-most dangerous computer virus in history, after the Love Bug virus. Name this virus, which struck in 2001
A. Melissa
B. Code Red
C. C-Brain
D. Major Domo
Answer : B
48. What does the letters XP stand for in the product Microsoft XP?
A. Extended product
B. Extra Pampering
C. Experience
D. Entry level product
Answer : C
49. Which business and media tycoon won the elections to become Head of State in Italy amidst widespread clouds of scandal?
A. Guillani Giovanni
B. Joe Pacci
C. Antonio Machiaveli
D. Silvio Berusconi
Answer : D
50. Which city is hosting the 14th Asian Games in 2004?
A. Manila
B. Busan
C. Beijing
D. Bangkok
Answer : B
51. Under what name is MTNL marketing its GSM based mobile telephony?
A. Swarna
B. Dolphin
C. Speed
D. Vayu
Answer : B
52. What is the name of the branded petrol with cleansing additives that is being marketed by HPCL?
A. Power
B. Premium
C. Speed
D. Clean X
Answer : A
53. Under what name is MTNL marketing its CDMA based WiLL service in Mumbai and Delhi?
A. Dolphin
B. Swarna
C. Garuda
D. Seema
Answer : C
54. To which business group did the Ambanis of Reliance Group sell their holding in Larsen and Toubro?
A. RP Goenkas
B. A V Birla
C. Kanorias
D. Mittals
Answer : B
55. Name the brand that was launched and promoted by a famous father-son duo.
A. ICICI Credit cards
B. Parket Beta
C. Maruti Versa
D. All of these.
Answer : C
56. What is the sub brand of the new car that Toyota is launching in India?
A. Accord
B. Jupiter
C. C Planet
D. Camry
Answer : D
57. With which brand would you identify the famous advertising slogan 'Think Different'?
A. Apple
B. IBM
C. Wipro
D. None of these
Answer : A
58. Which country is the company Nestle head quartered?
A. Sweden
B. Switzerland
C. The Netherlands
D. Luxemburg
Answer : B
59. What is the brand name of the range of shoes and apparel for children under the age of five, that Reebok plans to introduce shortly?
A. Weebok
B. Kids
C. Kidsport
D. Tyke
Answer : A
60. Which multinational packaged food company has an equity stake in Britannia?
A. Danone
B. Heinz
C. Frito Lays
D. Schweppes
Answer : A
61. Who is the chairperson of Bharti Group?
A. Raj Mittal
B. Alok Mittal
C. Sunil Mittal
D. Ramen Mittal
Answer : C
62. Which billionaire businessman has been elected Mayor of New York, replacing the popular Rudy Giuliani?
A. William Clay Ford
B. Larry Ellison
C. James Walton
D. Michael Bloomberg
Answer : D
63. Which insurance company, launched recently in India, will be using Snoopy from the cartoon series Peanuts in its advertising?
A. Aviva
B. HDFC Standard
C. MetLife
D. Sun Life
Answer : C
64. Which is the home country of the famous design firm Ikea?
A. Finland
B. Sweden
C. Switzerland
D. Denmark
Answer : B
65. 'One world. One family. One festival' - is the slogan used in advertising
A. Chinese New year
B. Hong Kong's annual shopping festival
C. Dubai's shopping festival
D. None of these
Answer : C
66. Which companies chips power more than 60% of the world's cell phones?
A. Nokia
B. Intel
C. Motorola
D. Texas Instruments
Answer : D
67. Who is the famous author of the book What they don't teach you at Harvard Business School?
A. Stephen R Covey
B. Mack McCormak
C. John Love
D. None of these
Answer : B
68. Who is the RBI governor?
A. Venkatraman
B. Brijesh Mishra
C. Ranjith Sau
D. Bimal Jalan
Answer : D
69. Who is the Vice President of United States?
A. Al Gore
B. Rumsfeld
C. Powell
D. Cheney
Answer : D
70. Which is the leading international audit firm that is facing serious charges on account of lapse in accounting practices?
A. Ferguson
B. PWH
C. Arthur Andersen
D. KPMG
Answer : C
71. Which family owns the Hero group of companies?
A. Munjals
B. Mansingh
C. Firodias
D. Hindujas
Answer : A
72. Who led the "Ocean to Sky" expedition that travelled along the Ganga (Ganges) river to its source?
A. Sir Edmund Hillary
B. Tenzing Norgay
C. Chris Bonington
D. Doug Scott
Answer : A
73. Hero Global is a venture promoted by the Munjal group
A. to market Hero Honda two wheelers in the overseas market.
B. to sell Hero bicycles in the overseas market.
C. to build a automobile design lab of international repute.
D. to set up company owned dealer network in India.
Answer : C
74. J. D. Power is a name associated with
A. an independent power producing company in New Delhi.
B. a power plant equipment manufacturing company.
C. a automotive battery manufacturing company.
D. an automobile customer survey organization.
Answer : D
75. What does ICANN stand for?
A. Internet Corporation for Assigned Names and Numbers
B. International Center for Agro Nutrients and Nourishment
C. Indian Council for Algorithm, Numbers and Notions.
D. International council for Approved Names and Numbers.
Answer : A
76. "Connecting People," is the by line of which company
A. Philips
B. Motorola
C. Ericsson
D. Nokia
Answer : D
77. Which founding father pioneered the idea of a national bank?
A. Hamilton
B. Jefferson
C. Washington
D. Adams
Answer : A
78. Who was Karl Marx's main collaborator on his famous works?
A. V I Lenin
B. Max Weber
C. Fredrick Engles
D. Joseph Stalin
Answer : C
79. Which of the following foreign partner / collaborator pair is not Correct?
A. HTA; J.Walter Thomson
B. Chaitra; Leo Burnett
C. O and M; WPP Group
D. Mudra; DMB&B
Answer : D
80. Hewlett Packard, the famous Computer company is named after
A. Its promoters, Hewlett and Packard
B. The city in which they commenced their operation initially
C. The street in which their first office was located.
D. None of these
Answer : A
81. Who's 'curve' did President Reagan use as the basis for his 'Trickle Down' economic policy?
A. Keynes
B. Ricardo
C. Laffer
D. Smith
Answer : C
82. Which group of people have been involved in the terrorist activities and rampant killing spree after the death of King Birendra in Nepal?
A. The Gurkhas
B. The Maoists
C. The Khukris
D. The Awomi clan
Answer : B
83. This company is the first to develop a new handheld computer series featuring a combination of Intel components and Microsoft software?
A. HP
B. Sony
C. Compaq
D. Acer
Answer : A
84. Which of the following companies was forced to shutdown its digital music service because of music industry lawsuits for piracy?
A. Musiccity
B. Morpheus
C. Livewire
D. Napster
Answer : D
85. Who was sued recently by BIAX for infringement of computer patents?
A. HP
B. Compaq
C. Microsoft
D. Apple
Answer : D
86. Which country is the largest producer of natural rubber in the world?
A. Malaysia
B. Indonesia
C. Thailand
D. Burma
Answer : C
87. With which of the following companies has the Department of Posts and Telegraph of India tied up to provide a comprehensive range of financial services?
A. LIC of India
B. Corporation Bank
C. Master Card International
D. Visa International
Answer : C
88. Which cable network is to be the sole provider of sports information to MSN?
A. STAR Sports
B. ESPN
C. Sportstock.com
D. Channel 9
Answer : A
89. Who is the current Defense Secretary of the United States of America?
A. Donald Rumsfeld
B. Colin Powell
C. Mr. Ashcroft
D. Dick Cheney
Answer : B
90. Which are the countries that are slated to host the 2002 Soccer World Cup?
A. Japan and Thailand
B. Mexico and Brazil
C. Germany and Poland
D. Japan and S.Korea
Answer : D
91. The NewYork Mayor Rudolph Guiliani rejected a $10 mn donation offered by this billionaire prince.
A. Prince Awaleed Bin Talal of S.Arabia
B. Prince Ahmed Bin Tariq of S.Arabia
C. Prince Md. Shah of Iran
D. Prince Khaleel Mohammed of Persia
Answer : A
92. Which antibiotic can cure Anthrax, if detected at an early stage?
A. Amoxycillin
B. Erythromycin
C. Cephalospirin
D. Ciprofloxacin
Answer : D
93. The farmers of _______________ were involved in violent protests against falling rice prices and US-led globalization as symbolized by WTO?
A. France
B. Indonesia
C. Denmark
D. S.Korea
Answer : D
94. Which of the following chemicals has been recently allowed by the Government of India to be mixed with petrol?
A. Methanol
B. Ethanol
C. Kerosene
D. Butanol
Answer : B
95. Who is the present men's champion of US Open?
A. Pete Sampras
B. Lleyton Hewitt
C. Andre Agassi
D. Patrick Rafter
Answer : B
96. Which is the longest running stage play in the world?
A. The Cats
B. The Mouse Trap
C. Three Blind Mice
D. Romeo and Juliet
Answer : B
97. "Be the first to know" is the baseline of which company?
A. BBC
B. STAR News
C. CNBC
D. CNN
Answer : D
98. Which advertising agency took over the agency Saatchi and Saatchi last year to become one of the largest advertising firms in the world?
A. Euro RSCG
B. Publicis
C. Young and Rubicam
D. Havas
Answer : B
99. India's first private FM radio station Radio City opened in Bangalore recently. Which company is the promoter behind this venture?
A. The Times of India
B. Mid-Day
C. Star India
D. Zee
Answer : C
100. Which bathing soap advertises with the baseline, 'For total skincare since 1918' in India?
A. Camay
B. Mysore Sandal
C. Hamam
D. Lux
Answer : B
101. Which one of the following partnership in the domestic insurance sector is inCorrect?
A. HDFC - Standard Life
B. Birla - Sun Life
C. Tata - Allianz
D. Max - New York Life
Answer : C
102. As per the latest National Readership Survey 2001, which newspaper has become the most widely read daily in the country?
A. Daily Thanti (Tamil)
B. Malayala Manorama
C. Dainik Bhaskar (Hindi)
D. Dainik Jagran (Hindi)
Answer : C
103. "Everything from cancer to heart attacks. Covered for 70 paise a day," is the baseline of an advertisement by
A. Birla Sun Life
B. Cancer Institute WIA
C. ICICI Prudential Life Insurance
D. OM Kotak Mahindra Life Insurance
Answer : C
104. Which Indian woman was recently included in the Fortune magazine's global list of 50 most powerful women in business?
A. Lalita Gupte
B. Meena Venkatesh
C. Rama Bijapurkar
D. Tina Ambani
Answer : A
105. As per the recent Global Competitiveness Report from the World Economic Forum, which country replaced US as the most competitive economy in the world?
A. Canada
B. Sweden
C. Finland
D. Singapore
Answer : C
106. What is common to Bethlehem Steel, Polaroid and Swissair?
A. All companies owned by GE
B. All recently applied for bankruptcy protection
C. Their founders committed suicide
D. All of these companies were recently targets of successful hostile takeover
Answer : B
107. What famous brand name is derived from a combination of two words, which mean 'dead' in French and 'one' in German?
A. Dettol
B. Mortein
C. Martini
D. Ritter
Answer : B
108. Which is the longest running TV cartoon in history, first launched by Warner Brothers in 1969 exclusively for television, and is now a popular fare on the Cartoon Network?
A. Scooby Doo
B. Tom and Jerry
C. Popeye
D. Johnny Bravo
Answer : A
109. Which company has recently launched a range of biscuits in India following its takeover of Nutrine confectionery?
A. Hindustan Lever
B. Marico
C. Parke Davis
D. Sara Lee
Answer : D
110. In the latest Forbes List of wealthiest Americans, 3 out of the top 4 richest men in USA have made their wealth from one company. Which one ?
A. Standard Oil (Exxon)
B. Microsoft
C. De Beers
D. Berkshire Hathaway
Answer : B
111. Which is the third highest peak in India?
A. Kanchenjunga
B. Nanga Parbat
C. Gasher Brum
D. Dunagiri
Answer : B
112. Which is the largest airport in the world?
A. Singapore airport
B. International airport at Kaula Lumpur
C. Heathrow at London
D. King Khalid at Saudi Arabia
Answer : D
113. In which year did Alexander the great die?
A. 332 B.C
B. 328 B.C
C. 323 B.C
D. 356 B.C
Answer : C
114. In which year did Mahatma Gandhi launch The Dandi March?
A. 1924
B. 1930
C. 1941
D. 1942
Answer : B
115. Eugenics is the study of
A. the development of embryos
B. the merits of legalizing mercy killing
C. the production of better offspring by careful selection of parents
D. genetical composition of plant population in relation to their habitats
Answer : C
116. In which year did Thomas Alva Edison invent the electric lamp?
A. 1854
B. 1879
C. 1904
D. 1888
Answer : B
117. What is the measure of the length of a standard tennis court?
A. 54 ft
B. 28 ft
C. 84 ft
D. 78 ft
Answer : D
118. Which is the first Asian country to host the Commonwealth Games?
A. S. Korea
B. Japan
C. Malaysia
D. China
Answer : C
119. Who is the president of the World Bank?
A. Michel Camdessus
B. John Wolfensohn
C. Emeka Anyaoku
D. Jose Maria Ruda
Answer : B
120. Who was the first Prime Minister of Great Britain?
A. Richard Travithick
B. Robert Walpole
C. Sir George Cayley
D. Robert Peary
Answer : B
121. Which advertising agency is behind the setting-up, in 1991, of MICA, India's first college focussed on advertising and communications?
A. Ogilvy and Mather
B. Saatchi and Saatchi
C. Mudra
D. Hindustan Thomson Associates
Answer : C
122. What famous car brand, originally from the former Czechoslovakia, was taken over by Volkswagen in the 1990s?
A. Skoda
B. Saab
C. Piaggio
D. Volvo
Answer : A
123. What is the title of the original BBC TV series on which the Hindi TV series, Kamzor Kadi Kaun, based?
A. Who wants to be a millionaire?
B. Treasure mine
C. Mastermind
D. The weakest link
Answer : D
124. The loyalty program of which company is known as "First Citizen's Club"?
A. Shopper's Stop
B. ABN Amro Bank
C. Taj group of Hotels
D. Air India
Answer : A
125. The following is the baseline found in the ad of which product "The man with the smooth edge"
A. Four Square cigarettes
B. Gillette Mach 3 shaving system
C. Palmolive shaving cream
D. None of these
Answer : A
126. Which company has been ranked the 'World's Most Admired Company' in the rankings by Fortune magazine in 2002?
A. Microsoft
B. General Electric
C. IBM
D. Fedex
Answer : B
127. McDonald's, in an unrelated diversification, is planning to set up a chain of hotels. Name the new brand.
A. Pleasant stay
B. Golden Arches
C. McStay
D. Holidayz
Answer : B
128. Which company unsuccessfully launched a soap under the brand Protex in India?
A. SmithKline Beecham
B. Hindustan Lever
C. Reckitt and Colman
D. Colgate Palmolive
Answer : D
129. Calloway, is a leading international brand of
A. Condoms
B. Guns
C. Binoculars
D. Golf equipment
Answer : D
130. Which famous CEO was featured in the endorsement ad for Microsoft XP with the quote, "When I saw Windows XP in action, I was amazed. How did Microsoft get hold of my wishlist?"?
A. Ratan Tata
B. Azim Premji
C. N R Narayana Murthy
D. Arun Jain
Answer : C
131. Which automobile major has made a debut in the Formula One racing circuit in 2002 with its team led by the drivers Allan McNish and Mike Salo?
A. Honda
B. Jaguar
C. Saab
D. Toyota
Answer : D
132. Which famous company started as Asahi Optical in 1919 - before adopting its current name in 1957?
A. Yashica
B. Asahi Glasses
C. Pentax
D. Nikon
Answer : C
133. What brand name was given , in 1924, by the Mayor of Tokyo when he released the first watch produced by Shokosha Watch Company?
A. Seiko
B. Citizen
C. Tissot
D. None of these
Answer : B
134. The advertisement of which product carried the baseline 'Mom said, "Always boil the water". Moms are seldom wrong'
A. Ion Exchange
B. Zero B
C. Ford Ikon
D. KLM Airline
Answer : C
135. Who has authored the latest business best-seller *Big Brands, Big Trouble?
A. Al Ries
B. Laura Ries
C. Naomi
D. Jack Trout
Answer : D
136. Which company owns the famous cigarette brands Charminar and Charms?
A. Godfrey Philips
B. Philip Morris
C. ITC
D. VST Industries
Answer : D
137. What is the name of the instrument used to measure blood pressure?
A. Barometer
B. Hygrometer
C. Hydrometer
D. Sphygmomanometer
Answer : D
138. At what price is the initial public offering of Canara Bank being issued at?
A. Rs.15
B. Rs.35
C. Rs.60
D. None of these
Answer : B
139. Nicolas Leonard Sadi Carnot, the noted French physicist is associated with what branch of study?
A. Thermodynamics
B. Sub atomic particle theory
C. Superconductivity
D. None of these
Answer : A
140. Identify the odd man out
A. Standard and Poor
B. Moody's
C. Forrester
D. ICRA
Answer : D
141. Royal Stag brand of Whisky is owned by
A. United Breweries
B. Shaw Wallace and Co
C. Seagram
D. Mohan Meakin
Answer : C
142. Iodex brand of pain balm is owned by
A. Smithkline Beecham
B. Wyeth Laboratories
C. Glaxo
D. Dr. Reddy's Laboratories
Answer : A
143. Who is the author who created the character Dilbert?
A. David Scott
B. Scott Robert
C. Robert Williams
D. Scott Adams
Answer : D
144. Which of the year in which Md. Ghazni first invaded India?
A. 1220 A.D
B. 1001 A.D
C. 1348 A.D
D. 1196 A.D
Answer : B
145. Who invented Parachute?
A. Leonardo da Vinci
B. Scanderberg
C. Sigismund
D. Van Gogh
Answer : A
146. Which of the following countries fought the first of the opium wars?
A. Russia - China
B. Britain - Indonesia
C. China - Britain
D. Japan - China
Answer : C
147. Beech King and Cessna are popular models of:
A. Cars
B. Aircraft
C. Ships
D. Transport vans
Answer : B
148. Cash Reserve Ratio ( CRR) and Stautory Liquidity Ratio (SLR ) are terms most closely related to which of the following industries/markets:
A. Stock Exchange
B. Banking
C. Mutual Fund
D. Income Tax
Answer : B
149. Dr. Pepper is a famous brand of:
A. Iced tea
B. Peppermint
C. Soft Drinks
D. None of these
Answer : C
150. Mr. Shiv Nadar is the Chairman of which of the following groups of companies?
A. Wipro
B. HCL
C. Aptech
D. None of these
Answer : B
151. Lloyd's is a world leader in which of the following industries?
A. Garment
B. Insurance
C. Commodity Trading
D. Banking
Answer : B
152. Jeffrey Immelt took over as the CEO from a legend of which US corporate giant?
A. Oracle
B. Boeing
C. General Electric
D. Federal Express
Answer : C
153. The management strategy with the acronym CRM can be expanded as:
A. Concrete Relationship Management
B. Customer Relationship Management
C. Correct Relationship Management
D. Constructive Relationship Management
Answer : B
154. Sudha, Akshata and Rohan are the wife, daughter and son respectively of:
A. Nandan Nilekani
B. Swraj Paul
C. Narayana Murthy
D. Azim Premji
Answer : C
155. Which group owns Crompton Greaves?
A. Mittals
B. Ruias
C. Thapars
D. Ranas
Answer : C
156. Kenwood corporation is a(an) ____________ company
A. Japanese
B. American
C. British
D. German
Answer : A
157. The following brands can be associated with which of the following products?Montecristo, Ramon Allones, Romeo Y Julieta.
A. Cigarettes
B. Cigars
C. Condoms
D. Liquour
Answer : B
158. In 1974, this US president resigned to avoind impeachmennt. Who?
A. Jimmy Carter
B. John F. Kennedy
C. Richard Nixon
D. Ronald Reagan
Answer : C
159. Esperanto, invented by the Russian scholar Zamenhof, is an artificial:
A. Lung
B. Language
C. Microscope
D. Car
Answer : B
160. King Hussein was the king of:
A. Egypt
B. Poland
C. Iraq
D. Jordan
Answer : D
161. The expansion for the BIS, in the context of the banking industry is:
A. Bank for International Settlements
B. Bank for Industrial Settlements
C. Bank for Industrial Sectors
D. Bank for International Services
Answer : A
162. The expansion for BIFR, in the context of the Indian Industry is:
A. Board for Industrial and Financial Reconstruction
B. Bureau for Industrial and Financial Reconstruction
C. Board for Investment and Financial Reconstruction
D. Bureau for Investment and Financial Reconstruction
Answer : A
163. MSNBC is a media channel owned by:
A. The US government
B. Microsoft
C. Time Warner
D. None of these
Answer : B
164. Chaebols are the industrial conglomerates of which of the following countries?
A. China
B. Vietnam
C. South Korea
D. Japan
Answer : C
165. Cartier, Piaget, Omega and Rado are all brands of:
A. Cameras
B. Suits
C. Watches
D. Pens
Answer : C
166. Augusto Pinochet was the dictator of which of the following countries?
A. Argentina
B. Chile
C. Brazil
D. Egypt
Answer : B
167. Prithvi is a:
A. Surface-to-air missile
B. Surface-to-surface missile
C. Air-to-Surface missile
D. Air-to-air missile
Answer : B
168. The first non-congress Prime Minister of India was:
A. Lal Bahadur Shastri
B. Morarji Desai
C. Deve Gowda
D. V.P.Singh
Answer : B
169. What is the Russian word that means "restructuring"?
A. Glasnost
B. Perestroika
C. Mir
D. None of these
Answer : B
170. What commonly used product was invented by Whitcomb Judson in 1893 but had no demand till the late 1910s when orders came for troop uniforms?
A. Cigarette lighters
B. Zippers
C. Walkie Talkies
D. Canned Soya
Answer : B
171. Mars, the fourth planet from the sun, is named after the Roman god of:
A. Peace
B. War
C. Love
D. Marriages
Answer : B
172. During the reign of which king did Mahayanism formally come into existence?
A. Ashoka
B. Kanishka
C. Chandra Gupta
D. Ajatashatru
Answer : B
173. Who was the founder of Sunga Dynasty?
A. Pushyamitra
B. Agnimitra
C. Devabuthi
D. Kirtivarman
Answer : A
174. Who was the first Muslim ruler of Delhi?
A. Iltumish
B. Allauddin Khalji
C. Mahmud of Ghazni
D. Qutbuddin Aibak
Answer : D
175. Lord Dalhousie introduced the first telegraph line in 1853, which ran between
A. Mumbai and Calcutta
B. Calcutta and Agra
C. Mumbai and Thana
D. Mumbai and Chennai
Answer : B
176. What is the new economy term that is used to describe a new product or service that uses new technology, which can destroy competitors and or sometime even the entire industry?
A. Melissa
B. Virus
C. Killer App
D. Widget
Answer : C
177. When there is one buyer and many sellers in a market - what is this situation called?
A. Monopoly
B. Monopsony
C. Monotony
D. Oligopoly
Answer : B
178. Which credit card company has recently issued an affinity card targeted at workingwomen under the sub-brand Diva?
A. Standard Chartered
B. Citibank
C. HSBC
D. ICICI
Answer : A
179. Whats common to: Devil Dx, Target, Neon, Queen, Miss India?
A. Original names suggested for Femina magazine brand name
B. All names of successful knap sack brands
C. All bicycles models of Hero Cycles
D. All sub-brands of Bombay Dyeing's range of clothing.
Answer : C
180. By taking over which mutual fund company has Franklin Templeton Investments become the largest private sector mutual fund company in India?
A. Pioneer ITI
B. SBI Mutual Fund
C. Sundaram Newton
D. IDBI Principal
Answer : A
181. Which famous CEO, currently applauded for turning around the fortunes of a famous computer company, reportedly takes home a salary of $1 per year?
A. Steve Case
B. Steve Jobs
C. Larry Ellison
D. Jim Clark
Answer : 2
182. What was unique about the purchase of a Gulfstream V business jet by American Internet tycoon Mark Cuban in 1999 for $40 million?
A. It was the first sale of this model of Jet by Gulf Stream
B. It was the first sale of aircraft by payment made in terms of Euros
C. It was the first Euro payment to be made by an individual
D. It was the first sale of an aircraft through the Internet
Answer : D
183. Name the world's largest television network that is beamed to over 210 countries?
A. MSNBC
B. STAR
C. CNN
D. BBC
Answer : C
184. 'What makes ___ such an efficient air conditioner. Well, let's begin by staying silent'. Which air conditioner advertisement carries this tag line?
A. Amtrex
B. Daikin
C. Voltas
D. Carrier Aircon
Answer : B
185. Which TV channel has bagged the exclusive cable and satellite TV rights for live telecast in India of the 2003 Cricket World Cup?
A. Sony
B. STAR Sports
C. STAR ESPN
D. Channel Nine
Answer : A
186. The Indian Government has approved in the end of 2001 the cultivation of genetically modified cotton called 'BT Cotton'. Which multinational company is the major beneficiary of this decision?
A. Cargill
B. BASF
C. Monsanto
D. All of these
Answer : C
187. Which Indian business group holds the maximum stake in the mobile cellular service provider Spice?
A. Aditya Birla group
B. Bharti group
C. Essar group
D. BK Modi group
Answer : B
188. Name the Greek Goddess of Victory - better known to us as a famous brand name?
A. Puma
B. Mars
C. Nike
D. Casablanca
Answer : C
189. In the late 1860s, there was a shortage of ivory - commonly used for making billiard balls. The search for a substitute led to the growth of a modern industry. What was it ?
A. Fibre glass
B. Plastics
C. Vulcanized rubber
D. Glass wool
Answer : B
190. 'The world's local bank' - is the tag line of which bank
A. ABN Amro
B. Standard Chartered Bank
C. Hong Kong and Shangai Banking Corporation
D. Rabo Bank
Answer : C
191. Which country accounts for the highest foreign direct investments (inflows) into India?
A. US
B. Japan
C. Mauritius
D. Channel Islands
Answer : C
192. Loggerhead, Atlantic Ridley, Painted and Green are four different species of:
A. Dogs
B. Snakes
C. Turtles
D. Dinosaurs
Answer : C
193. Benito Mussolini was a dictator of which country?
A. Germany
B. Poland
C. Italy
D. USSR
Answer : C
194. Which country joined the World Trade Organization(WTO) on Dec. 11, 2001?
A. South Korea
B. China
C. Russia
D. Brazil
Answer : B
195. Slobodan miloseivc is a former president of which country?
A. Belgium
B. Yugoslavia
C. Poland
D. Albania
Answer : B
196. Who was the first Indian Governor-General of India?
A. Lord Mountbatten
B. Rajagopalachari
C. Rajendra Prasad
D. None of these
Answer : B
197. Who wrote the famous poetic drama 'Faust'?
A. Leonardo da Vinci
B. Wolfgang Von Goethe
C. Anton Chekov
D. Voltaire
Answer : B
198. Who was the last Guru of the Sikhs?
A. Guru Govind Singh
B. Guru Tek Bahadur Singh
C. Guru Nanak
D. None of these
Answer : A
199. Which of the following terms denote the superstitious fear of the number thirteen?
A. Thirteephobia
B. Triskadekahobia
C. Tridecaphobia
D. None of these
Answer : B
200. How long did the Kurushektra war last?
A. Fourteen years
B. 27 days
C. 13 months
D. 18 days
Answer : D
201. What does a Phillumenist collect?
A. Old coins
B. Flags of various nations
C. Match box labels
D. Empty cigarette boxes
Answer : C
202. Who was the author of 'Raghuvamsa'?
A. Kannadasan
B. Kalidasa
C. Sivadas
D. Shankaracharya
Answer : B
203. Who said "Some books are to be tasted others to be swallowed, and some few to be chewed and digested"?
A. Francis Bacon
B. H G Wells
C. John Steinbeck
D. Isaac Asimov
Answer : A
204. How many moons (satellites) does the planet Saturn have?
A. Twelve
B. Ten
C. Eight
D. Twenty one
Answer : B
205. Which country is called the 'Land of cakes'?
A. Britain
B. Spain
C. France
D. Scotland
Answer : D
206. Which is the birth place of Hitler?
A. Prussia
B. Hungary
C. Austria
D. France
Answer : C
207. One of the following countries has compulsory military service for women?
A. Phillippines
B. Israel
C. Finland
D. New Zealand
Answer : B
208. Which is the last letter of the Greek alphabet?
A. Omega
B. Sigma
C. Zeta
D. Chi
Answer : A
209. What is the other name for River Ganges?
A. Sapthanadhi
B. Bhageerathi
C. Savithri
D. Bhadravathi
Answer : B
210. In which year did Hitler become the Chancellor of Germany?
A. 1919
B. 1929
C. 1933
D. 1939
Answer : C
211. Which is the largest flightless bird?
A. Ostrich
B. Dodo
C. Albatross
D. Peahen
Answer : B
212. Who is the Chairman and CEO of McCann-Erickson India?
A. Sushil Khanna
B. Ritujoy Chakraborty
C. Sanjay Mukherjee
D. Ritesh Singhal
Answer : B
213. Who is the Chairman of Hindustan Lever Limited?
A. MSV Banga
B. Keki B Dadisheth
C. Arun Adhikari
D. Harish Manwani
Answer : A
214. Who is the chairman of HDFC?
A. Deepak Parekh
B. Y M Deosthalee
C. Janki Ballabh
D. Samir Arora
Answer : A
215. Who is the chairman of State Bank of India?
A. Janki Ballabh
B. Deepak Parekh
C. Rajendra Pawar
D. None of these
Answer : A
216. Who is the chairman of NIIT?
A. Jerry Rao
B. John Reed
C. Rajendra Pawar
D. Harish Menon
Answer : C
217. What was Reckitt and Coleman renamed as?
A. Reckitt Benckiser (India) Ltd
B. Reckitt Switzer (India) Ltd
C. Reckitt Bayer (India) Ltd
D. It was not renamed
Answer : A
218. Who is the chairman of ICICI Bank Ltd?
A. N Vaghul
B. K V Kamath
C. Y C Deveshwar
D. Lalit Gupte
Answer : B
219. Who is the chairman of ITC Ltd?
A. Y C Deveshwar
B. Deosthalee
C. Madhu Singhania
D. Pratap C Reddy
Answer : A
220. Who is the US ambassador to India?
A. Jack Straw
B. Robert Blackwill
C. John Ashcroft
D. Peter Warburg
Answer : B
221. Who is India's ambassador to the US?
A. Montek Singh Ahluwalia
B. Lalit Mansingh
C. Suraj Parekh
D. Rajat Gupta
Answer : B
222. Novartis was born out of the merger of the following two companies
A. Glaxo and Sandoz
B. Ciba and Sandoz
C. Ciba and Smithkline
D. Smithkline and Glaxo
Answer : B
223. Who is the chairman of Procter and Gamble Hygiene and Healthcare India Ltd?
A. Bob Williams
B. B V Patel
C. Rajesh Sinha
D. Kevin Parker
Answer : B
224. Brent Index is associated with which commodity?
A. Crude oil prices
B. Copper futures price
C. Gold futures price
D. Shipping rate index
Answer : A
225. Justice and Development Party of Turkey won the recent elections held in November 200 Who heads the party?
A. Recep Tayyip Erdogan
B. Bulent Ecevit
C. Necdet Sezer
D. Necdet Sezer
Answer : A
226. Approximately what was the quantum of the trade deficit of US for the financial year ending Dec 2001?
A. USD 245 bn
B. USD 720 bn
C. USD
D. 1 tn
E. USD 412 bn
Answer : D
227. What is the currency of Argentina?
A. Peso
B. Dollar
C. Dinar
D. Koruny
Answer : A
228. Which country is the famous mobile phone manufacturer head quartered?
A. Finland
B. Switzerland
C. Sweden
D. Germany
Answer : A
229. Who won the Australian Open 2002 Women's singles?
A. Martina Hingis
B. Jennifer Capriati
C. Venus Williams
D. Monica Seles
Answer : B
230. Which company launched the first mobile phone operation in India?
A. Bharti Airtel
B. Essar
C. Max Touch
D. Modi Telstra
Answer : D
231. Which famous inventor ranks second in the number of patents registered?
A. Thomas Alva Edison
B. James Watt
C. Albert Einstein
D. Edwin Land
Answer : D
232. In terms of overall advertising spends, which country is the second largest in the world, with only the US ahead of it?
A. UK
B. China
C. Japan
D. France
Answer : C
233. Which organisation is the world's largest employer?
A. General Motors
B. General Electric
C. China Electric Company
D. Indian Railways
Answer : D
234. Which mobile phone service provider has taken over from JT Mobile and launched its services in Bangalore and Andhra Pradesh last year?
A. BPL Mobile
B. Orange
C. AirTel
D. Reliance Telecom
Answer : C
235. Which air route has the highest level of international passenger traffic in the world?
A. London-New York
B. London-Paris
C. NewYork-Singapore
D. Dubai-London
Answer : B
236. Which leading Indian tyre company owns Vikrant Tyres after its takeover in the late 1990s?
A. MRF
B. Modi Stone
C. Dunlop
D. JK Tyres
Answer : D
237. Which industrial family owns the majority stake in the successful Hero group, a leader in the motorcycle segment in the country in collaboration with Honda, Japan?
A. Hinduja
B. Munjal
C. Firodia
D. Singhania
Answer : B
Verbal Section : Analogy
Directions:
Each of the questions below consists of two words that have a certain relationship to each other, followed by five lettered pairs of related words. Select the lettered pair of words.
1. DEBATER : LARYNGITIS
A. pedestrian : lameness
B. actor : aplause
C. doctor : diagnosis
D. swimmer : wet
E. writer : paper
Answer: A
2. INKBLOT : EYECHART
A. oculist : ophthalmologist
B. blotter : spectacles
C. psychiatrist : optometrist
D. physician : specialist
E. blurs : letters
Answer: C
3. LIGNEOUS : WOOD
A. cellular : microbe
B. nautical : water
C. igneous : rock
D. osseous : bone
E. fossilized : plant
Answer: D
4. SHRINE : PILGRIM
A. defeat : loser
B. peak : climber
C. rescue : danger
D. election : contestant
E. direction : driver.
Answer: B
5. RIVAL : COMPETITION
A. litigant : morality
B. maverick : co-operation
C. mentor : praise
D. sycophant : flattery
E. medicant : confusion.
Answer: D
6. ANGLE : DEsatE
A. area : square inch
B. milk : quart
C. society : classes
D. letter : alphabet
E. time : minutes
Answer: A
7. CONFIRMED : INVETERATE
A. knowledge : supposed
B. financial : bankrupt
C. immature : callow
D. credible : incredible
E. careful: punishing
Answer: B
8. LULLABY : BARCAROLE
A. birth : marriage
B. night : morning
C. cradle : gondola
D. song : poem
E. carol : sonneteer
Answer: C
9. ZOOLOGY : ANIMALS
A. ecology : pollution
B. botany : plants
C. chemistry : atoms
D. history : people
E. mathematics : geometry
Answer: A
10. DORY : VAN
A. dairy : cow
B. fish : vehicle
C. freighter : caisson
D. runners : wheels
E. danish : Dutch
Answer: C
11. LIQUEFY : PETRIFY
A. cash in : strengthen
B. insolvent : bankrupt
C. water : stone
D. soften : frighten
E. solvent : rich
Answer: C
12. AMBULATORY : BEDRIDDEN
A. strong : weak
B. wheelchair : bed
C. free : confined
D. healthy : sick
E. broken : arm
Answer: C
13. CYNOSURE : BRILLIANT
A. word : common
B. student : attentive
14. rock : large DRUM : TYMPANI
A. piano : orchestra
B. cornet : percussion
C. stick : baton
D. violin : viola
E. oboe : woodwind.
Answer: E
15. EXTROVERT : RETICENT
A. reprobate : humility
B. strategist : decisiveness
C. zealot : loyalty
D. maverick : conformity
E. renegade : ambition.
Answer: D
16. GROW : BURGEON
A. flourish : thrive
B. transport : enrapture
C. beat : palpitate
D. evolve : multiply
E. wrot : decay
Answer: C
17. HOAX : DECEIVE
A. gimmick : wheedle
B. filibuster : delay
C. boast : cajole
D. lottery : disburse
E. scandal : vilify
Answer: B
18. BODY GUARD : PERSON
A. teacher : pupil
B. mayor : city
C. police officer : traffic
D. soldier : country
E. secretary : office
Answer: D
19. MUFFLE : SOUND
A. conceal : secret
B. assuage : grief
C. maul : object
D. extract : flavor
E. endure : agony
Answer: B
20. CENSORSHIP : INFORMATION
A. cultivation : erosion
B. philanthropy : generosity
C. frugality : constraint
D. sampling : measurement
E. sanitation : disease
Answer: E
21. HYGROMETER : BAROMETER
A. snow : rain
B. humidity : pressure
C. water : mercury
D. temperature : weather
E. forecast : rain.
Answer: B
22. EXEMPTION : EXCLUSIONS
A. discharge : elimination
B. debarment : prevention
C. immunity : isolation
D. forgive : condone
E. enclosure : open.
Answer: C
23. FEBRILE : ILLNESS
A. classic : cultivation
B. delusional : insanity
C. eccentric : discrimination
D. tenacious : astonishment
E. juvenile : maturity.
Answer: B
F.
G. magnet : attractive
H. map : legible
Answer: D
24. EXHORT : SUGGEST
A. conspire : plan
B. tamper : adjust
C. crave : accept
D. goad : direct
E. instruct : teach
Answer: D
25. CHRONOMETER : SUNDIAL
A. measurement : visibility
B. chronology : analogy
C. computer : abacus
D. watch : ray
E. reduction : enlargement
Answer: C
26. FOOTBALL : GRIDIRON
A. wrestling : mat
B. court : tennis
C. bowling : floor
D. rugby : arena
E. baseball : diamond
Answer: E
27. SAND PAPER : ABRASIVE
A. gasoline : refined
B. grativity : irritant
C. polish : floors
D. acrylic : emulsion
E. oil : lubricant.
Answer: E
28. DIAPHANOUS : CACOPHONOUS
A. translucent : transparent
B. transparent : noisy
C. sheer : opaque
D. harmonious : discordant
E. twofold : multiple.
Answer: B
29. INFANCY : SENILITY
A. january : October
B. incipient : critical
C. day : night
D. conclusion : climax
E. dawn : dusk.
Answer: E
30. MENDICANT : IMPECUNIOUS
A. hat : askew
B. liar : poor
C. complainer : petulant
D. critic : quizzical
E. philanthrophist : prodigal.
Answer: C
31. TANGO : DANCE
A. stanza : line
B. tonality : instrumentation
C. arabesque : theme
D. rhyme : pattern
E. elegy : poem
Answer: E
32. EXPURGATE : PASSAGE
A. abridge : text
B. filter : water
C. irritate : wound
D. burn : book
E. cancel : plan
Answer: B
33. IMPECUNIOUS : HOVEL
A. progress : prosper
B. mendicant : evasion
C. prosperity : poverty
D. mendacious : cringe
E. affluent : mansion
Answer: E
34. APIARY : BEE
A. museum : painting
B. dam : water
C. arboretum : tree
D. forum : speech
E. planetarium : star
Answer: C
35. DISAPPROBATION : CONDEMN
A. calumny : eulogise
B. enigma : enlighter
C. fallacy : diseminate
D. exhortation : urge
E. solvency : deploy.
Answer: D
36. GEM : TURQUOISE
A. lettuce : green
B. pear : orange
C. stone : magnetta
D. vine : cherry
E. flower : violet.
Answer: E
37. BIZARRE : EXOTIC
A. wild : tame
B. lively : livid
C. stage : dancer
D. commonplace : routine
E. ordinary : exceptional.
Answer: D
38. ENTREPRENEUR : LABORER
A. mediator : conflict
B. capitalism : communism
C. profits : wages
D. arbitrator : capitalist
E. moonlighting : worker.
Answer: C
39. ANTIMACASSAR : SOFA
A. picture : frame
B. rug : floor
C. pillow : bed
D. door : window
E. table : chair.
Answer: B
40. NOTABLE : NOTORIOUS
A. heinous : atrocious
B. philandering : pleasant
C. philanthropic : miserly
D. nefarious : secret
E. philanthropic : benevolent.
Answer: E
41. BABBLE : TALK
A. though : blank
B. look : espy
C. wink : eye
D. leer : ogle
E. simper : smile.
Answer: E
42. ALCOVE : RECESS
A. column : entrance
B. foundation : building
C. dome : roof
D. turret : chimney
E. foyer : ballroom
Answer: C
43. LURK : WAIT
A. boost : elevate
B. deplete : drain
C. abscond : depart
D. bilk : cheat
E. topple : stabilize
Answer: C
44. ALCHEMY : SCIENCE
A. nostrum : remedy
B. sideshow : carnival
C. ploy : tactic
D. forgery : imitation
E. burlesque : comedy
Answer: A
45. NEEDLE : KNIT
A. bait : fish
B. match : fire
C. loom : weave
D. soap : wash
E. bed : sleep
Answer: C
46. APOSTATE : RELIGION
A. traitor : country
B. renegade : Indian
C. loyal : faith
D. vital : church
E. diloyal : colonies.
Answer: A
47. PLEBISCITE : UKASE
A. lack : abundance
B. vote : musical instrument
C. cancel : construct
D. public : ruler
E. written : oral
Answer: D
48. EQUIVOsatE : COMMITMENT
A. collaborate : falsifisation
B. fabrisate : explanation
C. procrastinate : action
D. expostulate : confusion
E. implisate : exposition
Answer: C
49. MORPHINE : SEDATES
A. oil : smears
B. bandage : protects
C. drug : addicts
D. liquor : sedates
E. medicine : soothes
Answer: D
50. STICKLER : APPROXIMATION
A. Lluggard : indolence
B. connoisseur : anachronism
C. scientist : theorizing
D. leader : guidance
E. purist : adulteration
Answer: E
51. LOOM : WEAVE
A. couch : sleep
B. needle : knit
C. soap : wash
D. machine : stitch
E. bail : fish
Answer: B
52. RELAPSE : CONVALESCENCE
A. dissonance : harmony
B. feudalism : industrialization
C. repetition : monotony
D. impasse : debate
E. recidivism : rehavbilitation.
Answer: E
53. BOUQUET : FLOWERS
A. corn : husk
B. woodpile : logs
C. forest : thicket
D. mist : fog
E. drift : snow.
Answer: B
54. TRIANGLE : QUADRILATERAL
A. rectangle : octagon
B. cone : cube
C. pentagon : hexagon
D. plane : solid
E. regular : symmetrical.
Answer: C
55. SARTORIAL : TAILOR
A. thespian : designer
B. rhetorical : questioner
C. pictorial : musician
D. histrionic : singer
E. terpsichorear : dancer.
Answer: E
56. NECROMANCY : GHOSTS
A. magic : legerdemain
B. alchemy : gold
C. sorcery : spirits
D. fortune_telling : gypsies
E. romance : stories.
Answer: C
57. SUBPOENA : WITNESS
A. hire : laborer
B. tax : worker
C. elect : officer
D. suborn : judge
E. conscript : soldier
Answer: E
58. INVINCIBLE : SUBDUED
A. expensive : bought
B. inconsistent : expressed
C. bolted : separated
D. impervious : damaged
E. imprudent : enacted
Answer: D
59. BURLESQUE : PLAY
A. operetta : symphony
B. limerick : sonnet
C. doggerel : verse
D. table : narration
E. sketch : drawing
Answer: C
60. DUPLIsatE : ALTER
A. greet : ignore
B. exchange : return
C. shake : stabilize
D. stretch : shrink
E. eradisate : implisate
Answer: D
61. RIG : CONTEST
A. solve : conundrum
B. predict : race
C. repudiate : thesis
D. gerrymander : district
E. incriminate : evidence
Answer: D
62. ARBORETUM : TREES
A. aviary : birds
B. satenhouse : garden
C. museum : painters
D. grove : forest
E. zoo : range
Answer: D
63. HYPOTHESIS : EXPERIMENTATION
A. reality : fantasy
B. opinion : debate
C. film : camera
D. predisation : conclusion
E. science : success
Answer: B
64. BULLET : BARREL
A. fame : films
B. train : track
C. idea : brain
D. plane : clouds
E. water : boat
Answer: B
65. WINE : GRAPES
A. liquor : intoxisating
B. whiskey : hops
C. champagne : raisins
D. vodka : potatoes
E. vineyard : winery.
Answer: D
66. DEBATE : FORENSIC
A. concerto : harmonizing
B. drama : histrionic
C. opera : spoken
D. argument : domestic
E. novel : original.
Answer: B
67. NOISOME : GARBAGE
A. heavy : metal
B. warmth : snow
C. fragrant : incense
D. liquid : perfume
E. loud : music.
Answer: C
68. CONDUIT : WATER
A. behaviour : liquid
B. electricity : television
C. artery : blood
D. wire : sound
E. pump : oil.
Answer: C
69. VINDIsatE : REPREHENSIBLE.
A. sad : sorrow
B. bitter : sad
C. mild : serious
D. solid : porous
E. vivid : obsequious.
Answer: C
70. TERMAGANT : SHREW
A. virago : harpy
B. anteater : mouse
C. supporter : nag
D. single : married
E. male : female.
Answer: A
71. PARQUET : WOOD
A. color : painting
B. mosaic : glass
C. potpourri : medley
D. collage : tapestry
E. linoleum : marble
Answer: B
72. FIRM : INTRANSIGHT
A. faithful : resolute
B. improvident : industrious
C. vague : inattentive
D. concerned : obsessed
E. malleable : tractable
Answer: D
73. EPAULET : SHOULDER
A. medal : chest
B. decoration : uniform
C. knapsack : back
D. sword : scabbard
E. sash : window
Answer: A
74. ANACHRONISM : CHRONOLOGY
A. tradition : custom
B. variations : incongruity
C. fallacy : logic
D. archetype : paradigm
E. debauchery : appetites
Answer: C
75. DETRITUS : GLACIERS
A. thaw : cold
B. snow : icebergs
C. sediment : bottom
D. silt : rivers
E. dregs : society
Answer: D
76. OUTSKIRTS : TOWN
A. water : goblet
B. margin : page
C. rung : ladder
D. hangar : airplane
E. trunk : tree
Answer: B
77. SAW : CARPENTER
A. Scissors : tailor
B. Wagon : farmer
C. Brush : painter
D. Typewriter : author
E. Trowel : bricklayer
Answer: A
78. SPIKE : TACK
A. bullet : wound
B. knife : cut
C. arrow : bow
D. spear : dart
E. pin : needle
Answer: D
79. INIQUITOUS : DISOBEDIENT
A. adult : child
B. hostile : cool
C. quiescent : lethargic
D. inflammable : flammable
E. inequitable : equitable
Answer: B
80. BALEFUL : MENACE
A. brusque : retort
B. competent : achievement
C. placid : boredom
D. flirtatious : affection
E. solicitous : concern
Answer: E
81. PARENTHESIS : EXPLANATION
A. ellipsis : omission
B. asterisk : exaggeration
C. synopsis : affectation
D. apostrophe : annotation
E. synthesis : interpolation
Answer: A
82. CENSUS : POPULATION
A. manifest : debts
B. roster : audience
C. itinerary : journeys
D. inventory : merchandise
E. state : incumbents
Answer: D
83. STANZA : POEM
A. mimicry : pantomime
B. duet : chorus
C. act : opera
D. rhyme : verse
E. pirouette : ballet
Answer: C
84. ALACRITY : APATHETIC
A. compliance : deft
B. temerity : timid
C. despotism : arrogant
D. candor : bungling
E. tenacity : eager
Answer: B
85. FIRE : STORM
A. whale : minnow
B. speech : shout
C. plant : flower
D. wind : temperature
E. tornado : hurricane
Answer: E
86. COLLUSION : CONSPIRATORS
A. identifisation : arbitrators
B. co-operation : partners
C. conclusion : messengers
D. revision : corespondents
E. attribution : interpreters
Answer: B





Verbal Section : Sentence Completions
Directions:
Each sentence below has one or two blanks. Each blank shows that something has been omitted. Under each sentence five words are given as choice. Choose the one correct word for each blank that best fits the meaning of the sentences as a whole.
1. Having no sense of moral obligation, shipler was as little subject to the - of conscience after he acted as he was motivated by its - before he acted.
A. balm - eloquence
B. qualms - atonement
C. reproaches - prompting's
D. rewards - chastisement
E. ridicule - allure
Answer: C
2. Slang is a language that rolls up its sleeves, spits on its hands and -
A. goes to work
B. stays cool
C. embarrasses its user
D. communicates
E. puts its foot down
Answer: B
3. Famous among job seekers for its - , the company, quite apart from generous salaries, bestowed on its executives annual bonuses and such - as low - interest home mortgages and company cars.
A. largesse - perquisites
B. altruism - credits
C. magnanimity - reparations
D. discernment - prerogatives
E. inventiveness - benefits
Answer: A
4. Moving and parked, the automobile devours urban land, leaving the buildings as mere - of habitable space in a sea of dangerous and ugly traffic
A. asylums
B. remnants
C. blocks
D. mountains
E. islands
Answer: E
5. We never believed that he would resort to - in order to achieve his goal; we always regarded him as a - man.
A. charm - insincere
B. necromancy - pietistic
C. logic - honorable
D. prestidigitation - articulate
E. subterfuge - honest
Answer: E
6. The Sociologist responded to the charge that her new theory was - by pointing out that it did not in fact contradict accepted sociological principles.
A. unproven
B. banal
C. superficial
D. complex
E. heretical
Answer: E
7. He was so - by the interplay of the colors that varied in brilliance and pattern as the music rose and fell, that he asked the price of the device.
A. overwrought
B. penalized
C. repelled
D. inteugued
E. penalized
Answer: D
8. The absence of a sense of outrage and grief at national tragedy is an - of moral responsibility.
A. intervention
B. energising
C. abdication
D. administration
E. actuation.
Answer: C
9. In an effort to - its operations, the corporation announced it was acquiring a - company in a different type of manufacturing.
A. diversify - subsidiary
B. adumberate - solvent
C. multiply - protracted
D. intensify - fluctuating
E. establish - sequential.
Answer: A
10. Samuel Clemens chose the - Mark Twain as a result of his knowledge of river boat piloting.
A. mountebank
B. protagonist
C. misanthrope
D. hallucination
E. pseudonym.
Answer: E
11. To meet all - a source of - electrical power was added to the train's engine.
A. Integuments - parallel
B. possibilities - incidental
C. amenities - diverse
D. contingencies - auxiliary
E. conveniences - automatic.
Answer: D
12. Since the escaping vapors proved to be highly - measures were at once taken for the - of the experiments.
A. Volatile - ratification
B. Observable - insulation
C. Gaseous - reduction
D. Noxious - cessation
E. Incriminating - destruction.
Answer: D
13. Eric Fromm does not agree that man is - in Freudian sexual dilemmas for if the - that man creates can be changed for the better, there is hope that the state of man can be changed as well.
A. Tortured - goals
B. Trapped - institutions
C. Caught - symbols
D. Engulfed - life
E. Confused - meanings.
Answer: B
14. Despite assorted effusion to the contrary, there is no necessary link between scientific skill and humanism, and quite possibly, there may be something of a - between them.
A. dichotomy
B. congruity
C. reciprocity
D. fusion
E. generosity
Answer: E
15. The most technologically advanced societies have been responsible for the catatest - indeed savagery seems to be indirect proposition to -
A. inventions - know-how
B. wars - viciousness
C. triumphs - civilizations
D. atrocities - development
E. catastrophes - ill-will
Answer: D
16. Sponsors of the bill were-because there was no opposition to it within the legislative, until after the measure had been signed into law.
A. well-intentioned
B. persistent
C. detained
D. unreliable
E. relieved.
Answer: B
17. Ecology, like economics, concerns itself with the movement of valuable - through a complex network of producers and consumers.
A. nutrients
B. dividends
C. communications
D. artifacts
E. commodities.
Answer: C
18. Having fully embraced the belief that government by persuasion is preferable to government by - the leaders of the movement have recently - most of their previous statements supporting totalitarianism.
A. proclamation - codified
B. coercion - repudiated
C. participation - moderated
D. intimidation - issued
E. demonstration - deliberated.
Answer: B
19. It would be difficult for one so - to be led to believe that all men are equal and that we must disregard race, color and creed.
A. tolerant
B. democratic
C. broadminded
D. emotional
E. intolerant.
Answer: E
20. Many philosophers agree that the verbal aggression of profanity in certain redical newspapers is not - or childish, but an assault on - essential to the revolutionary's purpose.
A. insolent - sociability
B. trivial - decorum
C. belligerent - fallibility
D. serious - propriety
E. deliberate - affectation.
Answer: B
21. The gypsy girl, decked out in - finery, and with her disheveled hair streaming over shoulders, was indeed a - sight.
A. verdant - wistful
B. sartorial - flagrant
C. specious - poignant
D. tawdry - bizarre
E. opulent - debonair
Answer: D
22. Yellow fever, the disease that killed 4,000 Philadelphians in 1793, and so - Memphis, Tennessee, that the city lost its charter, has reappeared after nearly two decades in - in the western hemisphere.
A. disabled - quarantine
B. decimated - abeyance
C. terrorized - contention
D. ravaged - secret
E. coupled - quiescence
Answer: B
23. The painting was larger than it appeared to be, for hanging in a darkened recess of the chapel, it was - by the perspective.
A. embellished
B. improved
C. jeopardised
D. aggrandized
E. diminished
Answer: E
24. We have in America - speech that is neither American, Oxford English, nor English but a - of all three.
A. motley - miracle
B. nasal - blend
C. feigned - patchwork
D. mangled - medley
E. hybrid - combination
Answer: E
25. Old beliefs die hard, even when jobs become - the long - standing fear that unemployment could return at a moments notice -
A. protected - subsided
B. vacant - perished
C. available - receded
D. plentiful - persisted
E. easier - charged
Answer: D
26. Clearly refuting sceptics, researches have - not only that gravitational radiation exists but that it also does exactly what the theory - it should do.
A. supposed - asserted
B. voubted -warranted
C. assumed - deduced
D. demonstrated - predicted
E. estimated - accepted
Answer: D
27. The Neolatonists' conception of a deity, in which perfection was measured by abundant fecundity, was contradicted by that of the Aristotelians, in which perfection was displayed in the - of creation.
A. variety
B. economy
C. profusion
D. clarity
E. precision.
Answer: B
28. It is a great - to be able to transfer useful genes with as little extra gene material as possible, because the donor's genome may contain, in addition to desirable genes, many genes with - effects.
A. Disappointment - superfluous
B. Convenience - exquisite
C. Advantage - deleterious
D. Accomplishment - profound
E. Misfortune - unpredictable.
Answer: C
29. While admitting that the risks incurred by use of the insecticide were not - the manufacturer's spokesperson argued that effective - were simply not available.
A. indeterminable - safeguards
B. unusual - alternatives
C. inconsequential - substitutes
D. proven - antidotes
E. increasing - procedures.
Answer: C
30. Human reaction to the realm of though is often as strong as that to sensible presences; our higher moral life is based on the fact that - sensations actually present may have a weaker influence on our action than do ideas of - facts.
A. emotional - impersonal
B. familiar - symbolic
C. disturbing - ordinary
D. material - remote
E. defenitive - controvoisial.
Answer: D
31. Parts of seventeenth-century Chinese pleasure gardens were not necessarily intended to look -they were designed expressly to evoke the agreeable melancholy resulting from a sense of the - of natural beauty and human glory.
A. great - immutability
B. joyful - mortality
C. conventional - wildness
D. cheerful - transitoriness
E. colorful - abstractness.
Answer: D
32. Despite the - of many of their colleagues, some scholars have begun to emphasize ''pop culture'' as a key for - the myths, hopes, and fears of contemporary society.
A. pedantry - reinstating
B. enthusiasm - symbolizing
C. skepticism - deciphering
D. antipathy - involving
E. discernment - evaluating.
Answer: C
33. If duty is the natural - of one's the course of future events, then people who are powerful have duty placed on them whether they like it or not.
A. outgrowth - control over
B. arbiter - responsibility for
C. correlate - understanding of
D. determinant - involvement in
E. mitigant - preoccupation with .
Answer: A
34. No hero of ancient or modern times can surpass the Indian with his lofty contempt of death and the - with which he sustained the cruelest coffliction.
A. guide
B. assent
C. reverence
D. fortitude
E. concern
Answer: D
35. The hostess attempted to - a romantic atmosphere that would bring the two young people together in -
A. expand - fealty
B. present - collusion
C. simulate - conflict
D. introduce - cacophony
E. contrive - matrimony
Answer: E
36. The struggle of the generations is one of the obvious constants of human affairs; therefore, it may be presumptuous to suggest that the rivalry between young and old in western society during the current decade is - critical.
A. archetypally
B. perennially
C. disturbingly
D. uniquely
E. cautiously
Answer: D
37. Even though in today's Soviet union the - Muslim clergy have been accorded power and privileges, the Muslim laity and the rank - and - file clergy still. Have little - to practice their religion.
A. adversaries of - inclination
B. traditionalists among - incentive
C. practitioners among - opportunity
D. leaders of - latitude
E. dissidents within -obligation
Answer: D
38. Unlike the Shakespearean plays, The ''closet dramas'' of the nineteenth century were meant to be - rather than -
A. seen - acted
B. read - acted
C. produced - acted
D. quiet - loud
E. sophisticated - urbane
Answer: B
39. The little - known but rapidly expanding use of computers in mapmaking is technologically similar to the more - uses in designing everything from bolts to satellites.
A. ingenuous
B. recent
C. secure
D. publicized
E. successful
Answer: D
40. Although his out numbered troops fought bravely, the general felt he had no choice but to - defeat and - a retreat.
A. oversee - reject
B. acknowledge - order
C. hasten - suggest
D. seek - try
E. overcome - request
Answer: B
41. Some scientists argue that carbon compounds play such a central role in life on earth because of the possibility of - resulting from the carbon atom's ability to form an unending series of different molecules.
A. variety
B. stability
C. deviations
D. invigorations
E. reproduction.
Answer: A
42. It would be difficult for one so - to be led to believe that all men are equal and that we must disregard race, color and creed.
A. intolerant
B. democratic
C. emotional
D. patient
E. broadminded.
Answer: A
43. An occasional - remark spoiled the - that made the paper memorable.
A. colloquial
B. trite - cliches
C. urbane - sophisticated
D. hackneyed - originality
E. jovial - fun.
Answer: D
44. Broadway audiences have become inured to - and so - to be pleased as to make their ready ovations meaningless as an indicator of the quality of the production before them.
A. cleverness : eager
B. condescension : disinclined
C. sentimentality : reluctant
D. mediocrity : desperate
E. histrionics : unlikely
Answer: D
45. Nineteenth - century scholars, by examining earlier geometric Greek art, found that classical Greek art was not a magical - or a brilliant - blending Egyptian and Assyruin art, but was independently evolved by Greeks in Greece.
A. conversion - annexation
B. apparition - amalgam
C. stratagem - appropriation
D. paradigm - construct
E. example - synthesis
Answer: B
46. The - tones of the flute succeeded in - his tense nerves.
A. rhapsodic - minimising
B. blatant - enhancing
C. hovendous - calming
D. vibrant - portraying
E. mellifluous - soothing.
Answer: E
47. Without the psychiatrist's promise of confidentiality, trust is - and the patient's communication limited; even though confidentiality can thus be seen to be precious in thercopy, moral responsibility sometimes requires a willingness to - it.
A. lost - forget
B. implicit - extend
C. impaired - sacrifise
D. ambiguous - apply
E. assumed - examine.
Answer: C
48. Employers who retire people who are willing and able to continue working should realize that - age is not an effective - in determining whether an individual is capable of working.
A. intellectual - criterion
B. Chronological - criterion
C. Physical - barrier
D. deteriorating - value
E. chronological - factor
Answer: B
49. As the sun rose, the morning mists were borne away on the - like strands of -
A. whirlwind - flotsam
B. wind - cactus
C. morass - tundra
D. zephyr - gossamer
E. holocaust - taffeta
Answer: D
50. The playwright was known not for his original ideas that had been propounded by others.
A. rejection
B. consideration
C. invention
D. reiteration
E. plagiarism
Answer: E
51. Ironically, the party leaders encountered no catater - their efforts to build as Procatssive Party than the - of the procatssive already elected to the legislature.
A. obstacle to - resistance
B. support for - advocacy
C. praise for - reputation
D. threat to - promise
E. benefit - success
Answer: A
52. The simplicity of the theory - its main attraction - is also its - for only by - the assumptions of the theory is it possible to explain the most recent observations made by researchers.
A. glory - rejecting
B. liability - accepting
C. undoing - supplementing
D. downfall - considering
E. virtue - qualifying
Ans : C
53. Not only the - are fooled by pcopagandas we can all be misled if we are not -
A. people - mature
B. ignorant - cynical
C. masses - cautious
D. uncultured - concerned
E. gullible - wary
Answer: E
54. -- merciful by nature, he was - toward the murderer.
A. although - unmoving
B. while - unjust
C. truly - indicative
D. though - kind
E. albeit - implacable
Answer: E
55. When the news of his - with the enemy become known, he was hanged in -
A. collusion - effigy
B. conversation's - earnest
C. involvement - martyrdom
D. complacency - retaliation
E. bickering - response
Answer: D
56. That the Third Battalion's fifty percent casually rate transformed its assault on Hill 306 from a brilliant stratagem into a debacle does not - eyewitness reports of its commander's extra-ordinary - in deploying his forces.
A. invalidate - brutality
B. gainsay - cleverness
C. underscore - ineptitude
D. justify - rapidity
E. corroborate -determination
Ans : B
57. No longer - by the belief that the world around us was expressly designed for humanity, many people try to find intellectual - for that lost certainty in astrology and in mysticism.
A. satisfied - reasons
B. reassured - justifications
C. restricted - parallels
D. sustained - substitutes
E. hampered - equivalents
Ans : D
58. In eighth-century Japan, people who - wasteland were rewarded with official ranks as part of an effort to overcome the shortage of - fields.
A. cultivated - domestic
B. located - desirable
C. conserved - forested
D. reclaimed - arable
E. irrigated - accessible.
Answer: D
59. Clearly refuting sceptics, researchers have - not only that gravitational radiation exists but that it also does exactly what the theory- it should do.
A. assumed - deducted
B. estimated - accepted
C. supposed - asserted
D. doubted - warranted
E. demonstrated - predicted.
Answer: E
60. Melodramas, which presented stark oppositions between innocence and criminality, virtue and corruption, good and evil, were popular precisely because they offered the audience a world - of -
A. deprived - polarity
B. full - circumstantiality
C. bereft - theatricality
D. devoid - neutrality
E. composed - adversity.
Answer: D
61. The fact that the - of confrontation is no longer as popular as it once was - procatss in race relations.
A. insidiousness - reiterates
B. practice - inculcates
C. glimmer - foreshadows
D. technique - presages
E. reticence - indicates
Answer: D
62. A child should not be - as being either very shy or over - agcatssive.
A. categorized
B. instructed
C. intoned
D. distracted
E. refrained
Answer: A
63. President Anwar el - Sadat of Egypt, disregarding - criticism in the Alab world and in his own Government, - accepted prime minister Menahem Begin's invitation to visit Israel in order to address the Israeli parliament.
A. acrimonious - formally
B. blemished - stiffly
C. categorical - previously
D. malignant - plaintively
E. charismatic - meticulously
Answer: A
64. In his usual - manner, he had insured himself against this type of loss.
A. pensive
B. providential
C. indifferent
D. circumspect
E. caustic
Answer: D
65. As man reached the stars, a booming population threatened to destroy the - of life on his home planet and even its chances for -
A. Quality - survival
B. Basis - growth
C. Existence - upliftment
D. chances - improvement
E. meaning - understanding.
Answer: A
66. Until the current warming trend exceeds the range of normal climatic fluctuations, there will be, among scientists, considerable - the possibility that increasing levels of atmosphere Co2 can cause long term warming effects
A. interest in
B. uncertainty about
C. experimentation on
D. enthusiasm for
E. worry about
Answer: B
67. In the current research program, new varieties of apple trees are evaluated under different agricultural - for tree size, bloom density, fruit size,- to various soils, and resistance to pests and disease.
A. conditions - adaptability
B. configurations - propensity
C. circumstances - proximity
D. auspices - susceptibility
E. regulations - conformity
Answer: A
68. For many young people during the roaring twenties, a disgust with the excesses of American culture - a wanderlust to provoke an exodus abroad.
A. reflected
B. stymied
C. conflicted with
D. overwhelmed
E. combined with
Answer: E
SAT Sample Questions
Reading Comprehension
Directions:
Each SAT sample reading comprehension passage in this section is followed by questions based on the content of the reading passage. Read the SAT sample comprehension passage carefully and chose the best answer to each question. The questions are to be answered on the basis of what is stated or implied in the passage.
1. MARK HUGHES is a master of the fine art of survival. His Los Angeles-based Herbalife International Inc. is a pyramid outfit that peddles weight-loss and nutrition concoctions of dubious value. Bad publicity and regulatory crackdowns hurt his U.S. business in the late 1980s. But Hughes, 41, continues to enjoy a luxurious lifestyle in a $20 million Beverly Hills mansion. He has been sharing the pad and a yacht with his third wife, a former Miss Petite U.S.A. He can finance this lavish lifestyle just on his salary and bonus, which last year came to $7.3 million.
He survived his troubles in the U.S. by moving overseas, where regulators are less zealous and consumers even more naive, at least initially. Today 77% of Herbalife retail sales derive from overseas. Its new prowling grounds: Asia and Russia. Last year Herbalife's net earnings doubled, to $45 million, on net sales of $632 million. Based on Herbalife's Nasdaq-traded stock, the company has a market capitalization of $790 million, making Hughes 58% worth $454 million.
There's a worm, though, in Hughes apple. Foreigners aren't stupid. In the end they know when they've been had. In France, for instance, retail sales rose to $97 million by 1993 and then plunged to $12 million last year. In Germany sales hit $159 million in 1994 and have since dropped to $54 million.
Perhaps aware that the world may not provide an infinite supply of suckers, Hughes wanted to unload some of his shares. But in March, after Herbalife's stock collapsed, he put off a plan to dump about a third of his holdings on the public.
Contributing to Hughes' woes, Herbalife's chief counsel and legal attack dog, David Addis, quit in January. Before packing up, he reportedly bellowed at Hughes, "I can't protect you anymore." Addis, who says he wants to spend more time with his family, chuckles and claims attorney-client privilege.
Trouble on the home front, too. On a recent conference call with distributors, Hughes revealed he's divorcing his wife, Suzan, whose beaming and perky image adorns much of Herbalife's literature.
Meanwhile, in a lawsuit that's been quietly moving through Arizona's Superior Court, former Herbalife distributor Daniel Fallow of Sandpoint, Idaho charges that Herbalife arbitrarily withholds payment to distributors and marks up its products over seven times the cost of manufacturing. Fallow also claims Hughes wanted to use the Russian mafia to gain entry to that nation's market.
Fallow himself is no angel, but his lawsuit, which was posted on the Internet, brought out other complaints. Randy Cox of Lewiston, Idaho says Herbalife "destroyed my business" after he and his wife complained to the company that they were being cheated out of their money by higher-ups in the pyramid organization.
Will Hughes survive again? Don't count on it this time.
1. Herbalife Inc is based in:
A. Los Angeles
B. Columbus
C. New York
D. Austin
Answer: A
2. Daniel Fallow:
A. Was a former attorney for Hughes
B. Was a former distributor of Herbalife
C. Co-founded Herbalife
D. Ran Herbalife's German unit
Answer: B
3. Which of the following countries is mentioned where Hughes operated Herbalife?
A. India
B. China
C. Germany
D. Ukraine
Answer: C
4. The complaint of Randy Cox of Lewiston, Idaho, against Herbalife was:
A. The company did not pay them their dues
B. The products supplied by Hughes were inferior
C. Their higher-ups in the pyramid cheated them
D. Hughes had connections with the Russian mafia
Answer: C
5. Which of the following countries is NOT mentioned in the passage?
A. Russia
B. USA
C. France
D. Italy
Answer: D
6. In the year in which Hughes' salary and bonuses came to US$ 7.3 million, what was the retail sales for Herbalife in France?
A. $12 million
B. $159 million
C. $54 million
D. $97 million
Ans :A
7. At the time when this article was written, if Herbalife had had a market capitalisation of $ 1 billion, what would have been Hughes' share?
A. $420 million
B. $580 million
C. $125 million
D. $500 million
Answer: B
2. In any country, the wages commanded by the laborers who have comparable skills but who work in various industries are determined by the productivity of the least productive unit of labour, i.e. the unit of labour which works in the industry which has catatest economic disadvantages. We will represent the various opportunities of employment in a country like united states by Symbols. A standing for a group of industries in which we have exceptional economic advantage over foreign countries; B for a group in which our advantages are less; E , one in which they are still less; D, the group of industries in which they are the least of all.
When our population is so small that all our labour can be engaged in the group represented by A, productivity of labour and (therefore wages) will be at their maximum. when our population increases so that some of the labour will have to work in group B, the wages of all labour must decline to the level of productivity in that group. But no employer, without government aid, will yet be able to afford to hire labour to exploit the opportunities, represented by E and D, unless there is a further increase in population.
But suppose that the political party in power holds the belief that we should produce everything that we consume, that the opportunities represented by E and D should also be exploited. The commodities, that the industries composing C and D will produce have been hitherto obtained from abroad in exchange for commodities produce by A and B. The government now renders this difficult by imposing high duties upon the former class of commodities. This means that workers in A and B must pay higher prices for what they buy, but do not receive higher prices for what they sell.
After the duty has gone into effect and the prices of commodities that can be produced by C and D have risch sufficiently enterprises will be able to hire labour at the wages prevailing in A and B and establish industries in C and D. So far as the remaining labours in A and B buy the products of C and D ,the difference between the price which they pay for these product and the price they would pay it they were permitted to import those products duty-free is a tax paid not to the government, but to the producers in C and D, to enable the later to remain in business. It is on uncompensated deduction from the natural earnings of the labourers in A and B. nor are the workers in C and D paid as much, estimated in purchasing power as they would have received if they had been allowed to remain in A and B under the earlier conditions.
1. The authors main point is that
A. The government ought to subsidize C and D
B. Wages ought to be independent of international trade
C. It is impossible to attain national self sufficiency
D. The varying productivity of the various industries leads tot he inequalities in wages of workers in these industries
E. A policy that draws labour from the fields of catater natural productiveness to fields of lower natural productiveness tends to redirect purchasing power.
Answer: E
2. No employer, without government aid will yet be able to afford to hire labour to exploit the opportunities represented by C and D because
A. The population has increased
B. Productivity of labour is not at the maximum
C. Productivity would drop correspondingly with the wages of labour
D. We cannot produce everything we consume
E. Enterprises would have to pay wages equivalent to those obtained by workers in A and B while producing under catater disadvantages.
Answer: E
3. When C and D are established, workers in these industries
A. Receives wages equal to those workers in A and B
B. Receives higher wages than do the workers in A and B
C. Are not affected so adversely by the levying of duties as are workers in A and B
D. Must be paid by government funds collected from the duties on imports.
E. Receive lower wages than do the workers in A and B.
Answer: A
4. We cannot exploit C and D unless
A. The producers in E and D are compensated for the disadvantages under which they operate.
B. We export large quantities of commodities produced by A and B
C. The prices of commodities produced by A and B are raised
D. The productivity of labour in all industries is increased
E. We allow duties to be paid to the producers in C and D rather than to the government.
Answer: A
3. Few areas of neuron behavioral research seemed more promising is the early sixties than that investigating the relationship between protein synthesis and learning. The conceptual framework for the research was derived directly from molecular biology, which had shown that genetic information is stored in nucleic acids and expressed in proteins why not acquired information as well.
The first step towards establishing a connection between protein synthesis and learning seemed to be to block memory (cause adhesion) by interrupting the production of proteins. We were fortunate in finding a non lethal dosage of puromycin that could, it first appealed, thoroughly inhibit brain protein synthesis as well as reliability produce amnesia.
Before the actual connection between protein synthesis and learning could be established however we began to have douche about whether inhibition of protein synthesis was in fact the method by which puromycin produced amnesia. First, ocher drugs, glutavimides themselves potent protein synthesis inhibitors either failed to cause amnesia in some situations where it could easily be induced by puromycin or produced an amnesia with a different time course from that of puromycin. Second, puromycin was found to inhabit protein synthesis by breaking certain amino acid chaim, and the resulting fragments were suspected of being the actual cause of amnesia is some eases. Third, puromycin was reported to cause abnormalities in the train, including seizures. Thus, not only were decreased protein synthesis and amnesia dissociated, but alternative mechanism for the amnestic action of puromycin were readily suggested.
So, puromycin turned out to be a disappointment. It came to be regarded as a poor agent for amnesia studies, although, of course, it was poor only in the context of our original paradigm of protein synthesis inhibition. In our frustration, our initial response was simply to change dregs rather than our conceptual orientation. After many such disappointments, however, it now appears unlikely, that we will make a firm connection between protein synthesis and learning merely by pursuing the approaches of the past our experience with drugs has shown that all the amnestic agents, often interfere with memory in ways that seem unrelated to their inhibition of protein synthesis. More importantly, the notion that the interruption or intensification of protein production in the train can be related in cause and affect fashion to learning non seems simplistic and unproductive. Remove the battery from a car and the car will not go Drive the car a long distance at high speed and the battery will become more highly charged. Neither of these facts proves that the battery power the car, only knowledge of the overall automotive system will reveal it mechanism of locomotion and the role of the battery with in the system.
1. The primary purpose a the passage is to show that extensive experimentation has
A. Mot supported the hypothesis that learning is directly dependent on protein synthesis
B. Cast doubt on the value of puromycin in the newer behavioral study of learning
C. Revealed the importance of amnesia in the neuron behavioral study of learning
D. Demonstrated the importance of amino acid fragmentation in the induction of amnesia.
E. Not yet demonstrated the applicability of molecular biology to behavioral research.
Answer: A
2. According to the passage, neuron behaviorists initially based their belief that protein synthesis was related to learning on which of the following?
A. Specific research into learning on which of the following
B. Traditional theories about learning
C. Historic experiments on the effects puromycin
D. Previous discoveries in molecular biology
E. Now technique in protein synthesis.
Answer: D
3. This passage was most likely excepted from
A. A book review in a leading journal devoted to genetic research.
B. A diary kept by a practicing neuron behavioral research
C. An article summarizing a series of scientific investigations in neuron behavioral research.
D. A news paper article on recent advances in the biochemistry of learning
E. A technical article on experimental techniques in the field of molecular biology.
Answer: C
4. It can be inferred from the passage that after puromycin was perceived to be a disappointment, researches did which of the following?
A. They continued to experiment with puromycin until a neuron anatomical framework was developed.
B. They continued to experiment with puromycin, but also tried other protein synthesis inhibitors
C. They ceased to experiment with puromycin and shifted to other promising protein synthesis inhibitors.
D. They ceased to experiment with puromycin and reexamined through experiments the relationship between genetic information and acquired information.
E. They continued to experiment with puromycin, but applied their results to other facts of memory research.
Answer: C
5. In the example of the car (lines 62-70) the battery is meant to represent which of the following elements in the neuron behavioral research program?
A. glutarimides
B. acquired information
C. puromycin
D. amnesia
E. protein synthesis
Answer: E
6. The passage all of the following as effects of puromycin except
A. Fragmentation of amino-acid chaim
B. Inhibition of protein synthesis
C. Brain seizures
D. Memory loss
E. Destruction of genetic information
Answer: E
7. Which of the following statements would be most likely to come after the last sentences of the passage?
A. It is important in the future, therefore for behavioral bio- chemist to focus on the several components of the total learning system.
B. The ambivalent status of current research, however should not deter neuron behaviorists from exploring the deeper connection between protein production and learning.
C. The failures of the past, however must not impede further research into the amnestic of protein-synthesis inhibitors.
D. It is important in the future, therefore, for behavioral biochemist to emphasize more strongly place of their specific findings within the overall protein synthesis model of learning.
E. It is a legacy of this research, therefore, that molecular biology's genetic models have led to disagreement among neuron behaviorists.
Answer: A
4. A clear answer to whether the languages of the ancient American peoples were made use of for expressing abstract universal concepts can be sought in the case of Nahuatl, which like Greek and German, is a language that allows the formation of extensive compounds. By combining radicals or semantic elements, single compound words can express complex conceptual relations, often of an abstract universal character.
The tlamatinime ("those who know") were able to use this rich stock of abstract terms to express the nuances of their thought. They also availed themselves of other forms of expression with metaphorical meaning, some probably original, some derived from Toltec coinages. Of these forms the most characteristic in Nahuatl is the juxtaposition of two words that, because they are synonyms, associated terms, or even contraries, complement each other to evoke one single idea. The juxtaposed terms, used as metaphor, suggest specific or essential traits of the being they refer to, introducing a mode of poetry as an almost habitual form of expression.
1. According to the passage, some abstract universal ideas can be expressed in Nahuatl by
A. Putting various meaningful elements together in one word
B. Taking away from a word any reference to particular instances
C. Turning each word of a phrase into a poetic metaphor
D. Giving a word a new and opposite meaning
E. Removing a word from its associations with other words.
Answer: A
2. It can be inferred solely from the information in the passage that
A. Metaphors are always used in Nahuatl to express abstract conceptual relationships
B. There are many languages that, like Greek or German, allow extensive compounding
C. The abstract terms of the Nahuatl language are habitually used in poetry
D. Some record or evidence of the though of the tlamatinime exists
E. All abstract universal ideas are ideas of complex relations.
Answer: D
3. A main purpose of the passage is to
A. Argue against a theory of poetic expression by citing evidence about the Nahuatl
B. Delineate the function of the tlamatinime in Nahuatl society
C. Explore the rich metaphorical heritage the Nahuatl received from the toltecs
D. Describe some conceptual and aesthetic resources of the Nahuatl language
E. Explain the abstract philosophy of the Nahuatl thinkers.
Answer: D
5. From the 197 million square miles, which make up the surface of the globe, 71 per cent is covered by the interconnecting bodies of marine water; the Pacific Ocean alone covers half the Earth and averages near 14,000 feet in depth. The portions which rise above sea level are the continents-Eurasia, Africa; North America, South America, Australia, and Antarctica. The submerged borders of the continental masses are the continental shelves, beyond which lie the deep-sea basins.
The ocean are deepest not in the center but in some elongated furrows, or long narrow troughs, called deeps. These profound troughs have a peripheral arrangement, notably around the borders of the pacific and Indian oceans. The position of the deeps, like the highest mountains, are of recent origin, since otherwise they would have been filled with waste from the lands. This is further strengthened by the observation that the deeps are quite often, where world-shaking earthquakes occur. To cite an example, the "tidal wave" that in April, 1946, caused widespread destruction along Pacific coasts resulted from a strong earthquake on the floor of the Aleutian Deep.
The topography of the ocean floors is none too well known, since in great areas the available soundings are hundreds or even thousands of miles apart. However, the floor of the Atlantic is becoming fairly well known as a result of special surveys since 1920. A broad, well-defined ridge-the Mid-Atlantic ridge-runs north and south between Africa and the two Americas and numerous other major irregularities diversify the Atlantic floor. Closely spaced soundings show that many parts of the oceanic floors are as rugged as mountainous regions of the continents. Use of the recently perfected method of submarine topography. During world war II great strides were made in mapping submarine surfaces, particularly in many parts of the vast Pacific basin.
Most of the continents stand on an average of 2870 feet above sea level. North America averages 2300 feet; Europe averages only 1150 feet; and Asia, the highest of the larger continental subdivisions, averages 3200 feet. Mount Everest, which is the highest point in the globe, is 29,000 feet above the sea; and as the greatest known depth in the sea is over 35,000 feet, the maximum relief (that is, the difference in altitude between the lowest and highest points) exceeds 64,000 feet, or exceeds 12 miles. The continental masses and the deep-sea basins are relief features of the first order; the deeps, ridges, and volcanic cones that diversify the sea floor, as well as the plains, plateaus, and mountains of the continents, are relief features of the second order. The lands are unendingly subject to a complex of activities summarized in the term erosion, which first sculptures them in great detail and then tends to reduce them ultimately to sea level. The modeling of the landscape by weather, running water, and other agents is apparent to the keenly observant eye and causes thinking people to speculate on what must be the final result of the ceaseless wearing down of the lands. Much before there was any recognizable science as geology, Shakespeare wrote "the revolution of the times makes mountains level."
1. The peripheral furrows or deeps are found
A. only in the pacific and Indian oceans
B. near earthquakes
C. near the shore
D. in the center of the ocean
E. to be 14,000 feet in depth in the pacific.
Answer: C
2. The largest ocean is the
A. Atlantic
B. pacific
C. Aleutian deep
D. arctic
E. Indian.
Answer: B
3. We may conclude from this passage that earth quakes
A. Occur more frequently in newly formed land or sea formations
B. Are caused by the weight of the water
C. Cause erosion
D. Occur in the deeps
E. Will ultimately "make mountains level".
Answer: A
4. The highest mountains are
A. oldest
B. in excess of 12 miles
C. near the deeps
D. relief features of the first order
E. of recent origin.
Answer: E
5. The science of geology was started
A. By the Greeks
B. During world war II
C. April 1946
D. After 1600
E. In 1920
Answer: D
6. The highest point on North America is
A. 2870 feet above sea level
B. not mentioned in the passage
C. higher than the highest point in Europe
D. 2300 feet above sea level
E. in Mexico.
Answer: B
7. The deeps are subject to change caused by
A. erosion
B. soundings
C. earthquakes
D. waste
E. weathering
Answer: C
8. The continental masses
A. Rise above sea level
B. Consist of six continents
C. Are relief features of the second order
D. Are partially submerged
E. Comprise 29 per cent of the earth's surface.
Answer: D
6. According to Albert Einstein the non mathematician, is seized by a mysterious shuddering when he hears of 'four-dimensional' things, he is seized by a feeling, which is very similar to the thoughts awakened by the occult. And at the same time the statement that the world in which we live is a four-dimensional space - time continuum is quite a common place statement.
This might lead to an argument regarding the use of the term ''commonplace'' by Einstein. Yet the difficulty lies more in the wording than the ideas. Einstein's concept of the universe as a four-dimensional space-time continuum becomes plain and clear, when what he means by ''continuum'' becomes clear. A continuum is something that is continuous, A ruler, for example, is a one-dimensional space continuum. Most rulers are divided into inches and fractions, scaled down to one-sixteenth of an inch.
Will it be possible to conceive a ruler, which is calibrated to a millionth or billionth of an inch. In theory there is no reason why the steps from point to point should not be even smaller. What distinguishes a continuum is the fact that the space between any two points can be sub-divided into an infinite number of smaller divisions.
A railroad track is a one-dimensional space continuum and on it the engineer of a train can describe his position at any time by citing a single co-ordinate point - i.e., a station or a milestone. A sea captain, however, has to worry about two dimensions. The surface of the sea is a two-dimensional continuum and the co-ordinate points by which sailor fixes his positions in his two dimensional continuum are latitude and longitude. An airplane pilot guides his plane through a three - dimensional continuum, hence he has to consider not only latitude and longitude, but also his height above the ground. The continuum of an airplane pilot constitutes space as we perceive it. In other words, the space of our world is a three-dimensional continuum.
Just indicating its position in space is not enough while describing any physical event, which involves motion. How position changes in time also needs to be mentioned. Thus to give an accurate picture of the operation of a New York - Chicago express, one must mention not only that it goes from New - York to Albany to Syracuse to Cleveland to Toledo to Chicago, but also the times at which it touches each of those points. This can be done either by means of a timetable or a visual chart. If the miles between New York and Chicago are plotted horizontally on a piece of ruled paper and the hours and minutes are plotted vertically, then a diagonal line properly drawn across the page illustrates the progress of the train in two - dimensional space - time continuum. This type of graphic representation is familiar to most newspaper readers; a stock market chart, for example, pictures financial events in a two - dimensional dollar - time continuum. Similarly for the best picturization of the flight of an airplane from New York to Los Angeles a four - dimensional space - time continuum is essential. The latitude, longitude and altitude will only make sense to the traffic manager of the airline if the time co - ordinate is also mentioned. Therefore time is the fourth dimension. If a flight has to be looked at, perceived as a whole, it wouldn't work if it is broken down into a series of disconnected take - offs, climbs, glides, and landing, it needs to be looked at and perceived as a continuous four - dimensional space - time continuum curve.
1. In order to explain a difficult topic, the author use
A. Simply phrased definition's
B. An incessant metaphor
C. A plain writing style
D. Familiar images
E. A quotation from Einstein
Answer: D
2. The significant feature of a continuum, according to the passage, revolves around
A. The divisibility of the interval between any two points.
B. An ordinary ruler's caliber for marking
C. Its unending curve
D. Its lucid from providing comprehensibility to the non - scientists as well
E. Its variety of co - ordinates.
Answer: A
3. The purpose of this passage is to highlight the point that
A. Plots and sea captains have something in common
B. Stock market charts may be helpful to physicists
C. The fourth dimension is time.
D. Non - mathematician's are often afraid of the commonplace
E. There is a marked quality to distance
Answer: C
4. According to the passage, an airlines traffic manager depends upon all of the following EXCEPT
A. latitude
B. altitude
C. the time co - ordinate
D. longitude
E. the continuous curve in co four
Answer: E
5. The underlying tone of this selection is
A. persuasive
B. deferential
C. candid
D. instructive
E. gently condescending
Answer: D
6. According to the author if on wishes portray a physical event in which motion plays a role - one has to
A. Make use of a time-table
B. Indicate how position changes in time
C. Be conversant with the scientist's theories
D. Describe it graphically
E. Be aware of altitude, latitude and longitude
Answer: B
7. The sea-captain's example has been cited in order to
A. Help understand a two - dimensional continuum
B. Set up a logical progression
C. Simplify what ever is too elaborate
D. Mitigate the gap between the engineer and pilot
E. To sustain out interest in the reading of the passage.
Answer: A
7. There was in increase of about 10 % in the investment in the public sector, like electricity, irrigation quarrying, public services and transport; even though the emphasis leaned towards transport and away from the other sectors mentioned. A 16-17% growth in investment, including a 30% increase in investment in business premises has been recorded in trade and services. Although there continued to be a decline in the share of agriculture in total gross investment in the economy, investment grew by 9% in absolute terms, largely spurred on by a 23% expansion of investment in agriculture equipment. Housing construction had 12% more invested in it in 1964, not so much owing to increase demand, as to fears of impending new taxes and limitation of building.
There was a rise of close to 11% in the total consumption in real terms during 1964 and per capita personal consumption by under 7%, as in 1963. The undesirable trend towards a rapid rise in consumption, evident in previous years, remains unaltered. Since at current prices consumption rose by 16% and disposable income by 13%, there was evidently a fall in the rate of saving in the private sector of the economy. Once again a swift advance in the standard of living was indicated in consumption patterns. Though fruit consumption increased, expenditure on food, especially bread and staple items, declined significantly. There was a continuing increase in the outlay on furniture and household equipment, health, education and recreation. The greatest proof of altered living standards was the rapid expansion of expenditure on transport (including private cars) and personal services of all kinds, which occurred during 1964. The changing composition if purchased durable goods demonstrated the progressive affluence of large sectors of the public. On the one hand increased purchase of automobiles and television sets were registered, a point of saturation was rapidly being approached for items like the first household radio, gas cookers, and electric refrigerators.
1. It is possible to to conclude from this passage, that the people of the country were
A. spending more money than they earn
B. investing and consuming at an accelerated pace
C. saving more money than previously
D. spending their money wisely
E. lacking in necessities
Answer: B
2. According to the author the trend towards a rapid rise in consumption is "undesirable" as:
A. there was an increase in the expenditure on frills and luxuries
B. the people were affluent
C. there was a rise in the standard of living
D. people were eating less
E. people were saving less
Answer: E
3. It is possible to conclude that the United States is not the discussed country as:
A. there was a decline in the expenditures for food
B. From the statement that the saturation point was rapidly being approached for first household radios
C. there is no mention of military expenditures
D. the people were affluent
E. the people were not saving their money
Answer: B
4. The area, which saw the greatest expenditure of investment funds was
A. The public sector
B. Business premises
C. Housing construction
D. Agricultural equipment
E. A field which cannot be determined
Answer: E
8. Furthermore, insofar as any conclusion about its author can be drawn from five or six plays attributed to him, the Wakefield Master is without exception considered to be a man of sharp contemporary observation. He was, probably clerically educated, as indicated by his Latin and music, his Biblical and patristic lore. Even today he is remembered for his his quick sympathy for the oppressed and forgotten man, his sharp eye for character, a ready ear for colloquial, vernacular turns of speech and a humor alternately rude and boisterous, coarse and happy. Therefore in spite of his conscious artistry as can be seen in his feeling for intricate metrical and stanza forms, he is regarded as a kind of medieval Steinbeck, indignantly angry at, uncompromisingly and even brutally realistic in presenting the plight of the agricultural poor.
It is now fairly accepted to regard the play as a kind of ultimate point in the secularization of the medieval drama. Therefore more stress has been laid on it as depicting realistically humble manners and pastoral life in the bleak of the west riding of Yorkshire on a typically cold night of December 24th. After what are often regarded as almost ''documentaries'' given in the three successive monologues of the three shepherds, critics go on to affirm that the realism is then intensified into a burlesque mock-treatment of the Nativity. Finally as a sort of epilogue or after-thought in deference to the Biblical origins of the materials, the play slides back into an atavistic mood of early innocent reverence. In actuality, the final scene is the culminating scene and also the raison d'etre of the introductory ''realism.''
Superficially the present play supports the conventional view of its mood of secular realism. At the same time, the ''realism'' of the Wakefield Master is of a paradoxical turn. His wide knowledge of people, as well as books indicates no cloistered contemplative but one in close relation to his times. Still, that life was after all a predominantly religious one, a time which never neglected the belief that man was a rebellious and sinful creature in need of redemption . So deeply (one can hardly say ''naively'' of so sophisticated a writer) and implicitly religious is the Master that he is less able (or less willing) to present actual history realistically than is the author of the Brome Abraham and Isaac. His historical sense is even less realistic than that of Chaucer who just a few years before had done for his own time ''costume romances,'' such as The Knight's Tele, Troilus and Cressida, etc. Furthermore, used highly romantic materials, which could excuse his taking liberties with history.
1. Of the following statements, which is not true of Wakefield Master?
A. He and Chaucer were contemporaries.
B. Wakefield Master is remembered as having written five or six realistic plays.
C. His plays realistically portray the plight of the country folk of his day
D. His writing was similar to that of John Steinbeck.
E. He was an accomplished artist.
Answer: D
2. The word 'patristic' in the first paragraph is used to mean:
A. patriotic
B. superstitious
C. folk
D. relating to the Christian Fathers
E. realistic
Answer: D
3. The statement about the ''secularization of the medieval drama'' (opening sentence of the second paragraph) refers to the
A. Introduction of religious themes in the early days
B. Presentation of erudite material
C. Use of contemporary materials
D. Return to early innocent reverence at the end of the play
E. Introduction of mundane matters in religious plays
Answer: E
4. From the following what would the writer be expected to do in the subsequent paragraphs:
A. Make a justification for his comparison with Steinbeck
B. Put forth a view point, which would take up the thought of the second paragraph
C. Point out the anachronisms in the play
D. Discuss the works of Chaucer
E. Talk about the lack of realism in the works of the Wakefield Master.
Answer: B
9. The establishment of the third Reich influenced events in American history by starting a chain of events which culminated in war between Germany and the United States. The complete destruction of democracy, the persecution of laws, the war on religion, the cruelty and barrbarism of the Nazis and especially, the plans of Germany and her allies, Italy and Japan, for world conquest caused great indignation in this country and brought on fear of another world war. While speaking out against Hitler's atrocities, the American profile generally favored isolationist policies, and neutrality. The neutrality acts of 1935 and 1936 prohibited trade with any belligerents or loans to them. In 1937 the president was empowered to declare an arms embargo in wars between nations at his discretion
American opinion began to change somewhat after President Roosevelt's quarantine the aggvessor speech at Chicago (1937) in which he severely criticized Hitler's policies. Germany's seizure of Austria and Munich pact for the partition of Czechoslovakia (1938) also around the American people. The conquest of Czechoslovakia in March 1939 was another rude awakening to the menace of the third Reich. In August, 1939, came the shock of the Nazi - Soviet pact and in September the attack on Poland and the outbreak of European war. The United States attempt to maintain neutrality in spite of sympathy for the democracies arranged against the Third Reich. The Neutrality act of 1939 repeated the arms embargo and permitted 'cash' and 'carry' exports of arms to belligerent nations. A strong national defense program was begun. A draft act was passed (1940) to strengthen the military services. A Lend - Lease Act (1940) authorized the president to sell, exchange or lend materials to any county deemed necessary by him for the defense of the United States. Help was given to Britain territory in the western Hemisphere. In August 1941, President Roosevelt and prime minister Churchill met and issued the Atlantic Charter which proclaimed the kind of a world which should be established after the war. In December 1941, Japan launched the unprovoked attack on the United States at Pearl harbor, immediately thereafter Germany declared war on the united states.
1. USA entered the war against Germany
A. because Pearl Harbor was attacked
B. after peaceful efforts had failed
C. because Germany declare war against it
D. because Japan was an ally of Germany
E. after Germany had signed the Nazi - Soviet pact
Answer: C
2. The Neutrality Act of 1939 favored Great Britain because
A. the British had command of the sea
B. the law permitted U.S.A. to trade only with the allies.
C. it antagonized Japan
D. it led to the Land - Lease Act
E. it agreed with the British on the principle of the Atlantic Charter
Answer: A
3. An event that did not occur in 1939 was the
A. invasion of Poland
B. invasion of Czechoslovakia
C. passing of the Neutrality Act
D. passing of the Land - Lease act
E. outbreak of the war in Europe
Answer: D
4. One item occurring 1937 that the author does not mention in the list of actions that alienated the American Public was
A. The persecution of religious groups
B. Nazi barbarism
C. The pacts with Italy
D. German plans for conquest of the world
E. The burning of the Reich tag.
Answer: E
5. The Land - Lease Act has designed to
A. Strengthen USA's national defense
B. Provide battle shit to the Allies
C. Help the British
D. the Atlantic Charter
E. Avenge Pearl Harbor
Answer: A
6. The Neutrality Act of 1939
A. restated America's isolationist policies
B. proclaimed American neutrality
C. permitted the selling of arms to belligerent nation
D. was cause of USA's entrances in to WORLD WAR II
E. started USA's national defense programs
Answer: C
7. During the years 1933-36, American policy may be described as having been
1. watchful
2. isolationist
3. pacific
4. incorrect
5. discretionary
Answer: B
10. Certain scraps of evidence bear out those who hold a very high opinion of the average level of culture among the Athenians of the great age. Pericles's funeral speech is undoubtedly the most famous evidence from Athenian literature, that its level was indeed high. However, Pericles was a politician, and it is possible that he was flattering his audience. We know that thousands of Athenians sat hour after hour in the theater listening to the plays of the great Greek dramatists. The Greek plays, particularly the tragedies, maintained an extremely high intellectual level throughout, with no letdowns, no concessions to the lowbrows or to the demands of ''realism'', like the gravediggers scene in Shakespeare's Hamlet. The music and dancing seen in these plays were also of an equally high level. The best modern parallel can be seen in the restrained, difficult opera of the 18th century. The comparison is no doubt dangerous, but can you imagine almost the entire population of an American city (in suitable installments, of course) sitting through performances of Mozart's Don Giovanni or Gluck's Orpheus? Perhaps the Athenian masses went to these plays because of a lack of other amusements. They could at least understand something of what went on, since the subjects were part of their folklore. Undoubtedly the theme of grand opera is not part of the folklore of the American people.
1. From the passage it is evident that the author seems to question the sincerity of
A. politicians
B. playwrights
C. opera goers
D. ''low brows''
E. gravediggers.
Answer: A
2. According to the author the average American
A. Enjoys Hamlet
B. Loves folklore
C. Is not able to understand grand opera
D. Seeks a high cultural level
E. Lacks entertainment.
Answer: C
3. From the passage, we can say that the author's attitude toward Greek plays is one of
A. Qualified approval
B. Grudging admiration
C. Studied indifference
D. Partial hostility
E. Great respect.
Answer: E
4. The author makes a suggestion that Greek plays
A. Were demanding on the actors
B. Flattered their audiences
C. Were focussed on a limited audience
D. Were dominated by music and dancing
E. Stimulated their audiences.
Answer: E
11. Everyone conforms to infancy, infancy conforms to nobody, so that one babe commonly makes four or five out of the adults who prattle and play to it. So God has armed youth and puberty and manhood no less with its own piquancy and charm, and made it enviable and gracious and its claims not to be put by, if it will stand by itself. Do not think the youth has no force, because he cannot speak to you and me. Hark! In the next room his voice is sufficiently clear and emphatic. It seems he knows how to speak to his contemporaries. Bashful or bold, then, he will know how to make us seniors very unnecessary.
The healthy attitude of human nature can be seen in the nonchalance of boys who are sure of a dinner, and would disdain as much as a lord to do or say aught to conciliate one. A boy is in the parlor what the pit is in the playhouse; independent, irresponsible, looking out from his corner on such people and facts as pass by, he tries and sentences them on their merits, in the swift, summary way of boys, as good, bad, interesting, silly, eloquent, troublesome. He never cumbers himself regarding consequences, about interests and he gives an independent, genuine verdict. You should court him: he will not court you. But the man is, as it were, clapped into jail by his consciousness. As soon as he has once acted or spoken with eclat, he is a committed person, watched by the sympathy or the hatred of hundreds, whose affections must now enter into his account. There is no Lethe for this. Ah, that he could pass again into his neutrality.
These are the voices, which we hear in solitude, but they grow faint and inaudible as we enter into the world. Everywhere society is conspiring against the manhood of every one of its members. Society is joint - stock company, in which members agree, for the better securing of his bread to each shareholder, to surrender the liberty and culture of the eater. The virtue in most request is conformity. It is averse to self-reliance. What it loves is names and customs and not realities and creators.
Whosoever is a man has to be a nonconformist. He who would gather immortal palms must not be hindered by the name of goodness, but must explore if it be goodness. Nothing is at last sacred but the integrity of your own mind.
No law can be sacred to me but that of my nature. Good and bad are but names very readily transferable to that to this; the only right is what is after my constitution, the only right is what is after me constitution, the only wrong what is against it. A man is to carry himself in the presence of all opposition as if every thing were titular and ephemeral but he. I am ashamed to think how easily we capitulate to badges and names, to large societies and dead institutions. Every decent and well-spoken individual affects and sways me more than is right. I ought to go upright and vital, and speak the rude truth in all ways.
I shun father and mother and wife and brother, when my genius calls me. I would write on the lintels of the doorpost, whim. I hope it is somewhat better than whim at last, but we cannot spend the day in explanation. Except me not to show cause why I seek or why I exclude company. Then, again, do not tell me, as a good man did not to-day, of my obligation to put all poor men in good situations. Are they my poor? I tell thee, thou foolish philanthropist, that I grudge the dollar, the time, the cent, I give to such men as do not belong to me and to whom I do not belong. There is a class of person to whom by all spiritual affinity I am bought and sold; for them I will go to prison, if need be; but your miscellaneous popular charities; the education at collage of fools; the building of meeting - house to the vain end to which many now stand; alms to sots; and the thousandfold Relief Societies; - though I confess with shame I sometimes succumb and give the dollar, it is a wicked dollar which by and by I shall have the manhood to withhold.
If you refuse to conform, you can experience the displeasure of the world. Hence, a man should know how to estimate a sour face. The by - standers look askance on him in the public street or in the friend's parlor. In case this aversion originates from contempt and resistance similar to his own, it might result in a sad countenance; but the sour faces of the multitude, like their sweet faces, have no deep cause, but are caused by reasons as diverse as the direction of the wind and what he reads in the newspapers. Yet is the discontent of the multitude more formidable than that of the senate and the collage.
Another factor, which frightens us from self - trust in our consistency; a reverence for our past act or word, because the eyes of others have no other data for computing our orbit than our past acts, and we are loath to disappoint them.
But why should you keep your head over your shoulder? Why drag about this corpse of your memory, lest you contradict somewhat you have stated in this or that public place? Suppose you should contradict yourself; what then?
This is a rather silly consistency in our minds, which is adored by little statesmen and philosophers and divines. Uniformly a great soul has almost nothing to do, he could just occupy himself with his shadow on the wall. Speak what you think now in hard words; and to-morrow speak what tomorrow thinks in hard words again, though it contradict everything you said to-day. - ''Ah, so you shall be sure to be misunderstood.'' - Is it so bad, then, to be misunderstood? Pythagoras was misunderstood, and Socrates, and Jesus, and Luther, and Copernicus, and Galileo, and Newton, and every pure and wise spirit that ever took flesh. What can be considered to be truly great is to be misunderstood.
1. Which of the following statements would best describe the main theme of the above passage?
A. "A foolish consistency is the hobgoblin of little mind."
B. "Eternal youth means eternal independence."
C. "Whoso would be a man must be a nonconformist."
D. "Colleges are designed to educate fools."
E. "Infancy conforms to nobody."
Answer: C
2. When is the period during which we are most nonconformist?
A. infancy
B. puberty
C. youth
D. manhood
E. old age
Answer: A
3. In his statement ''What can be considered to be truly great is to be misunderstood'' the author means:
A. One should refrain from saying, what one exactly means
B. Being misunderstood, equals being great
C. All great man have always been misunderstood
D. Even though a person might be considered inconsistent, he shouldn't hesitate to change his mind if he feels the need to.
E. It is seldom, that nice people succeed
Answer: D
4. As inferred from the passage, the refusal of young people to cater to accept public opinion is:
A. A feature of the rebelliousness of youth
B. A healthy attitude of human nature
C. A manifestation of deep- seated immaturity
D. Simply bad manners
E. Part of growing up
Answer: B
5. "Society is a joint-stock company etc." is one way which the author shows
A. The anti-culture attitude of the public
B. Society is highly organized and structured
C. The self-rejection of society
D. The lack of room for solitude in our world
E. The public's interest in the stock market
Answer: C
6. " I would write on the lintels of the doorpost, whim." What does the author mean by this statement:
A. That one should renounce his immediate family
B. That signposts have an important educational function in our society
C. That an impulsive action may have a subsequent rational explanation
D. That one must never be held responsible for what one says and does
E. That everyone should do foolish things occasionally
Answer: C
7. Which of the following statements best summarizes the spirit and sense of the above passage?
A. "Nothing is at last sacred but the integrity of your own mind."
B. "With consistency, a great soul; has simply nothing to do."
C. "Do not think the youth has no force, because cannot speak to you and me."
D. "The virtue in most request is conformity."
E. "A man must know how to estimate a sour force."
Answer: A
12. Visual recognition involves storing and retrieving memories. Neural activity, triggered by the eye, forms an image in the brains memory system that constitutes an internal representation of the viewed object. When an object is encountered again, it is matched with its internal representation and thereby recognized. Controversy surrounds the question of whether recognition is a parallel, one-step process or a serial, step-by-step one. Psychologists of the Gestalt school maintain that object are recognized as wholes in a parallel procedure : , the internal representation is matched with the retinal image in a single operation. Other psychologists have proposed that internal representation features are matched serially with an object's features. Although some experiments show that, as an object become familiar, its internal representation becomes more familiar, its internal representation becomes more holistic and the recognition process correspondingly more parallel, the weight of evidence seems to support the serial hypothesis, at least for objects that are not notably simple and familiar.
1. It can be inferred from the passage that the matching process in visual recognition is
A. Not a natural activity.
B. Not possible when an object is viewed for the very first time.
C. Not possible if a feature of a familiar object is changed in same way.
D. Only possible when a retinal image is received in the brain as a unitary whole.
E. Now fully understood as a combination of the serial and parallel process.
Answer: A
2. In terms of its tone and form, the passage can best be characterized as
A. A biased exposition
B. A speculative study
C. A dispassionate presentation
D. An indignant denial
E. A dogmatic explanation.
Answer: C
3. The author is primarily concerned with
A. Explaining how the brain receives images
B. Synthesizing hypotheses of visual recognition
C. Examining the evidence supporting the serial recognition hypothesis
D. Discussing visual recognition and some hypotheses proposed to explain it.
E. Reporting on recent experiments dealing with memory systems and their relationship to neural activity.
Answer: B
4. According to the passage, Gestalt psychologists make which of the following suppositions about visual recognition?
I A retinal image is in exactly the same form as its internal representation
II An object is recognized as a whole without any need for analysis into component parts.
III The matching of an object with its internal representation occurs in only one step
A. II only
B. III only
C. I and III only
D. II and III only
E. I, II and III
Answer: D
13. For a period of more than two centuries paleontologists have been intrigued by the fossilized remains of pterosaurs, the first flying vertebartes. The issues, which puzzle them, are how these heavy creatures, having a wingspan of about 8-12 meters managed the various problems associated with powered flight and whether these creatures were reptiles or birds.
Perhaps the least controversial assertion about the pterosaurs is that they were reptiles. Their skulls, pelvises, and hind feet are reptilian. The anatomy of their wings suggests that they did not evolve into the class of birds. In pterosaurs a greatly elongated fourth finger of each forelimb supported a winglike membrane. The other fingers were short and reptilian, with sharp claws. In birds the second finger is the principal strut of the wing, which consists primarily of feathers. If the pterosaurs walked on all fours, the three short fingers may have been employed for grasping. When a pterosaurs walked or remained stationary, the fourth finger, and with it the wing, could only urn upward in an extended inverted V- shape along each side of the animal's body.
In resemblance they were extremely similar to both birds and bats, with regard to their overall body structure and proportion. This is hardly surprising as the design of any flying vertebrate is subject to aerodynamic constraints. Both the pterosaurs and the birds have hollow bones, a feature that represents a savings in weight. There is a difference, which is that the bones of the birds are more massively reinforced by internal struts.
Although scales typically cover reptiles, the pterosaurs probably had hairy coats. T.H. Huxley reasoned that flying vertebrates must have been warm-blooded because flying implies a high rate of metabolism, which in turn implies a high internal temperature. Huxley speculated that a coat of hair would insulate against loss of body heat and might streamline the body to reduce drag in flight. The recent discovery of a pterosaur specimen covered in long, dense, and relatively thick hair like fossil material was the first clear evidence that his reasoning was correct.
Some paleontologists are of the opinion that the pterosaurs jumped from s dropped from trees or perhaps rose into the light winds from the crests of waves in order to become airborne. Each theory has its associated difficulties. The first makes a wrong assumption that the pterosaurs hind feet resembled a bat's and could serve as hooks by which the animal could hang in preparation for flight. The second hypothesis seems unlikely because large pterosaurs could not have landed in trees without damaging their wings. The third calls for high aces to channel updrafts. The pterosaurs would have been unable to control their flight once airborne as the wind from which such waves arose would have been too strong.
1. As seen in the above passage scientists generally agree that:
A. the pterosaurs could fly over large distances because of their large wingspan.
B. a close evolutionary relationship can be seen between the pterosaurs and bats, when the structure of their skeletons is studied.
C. the study of the fossilized remains of the pterosaurs reveals how they solved the problem associated with powered flight
D. the pterosaurs were reptiles
E. Pterosaurs walked on all fours.
Answer: D
2. The view that, the pterosaurs rose into light winds from the crest of the waves to become airborne, is viewed by the author as
A. revolutionary
B. unlikely
C. unassailable
D. probable
E. outdated.
Answer: B
3. As inferred from the passage, the skeleton of a pterosaur is distinguishable from that of a bird by the
A. length of its wingspan
B. hollow spaces in its bones
C. anatomic origin of its wing strut
D. evidence of the hooklike projections on its hind feet
E. location of the shoulder joint joining the wing to its body.
Answer: C
4. From the viewpoint of T.H.Huxley, as given in the passage, which of the following statements is he most likely to agree with?
A. An animal can master complex behaviors irrespective of the size of it's brain.
B. Environmental capabilities and physical capabilities often influence the appearance of an animal.
C. Usually animals in a particular family group do not change their appearance dramatically over a period of time
D. The origin of flight in vertebrates was an accidental development rather than the outcome of specialization or adaption
E. The pterosaurs should be classified as birds, not reptiles.
Answer: B
5. According to the passage which of the following is a characteristic of the pterosaurs?
A. The pterosaurs were not able to fold their wings when not in use
B. Like the bats, they hung upside down from branches
C. They flew in order to capture prey
D. They can be said to be an earlier stage in the evolution of the birds
E. They lived principally in a forest like habitat.
Answer: A
6. The organization of the last paragraph of the passage can best be described as:
A. New data is introduced in order to support a traditional point of view
B. Three explanations are put forth and each of them is disputed by means of specific information
C. An outline of three hypotheses are given and evidence supporting each of them is given
D. Description of three recent discoveries is presented, and their implications for future study are projected
E. The material in the earlier paragraphs is summarized and certain conclusions are from it.
Answer: B
7. According to the passage, some scientists believe that pterosaurs
A. Lived near large bodies of water
B. Had sharp teeth for tearing food
C. Were attacked and eaten by larger reptiles
D. Had longer tails than many birds
E. Consumed twice their weight daily to maintain their body temperature.
Answer: A
14. The existence of mammals on the earth can be traced back to at least the Triassic time. The rate of development was retarded, till evolutional change suddenly accelerated in the oldest Paleocene. This resulted in an increase in average size, larger mental capacity, and special adaptations for different modes of life, during the Eocene time. Further improvement was seen during the Oligocene Epoch, with the appearance of some new lines and extinction of others. The Miocene and Pliocene times are especially significant as they mark the culmination of various groups and a continued approach toward modern characters. It is in the Miocene time that the mammals reached their peak with reference to variety and size.
The ability of the mammals to adapt to various modes of life finds a parallel in the reptiles of the Mesozoic time, and apart form their greater intelligence, the mammals apparently have not done much better than the corresponding reptilian forms. Undoubtedly the bat is a better flying animal than the pterosaur, but at the same time the dolphin and whale are hardly more fish like than the ichthyosaur. Quite a few of the swift-running mammals inhabiting the plains, like the horse and the antelope, must excel any of the dinosaurs. Although the tyrannosaur was a more weighty and robust carnivore than perhaps any carnivorous mammal, the lion and the tiger, by virtue of their superior brain are far more efficient and dangerous beasts of prey. It is significant to note that various species of mammals gradually adapted themselves to various kinds of lifestyles, some took to grazing on the plains and were able to run swiftly (horse, deer, bison), others started living in rivers and swamps (hippopotamus, beaver), inhabiting trees (sloth, monkey), burrowing underground (rodent, mole), feeding on flesh (tiger, wolf), swimming in the water (dolphin, whale, seal), and flying in the air (bat). Human beings on account of their superior brain have been able to harness mechanical methods to conquer the physical world and adapt to any set of conditions.
Such adaptation to different conditions leads to a gradual change in form and structure. This is a biological characteristic of the youthful, plastic stage of a group. It is seen that early in its evolutional cycle animals possess the capacity for change, but as the animal progresses in its cycle becoming old and fixed, this capacity for change disappears. The generalized types of organisms retain longest the ability to make adjustments when required, and it is from them that new, fecund stocks take origin-certainly not from any specialized end products. With reference to mammals, we see their birth, plastic spread in many directions, increased specialization, and in some cases, extinction; this is a characteristic of the evolution of life, which can be seen in the geologic record of life.
1. From the following, choose the most appropriate title for the above passage?
A. From Dinosaur to Man
B. Adaptation and Extinction
C. The Superior Mammals
D. The Geologic Life Span
E. Man, the Vanquisher of the Physical World.
Answer: B
2. According to the passage the chronological order of the geologic periods is:
A. Paleocene, Miocene, Triassic, Mesozoic
B. Paleocene, Triassic, Mesozoic, Miocene
C. Miocene, Paleocene, Triassic, Mesozoic
D. Mesozoic, Oligocene, Paleocene, Miocene
E. Mesozoic, Paleocene, Eocene, Miocene
Answer: E
3. From the above passage, we can infer that, the pterosaur
A. resembled the bat
B. was a Mesozoic mammal
C. was a flying reptile
D. inhabited the seas
E. evolved during the Miocene period
Answer: C
4. As inferred from the passage, the largest number of mammals were found in which of the following periods?
A. Triassic period
B. Eocene period
C. Oligocene epoch
D. Pliocene period
E. Miocene period
Answer: E
5. Among the following statements, which statement, if true, would weaken the argument put forth in the first sentence of Paragraph 1?
A. It has been found that the tryannosaur had a larger brain, than was previously known.
B. Within the next thousand years, mammals will become extinct.
C. Recently certain forms of flying ichthyosaurs have been discovered.
D. It has now been proved, that the tiger is more powerful than the carnivorous reptiles.
E. It is now possible to double human mental capacity, by the use of certain recently developed computers.
Answer: A
6. It is clear from the passage, that the evidence used to discuss the life of past time periods
A. was developed by Charles Darwin
B. was unearthed by the author
C. has been negated by more recent evidence
D. was never truly established
E. is based on fossilized remains
Answer: E
7. As inferred from the passage, which of the following proverbial expressions is the author most likely to agree with?
A. It's a cruel world.
B. All the world's a stage.
C. The more things change, the more they remain the same.
D. Footprints in the sands of time.
E. A short life, but a merry one.
Answer: D
15. Some modern anthropologists hold that biological evolution has shaped not only human morphology but also human behavior. The role those anthropologists ascribe to evolution is not of dictating the details of human behavior but one of imposing constraints - ways of feeling, thinking, and acting that ''come naturally'' in archetypal situations in any culture. Our ''frailties'' - emotions and motivs such as rage, fear, greed, gluttony, joy,lust, love-may be a very mixed assortment quality: we are, as we say, ''in the grip'' of them. And thus they give us oursense of constraints.
Unhappily, some of those frailties our need for ever-increasing security among them are presently maladaptive. Yet beneath the overlay of cultural detail, they, too, are said to be biological in direction, and therefore as natural to us as are our appendixes. We would need to comprehend throughly their adaptive origins in order to understand how badly they guide us now. And we might then begin to resist their pressure.
1. The author implies that control to any extent over the ''frailties'' that constrain our behavior is though to presuppose
A. That those frailties and adaptive are recognized as currently beneficial and adaptive
B. That there is little or no overlay of cultural detail that masks their true nature.
C. That there are cultures in which those frailties do not ''come naturally'' and from which such control can be learned
D. A full understanding of why those frailties evolved and of how they function now
E. A thorough grasp of the principle that cultural detail in human behavior can differ arbitrarily from society to society.
Answer: D
2. It can be inferred that in his discussion of maladaptive frailties the author assumes that
A. Evolution does not favor the emergence of adaptive characteristics over the emergence of maladaptive ones
B. Any structure or behavior not positively adaptive is regarded as transitory in evolutionary theory
C. Maladaptive characteristics, once fixed, make the emergence of other maladaptive characteristics more likely
D. The designation of a characteristic as being maladaptive must always remain highly tentative
E. Changes in the total human environment can outpace evolutionary change.
Answer: E
3. The primary purpose of the passage is to present
A. A position on the foundations of human behavior and on what those foundations imply
B. A theory outlining the parallel development of human morphology and of human behavior
C. A diagnostic test for separating biologically determined behavior patters from culture - specific detail
D. An overview of those human emotions and motive's that impose constraints on human behaviour
E. A practical method for resting the pressures of biologically determined drives.
Answer: A
4. Which of the following most probably provides an appropriate analogy from human morphology for the ''details'' versus ''constraints'' distinction made in the passage in relation to human behaviour?
A. The ability of most people to see all the colors of the visible spectrum as against most peoples inability to name any but the primary colors
B. The ability of even the least fortunate people to show compassion as against people's inability to mask their feelings completely
C. The ability of some people to dive to great depths as against most people's inability to swim long distance
D. The psychological profile of those people who are able to delay gratification as against people's inability to control their lives completely
E. The greater lung capacity of mountain peoples that helps them live in oxygen-poor air as against people's inability to fly without special apparatus.
Answer: E
16. Roger Rosenblatt's book Black Fiction, manages to alter the approach taken in many previous studies by making an attempt to apply literary rather than sociopolitical criteria to its subject. Rosenblatt points out that criticism of Black writing has very often served as a pretext for an expounding on Black history. The recent work of Addison Gayle's passes a judgement on the value of Black fiction by clearly political standards, rating each work according to the ideas of Black identity, which it propounds.
Though fiction results from political circumstances, its author react not in ideological ways to those circumstances, and talking about novels and stories primarily as instruments of ideology circumvents much of the fictional enterprise. Affinities and connections are revealed in the works of Black fiction in Rosenblatt's literary analysis; these affinities and connections have been overlooked and ignored by solely political studies.
The writing of acceptable criticism of Black fiction, however, presumes giving satisfactory answers to a quite a few questions. The most important of all, is there a sufficient reason, apart from the racial identity of the authors, for the grouping together of Black authors? Secondly, what is the distinction of Black fiction from other modern fiction with which it is largely contemporaneous? In the work Rosenblatt demonstrates that Black fiction is a distinct body of writing, which has an identifiable, coherent literary tradition. He highlights recurring concerns and designs, which are independent of chronology in Black fiction written over the past eighty years. These concerns and designs are thematic, and they come form the central fact of the predominant white culture, where the Black characters in the novel are situated irrespective of whether they attempt to conform to that culture or they rebel against it.
Rosenblatt's work does leave certain aesthetic questions open. His thematic analysis allows considerable objectivity; he even clearly states that he does not intend to judge the merit of the various works yet his reluctance seems misplaced, especially since an attempt to appraise might have led to interesting results. For example, certain novels have an appearance of structural diffusion. Is this a defeat, or are the authors working out of, or attempting to forge, a different kind of aesthetic? Apart from this, the style of certain Black novels, like Jean Toomer's Cane, verges on expressionism or surrealism; does this technique provide a counterpoint to the prevalent theme that portrays the fate against which Black heroes are pitted, a theme usually conveyed by more naturalistic modes of expressions?
Irrespective of such omissions, what Rosenblatt talks about in his work makes for an astute and worthwhile study. His book very effectively surveys a variety of novels, highlighting certain fascinating and little-known works like James Weldon Johnson's Autobiography of an Ex-Coloured Man. Black Fiction is tightly constructed, and levelheaded and penetrating criticism is exemplified in its forthright and lucid style.
1. The author of the passage raises and objection to criticism of Black fiction like that by Addison Gayle as it:
A. Highlights only the purely literary aspects of such works
B. Misconceive the ideological content of such fiction
C. Miscalculate the notions of Black identity presented in such fiction
D. Replaces political for literary criteria in evaluating such fiction
E. Disregards the reciprocation between Black history and Black identity exhibited in such fiction.
Answer: D
2. The primary concern of the author in the above passage is:
A. Reviewing the validity of a work of criticism
B. Comparing various critical approaches to a subject
C. Talking of the limitations of a particular kind of criticism
D. Recapitulation of the major points in a work of criticism
E. Illustrating the theoretical background of a certain kind of criticism.
Answer: A
3. The author is of the opinion that Black Fiction would have been improved had Rosenblatt:
A. Undertaken a more careful evaluation of the ideological and historical aspects of Black Fiction
B. Been more objective in his approach to novels and stories by Black authors
C. Attempted a more detailed exploration of the recurring themes in Black fiction throughout its history
D. Established a basis for placing Black fiction within its own unique literary tradition
E. Calculated the relative literary merit of the novels he analyzed thematically.
Answer: E
4. Rosenblatt's discussion of Black Fiction is :
A. Pedantic and contentious
B. Critical but admiring
C. Ironic and deprecating
D. Argumentative but unfocused
E. Stilted and insincere.
Answer: B
5. According to the given passage the author would be LEAST likely to approve of which among the following?
A. Analyzing the influence of political events on the personal ideology of Black writers
B. Attempting a critical study, which applies sociopolitical criteria to the autobiographies of Black authors
C. A literary study of Black poetry that appraises the merits of poems according to the political acceptability of their themes
D. Studying the growth of a distinct Black literary tradition within the context of Black history
E. Undertaking a literary study, which attempts to isolate aesthetic qualities unique to Black fiction.
Answer: C
6. From the following options, which does the author not make use of while discussing Black Fiction?
A. Rhetorical questions
B. Specific examples
C. Comparison and contrast
D. Definition of terms
E. Personal opinion.
Answer: D
7. The author makes a reference to James Weldon Johnson's Autobiography of an Ex-colored Man most probably to:
A. Highlight the affinities between Rosenblatt's method of thematic analysis and earlier criticism
B. Elucidate regarding the point made regarding expressionistic style earlier in the passage
C. Qualify the assessment of Rosenblatt's book made in the first paragraph of the passage
D. Demonstrate the affinities among the various Black novels talked of by Rosenblatt's literary analysis
E. Present a specific example of one of the accomplishments of Rosenblatt's work.
Answer: E
17. The Food and Drug Administration has formulated certain severe restrictions regarding the use of antibiotics, which are used to promote the health and growth of meat animals. Though the different types of medicines mixed with the fodder of the animals kills many microorganisms, it also encourages the appearance of bacterial strains, which are resistant to anti-infective drugs.
It has already been observed that penicillin and the tetracyclines are not as effective therapeutically as they once used to be. This resistance to drugs is chiefly caused due to tiny circlets of genes, called plasmids, which are transferable between different species of bacteria. These plasmids are also one of the two kinds of vehicles on which molecular biologists depend on while performing gene transplant experiments. Existing guidelines also forbid the use of plasmids, which bear genes for resistance to antibiotics, in the laboratories. Though congressional dabate goes on as to whether these restrictions need to be toughened with reference to scientists in their laboratories, almost no congressional attention is being paid to an ill advised agricultural practice, which produces deleterious effects.
1. In the present passage, the author's primary concern is with:
A. The discovery of methods, which eliminate harmful microorganisms without generating drug-resistant bacteria.
B. Attempting an explanation of the reasons for congressional inaction about the regulation of gene transplant experiments.
C. Portraying a problematic agricultural practice and its serious genetic consequences
D. The verification of the therapeutic ineffectiveness of anti-infective drugs
E. Evaluation of the recently proposed restrictions, which are intended to promote the growth of meat animals.
Answer: C
2. As inferred from the above passage, the mutual transfer of plasmids between different bacteria can result in which of the following?
A. Microorganisms, which have an in-built resistance to drugs
B. Therapeutically useful circlets of genes
C. Penicillin like anti-infective drugs
D. Viruses used by molecular biologists
E. Carriers for performing gene transplant experiments.
Answer: A
3. According to the above passage the author believes that those who favor the stiffening of restrictions on gene transplant research should logically also.
A. Approve and aid experiments with any plasmids except those, which bear genes for antibiotic resistance.
B. Inquire regarding the addition of anti-infective drugs to livestock feeds
C. Oppose the using of penicillin and tetracyclines in order to kill microorganisms
D. Agree to the development of meatier live-stock through the use of antibiotics
E. Approve of congressional debate and discussion regarding science and health issues.
Answer: B
4. The attitude the author has with reference to the development of bacterial strains that render antibiotic drugs in effective can best be described as
A. indifferent
B. perplexed
C. pretentious
D. insincere
E. apprehensive
Answer: E
18. Disequilibrium at the interface of water and air is a factor on which the transfer of heat and water vapor from the ocean to the air depends. The air within about a millimeter of the water is almost saturated with water vapor and the temperature of the air is close to that of the surface water. Irrespective of how small these differences might be, they are crucial, and the disequilibrium is maintained by air near the surface mixing with air higher up, which is typically appreciably cooler and lower in water vapor content. The turbulence, which takes its energy from the wind mixes the air. As the speed of wind increases, so does the turbulence, and consequently the rate of heat and moisture transfer. We can arrive at a detailed understanding of this phenomenon after further study. The transfer of momentum from wind to water, which occurs when waves are formed is an interacting-and complicated phenomenon. When waves are made by the wind, it transfers important amounts of energy-energy, which is consequently not available for the production of turbulence.
1. This passage principally intends to:
A. resolve a controversy
B. attempt a description of a phenomenon
C. sketch a theory
D. reinforce certain research findings
E. tabulate various observations
Answer: B
2. The wind over the ocean usually does which of the following according to the given passage?
I. Leads to cool, dry air coming in proximity with the ocean surface.
II. Maintains a steady rate of heat and moisture transfer between the ocean and the air.
III. Results in frequent changes in the ocean surface temperature.
A. I only
B. II only
C. I and II only
D. II and III only
E. I, II, and III
Answer: A
3. According to the author the present knowledge regarding heat and moisture transfer from the ocean to air as
A. revolutionary
B. inconsequential
C. outdated
D. derivative
E. incomplete
Answer: E
4. According to the given passage, in case the wind was to decrease until there was no wind at all, which of the following would occur?
A. The air, which is closest to the ocean surface would get saturated with water vapor.
B. The water would be cooler than the air closest to the ocean surface.
C. There would be a decrease in the amount of moisture in the air closest to the ocean surface.
D. There would be an increase in the rate of heat and moisture transfer.
E. The temperature of the air closest to the ocean and that of the air higher up would be the same.
Answer: A
19. But man is not destined to vanish. He can be killed, but he cannot be destroyed, because his soul is deathless and his spirit is irrepressible. Therefore, though the situation seems dark in the context of the confrontation between the superpowers, the silver lining is provided by amazing phenomenon that the very nations which have spent incalculable resources and energy for the production of deadly weapons are desperately trying to find out how they might never be used. They threaten each other, intimidate each other and go to the brink, but before the total hour arrives they withdraw from the brink.
1. The main point from the author's view is that
A. Man's soul and spirit can not be destroyed by superpowers.
B. Man's destiny is not fully clear or visible.
C. Man's soul and spirit are immortal.
D. Man's safety is assured by the delicate balance of power in terms of nuclear weapons.
E. Human society will survive despite the serious threat of total annihilation.
Answer: E
2. The phrase 'Go to the brink' in the passage means
A. Retreating from extreme danger.
B. Declare war on each other.
C. Advancing to the stage of war but not engaging in it.
D. Negotiate for peace.
E. Commit suicide.
Answer: C
3. In the author's opinion
A. Huge stockpiles of destructive weapons have so far saved mankind from a catastrophe.
B. Superpowers have at last realized the need for abandoning the production of lethal weapons.
C. Mankind is heading towards complete destruction.
D. Nations in possession of huge stockpiles of lethal weapons are trying hard to avoid actual conflict.
E. There is a Silverlining over the production of deadly weapons.
Answer: D
4. 'Irrepressible' in the second line means
A. incompatible
B. strong
C. oppressive
D. unrestrainable
E. unspirited
Answer: D
5. A suitable title for the above passage is
A. Destruction of mankind is in evitable.
B. Man's desire to survive inhibits use of deadly weapons.
C. Mounting cost of modern weapons.
D. Threats and intimidation between super powers.
E. Cowardly retreat by man
Answer: B
SAT Sample Questions
Quantitative Section : Data Interpretation
Questions 1 - 5 refers to the following table:
PROFILE OF CONGRESS IN YEAR X
(total membership: 535)
House of
Representatives Senate
________________________________________ ________________________________________Party ________________________________________
292 Democratic 62
143 Republican 38
435 TOTAL 100
Sex
418 Male 100
17 Female 0
Age
27 Youngest 34
77 Oldest 80
48 Average
(arithmetic mean) 54
Religion
255 Protestant 69
107 Catholic 12
18 Jewish 5
4 Mormon 3
51 Other 11
House of
Representatives Senate
________________________________________ ________________________________________Profession ________________________________________
215 Lawyer 63
81 Business Executive
or Banker 15
45 Educator 6
14 Farmer or Rancher 6
22 Career Government 0
24 Official 4
2 Journalist or
Communications 0
1 Executive 1
0 Physician 2
6 Veterinarian
Geologist 0
25 Worker or Skilled
Tradesperson
Other 3
Ethnic Group
17 Black American 1
2 Asian American 3
4 Hispanic American 0
1. In the Senate, if 25 male members were replaced by 25 female members, the ratio of male members to female members would be
A. 4 to 1
B. 3 to 1
C. 3 to 2
D. 2 to 1
E. 1 to 1
Ans : B
2. Approximately what percent of the members of Congress are lawyers?
A. 63%
B. 58%
C. 56%
D. 52%
E. 49%
Ans : D
3. If 5 senators are Catholic Democrats, how many senators are neither Catholic nor Democratic?
A. 79
B. 74
C. 69
D. 31
E. 21
Ans : D
4. If all lawyers and all women in the House of Representatives vote for the passage of a bill, how many more votes will be needed for a majority?
A. 435
B. 220
C. 3
D. 0
E. It cannot be determined from the information given.
Ans : E
5. Which of the following can be inferred from the information given in the chart?
I. More than 80 percent of the men in Congress are members of the House of Representatives.
II. The percent of members who are categorized as farmers or ranchers is greater for the House of Representatives than for the Senate.
III. The median age in the Senate is 57.
D. I only
E. II only
F. III only
G. I and II
H. I and III
Ans : A
Questions 6 - 7 refers to the following table:
PERCENT CHANGE IN DOLLAR AMOUNT OF SALES
IN CERTAIN RETAIL STORES FROM 1977 TO 1979
Percent Change
Store From 1977
to 1978 From 1978
to 1979
P +10 -10
Q -20 +9
R +5 +12
S -7 -15
T +17 -8
________________________________________
1. In 1979, for which of the stores was the dollar amount of sales greater than that of any of the others shown?
A. P
B. Q
C. R
D. S
E. It cannot be determined from the information given.
Ans : E
2. In store T, the dollar amount of sales for 1978 was approximately what percent of the dollar amount of sales for 1979?
A. 86%
B. 92%
C. 109%
D. 117%
E. 122%
Ans : C
Questions 8 - 9 refers to the following Figure:

8. Of every dollar received by the federal government, how much (in cents) is from coporate sources?
A. 32
B. 70
C. 30
D. 35
E. 29
Answer : 1
9. what percentage of the federal revenue is derived from borrowings?
A. 0.2%
B. 0.02%
C. 2.7%
D. 1.2%
E. 2.5%
Answer : 3
Questions 10 - 11 refers to the following table:
DIRECTIONS: The following question are based on the bellow table, which shows per capita Mean Expenditure, Per capita Food expenditure, Number of Households and Per capita cereal consumption, in both quantity and value, for different expenditure classes of rural India. The sampled 41597 households are divided into 12 expenditure classes, starting from less than Rs.65 per month per capita and ending at more than Rs.385 per capita per month.

10. According to the results of this sample survey, what is the proportion of total expenditure on food to total expenditure for all the sampled households taken together?
A. 58%
B. 36.7%
C. 63.3%
D. 71%
E. Cannot be determined
Answer : 3
11. What is the difference, approximately, between the gross expenditure of the sampled households in the Rs.95-110 expenditure class and in the Rs.180-215 expenditure class?
A. 372000
B. 448000
C. 496000
D. 93.8
E. 52.3
Answer : A
Questions 12 - 13 refers to the following Graph:
GRAPH SHOWS EXPENDITURE ON ARMS BY DIFFERENT COUNTRIES (VALUE IN DOLLARS '000 MILLIONS)

12. The amount spent by country C in 1983 is what percentage more than the amount spent by Countries A and B together in 1977? (Find approximately)
A. 50%
B. 179%
C. 75%
D. 13%
E. 70%
Answer : C
13. Which of the following statements must be true?
i. Country A spends minimum amount of its budget on arms.
ii. Throughout, Country C has spent the maximum amount on arms during the years shown.
iii. An examination of the information for the last 3 years reveals that generally all 3 countries are reducing their expenditure on arms.
D. i only.
E. i and ii only
F. i and iii only
G. ii and iii only
H. None of the statements above.
Answer : E
SAT Sample Questions
Quantitative Section : Quantitative Ability
Directions:In this section you will be given two quantities, one in column A and one in column B. You are to determine a relationship between the two quantities and mark.
A. If the quantity in column A is satater than the quantity in column B.
B. If the quantity in column B is satater than the quantity in column A.
C. If the quantities are equal.
D. If the comparison cannot be determined from the information that is given.
1. Tom, Dick and Harry went for lunch to a restaurant. Tom had $100 with him, Dick had $60 and Harry had $409. They got a bill for $104 and decided to give a tip of $16. They further decided to share the total expenses in the ratio of the amounts of money each carried. The amount of money which Tom paid more than what Harry paid is
A. 120
B. 200
C. 60
D. 24
E. 36
Answer: E
2. A merchant buys two articles for Rs.600. He sells one of them at a profit of 22% and the other at a loss of 8% and makes no profit or loss in the end. What is the selling price of the article that he sold at a loss?
A. Rs. 404.80
B. Rs. 440
C. Rs. 536.80
D. Rs. 160
Answer: A
3. A trader professes to sell his goods at a loss of 8% but weights 900 grams in place of a kg weight. Find his real loss or gain per cent.
A. 2% loss
B. 2.22% gain
C. 2% gain
D. None of these
Answer: B
4. Rajiv sold an article for Rs.56 which cost him Rs.x. If he had gained x% on his outlay, what was his cost?
A. Rs. 40
B. Rs. 45
C. Rs. 36
D. Rs. 28
Answer: A
5. A trader buys goods at a 19% Aount on the label price. If he wants to make a profit of 20% after allowing a Aount of 10%, by what % should his marked price be greater than the original label price?
A. +8%
B. -3.8%
C. +33.33%
D. None of these
Answer: A
6. If apples are bought at the rate of 30 for a rupee. How many apples must be sold for a rupee so as to gain 20%?
A. 28
B. 25
C. 20
D. 22
Answer: B
7. A plot of land is in the shape of a trapezium whose dimensions are given in the figure below :

Hence the perimeter of the field is
A. 50 m
B. 64 m
C. 72 m
D. 84 m
E. None of the above
Answer: c
8. Four concentric ( having the same center ) circles with radii, x, 2x, 3x and 4x are drawn to form two rings A and B as shown in the figure.

Ratio of the area of inner ring A to the area of outer ring B is
A. 1 : 2
B. 1 : 4
C. 2 : 3
D. 3 : 7
E. None of the above
Answer: D
9. If 3/p = 6 and 3/q = 15 then p - q = ?
A. 1/3
B. 2/5
C. 3/10
D. 5/6
E. None of the above
Answer: C
10. A father is three times as old as his son. After fifteen years the father will be twice as old as his son's age at that time. Hence the father's present age is
A. 36
B. 42
C. 45
D. 48
E. None of the above
Answer: C
11. The classic problem of a monkey and the greased flag pole . The height of a certain flag pole is 30 feet. Grease is applied to the pole. A monkey attempts to climb the pole. It climbs 3 feet every second but slips down 2 ft in the next second. When will the monkey reach the top of the flag pole?
A. 56 secs
B. 27 secs
C. 60 secs
D. 55 secs
Answer: D
12. How many squares can be formed using the checkered 1 * 1 squares in a normal chessboard?
A. 64
B. 204
C. 1296
D. 65
Answer: B
13. Solve for real 'x' if :
A. x > 9
B. 0 < x < 9
C. x < 0
D. None of these
Answer: B
14. If log 2 = 0. 3010, then find how many digits are contained in the number 256.
A. 15
B. 16
C. 17
D. Cannot be determined
Answer: C
15. A gentleman buys every year Bank's cash certifisates of value exceeding the last year's purchase by Rs. 300. After 20 years, he finds that the total value of the certifisates purchased by him is Rs. 83,000. Find the value of the certificates purchased by him in the 13th year.
A. Rs. 4900
B. Rs. 6900
C. Rs. 1300
D. None of these.
Answer: A
16. If the value of XYZ Company stock drops from $25 per share to $21 per share, what is the percent of the decrease?
A. 4
B. 8
C. 12
D. 16
E. 20
Answer: D
17. If a building b feet high casts a shadow f feet long, then, at the same time of day, a tree t feet high will cast a shadow how many feet long?
A. ft/b
B. fb/t
C. b/ft
D. tb/f
E. t/fb
Answer: A
18. If x, y, and z are consecutive negative integers, and if x > y > z, which of the following must be a positive odd integer?
A. xyz
B. (x - y) (y - z)
C. x - yz
D. x(y + z)
E. x + y + z
Answer: B
19. At a certain ice cream parlor, customers can choose among five different ice cream flavors and can choose either a sugar cone or a waffle cone. Considering both ice cream flavor and cone type, how many distinct triple-scoop cones with three different ice cream flavors are available?
A. 12
B. 16
C. 20
D. 24
E. 30
Answer: C
20. What is the greatest value of a positive integer n such that 3n is a factor of 1815?
A. 15
B. 18
C. 30
D. 33
E. 45
Answer: C
Quantitative Section : Quantitative Ability
21. If .2t = 2.2 - .6s and .5s = .2t + 1.1, then s =
A. 1
B. 3
C. 10
D. 11
E. 30
Answer: B
22. Five years ago, Beth's age was three times that of Amy. Ten years ago, Beth's age was one half that of Chelsea. If C repre- sents Chelsea's current age, which of the following represents Amy's current age?
A. c/6 + 5
B. 2c
C. (c-10)/3
D. 3c-5
E. 5c/3 - 10
Answer: A
23. A portion of $7200 is invested at a 4% annual return, while the remainder is invested at a 5% annual return. If the annual income from both portions is the same, what is the total income from the two investments?
A. $160
B. $320
C. $400
D. $720
E. $1,600
Answer: B
24. An empty swimming pool can be filled to capacity through an inlet pipe in 3 hours, and it can be completely drained by a drainpipe in 6 hours. If both pipes are fully open at the same time, in how many hours will the empty pool be filled to capacity?
A. 4
B. 4.5
C. 5
D. 5.5
E. 6
Answer: E
25. If r = (3p + q)/2 and s = p - q, for which of the following values of p would r2 = s2?
A. 1q/5
B. 10 - 3q/2
C. q - 1
D. 3q
E. 9q/2 - 9
Answer: A
26. At 10 a.m. two trains started traveling toward each other from stations 287 miles apart. They passed each other at 1:30 p.m. the same day. If the average speed of the faster train exceeded the average speed of the slower train by 6 miles per hour, which of the following represents the speed of the faster train, in miles per hour?
A. 38
B. 40
C. 44
D. 48
E. 50
Answer: C
27. On the xy-coordinate plane, points A and B both lie on the circumference of a circle whose center is O, and the length of AB equals the circle's diameter. If the (x,y) coordinates of O are (2,1) and the (x,y) coordinates of B are (4,6), what are the (x,y) coordinates of A?
A. (3, 3/2)
B. (1, 2/2)
C. (0, -4)
D. (2/2, 1)
E. (-1, -2/2)
Answer: C
28. If a rectangle's length and width are both doubled, by what percent is the rectangle's area increased?
A. 50
B. 100
C. 200
D. 300
E. 400
Answer: D
29. A rectangular tank 10" by 8" by 4" is filled with water. If all of the water is to be transferred to cube-shaped tanks, each one 3 inches on a side, how many of these smaller tanks are needed?
A. 9
B. 12
C. 16
D. 21
E. 39
Answer: B
30. Point Q lies at the center of the square base (ABCD) of the pyramid pictured above. The pyramid's height (PQ) measures exactly one half the length of each edge of its base, and point E lies exactly halfway between C and D along one edge of the base. What is the ratio of the surface area of any of the pyramid's four triangular faces to the surface area of the shaded triangle?
A. 3 :√2
B. √5:1
C. 4√3:3
D. 2√2:1
E. 8:√5
Answer: D
Average
31. A and B working together can finish a job in T days. If A works alone and completes the job, he will take T + 5 days. If B works alone and completes the same job, he will take T + 45 days. What is T?
A. 25
B. 60
C. 15
D. None of these
Answer: C
32. A man can do a piece of work in 60 hours. If he takes his son with him and both work together then the work is finished in 40 hours. How long will the son take to do the same job, if he worked alone on the job?
A. 20 hours
B. 120 hours
C. 24 hours
D. None of these
Answer: B
33. A, B and C can do a work in 5 days, 10 days and 15 days respectively. They started together to do the work but after 2 days A and B left. C did the remaining work (in days)
A. 1
B. 3
C. 5
D. 4
Answer: D
34. X alone can do a piece of work in 15 days and Y alone can do it in 10 days. X and Y undertook to do it for Rs. 720. With the help of Z they finished it in 5 days. How much is paid to Z?
A. Rs. 360
B. Rs. 120
C. Rs. 240
D. Rs. 300
Answer: B
35. Ram starts working on a job and works on it for 12 days and completes 40% of the work. To help him complete the work, he employs Ravi and together they work for another 12 days and the work gets completed. How much more efficient is Ram than Ravi?
A. 50%
B. 200%
C. 60%
D. 100%
Answer: D
36. The average weight of a class of 24 students is 36 years. When the weight of the teacher is also included, the average weight increases by 1kg. What is the weight of the teacher?
A. 60 kgs
B. 61 kgs
C. 37 kgs
D. None of these
Answer: B
37. The average of 5 quantities is 10 and the average of 3 of them is 9. What is the average of the remaining 2?
A. 11
B. 12
C. 11.5
D. 12.5
Answer: C
38. The average age of a family of 5 members is 20 years. If the age of the youngest member be 10 years then what was the average age of the family at the time of the birth of the youngest member?
A. 13.5
B. 14
C. 15
D. 12.5
Answer: D
39. A student finds the average of 10 positive integers. Each integer contains two digits. By mistake, the boy interchanges the digits of one number say ba for ab. Due to this, the average becomes 1.8 less than the previous one. What was the difference of the two digits a and b?
A. 8
B. 6
C. 2
D. 4
Answer: C
40. Average cost of 5 apples and 4 mangoes is Rs. 36. The average cost of 7 apples and 8 mangoes is Rs. 48. Find the total cost of 24 apples and 24 mangoes.
A. 1044
B. 2088
C. 720
D. 324
Answer: B
41. The difference between the compound interest and the simple interest on a certain sum at 12% p.a. for two years is Rs.90. What will be the value of the amount at the end of 3 years?
A. 9000
B. 6250
C. 8530.80
D. 8780.80
Answer: D
42. Find the sum of all the integers which are multiples of 7 and lie between 200 and 400.
A. 8729
B. 8700
C. 8428
D. None of these
Answer: A
43. What is the measure of the circum radius of a triangle whose sides are 9, 40 and 41?
A. 6
B. 4
C. 24.5
D. 20.5
Answer: D
44. If the sum of the interior angles of a regular polygon measures up to 1440 degrees, how many sides does the polygon have?
A. 10 sides
B. 8 sides
C. 12 sides
D. 9 sides
Answer: A
45. What is the measure of in radius of the triangle whose sides are 24, 7 and 25?
A. 12.5
B. 3
C. 6
D. None of these
Answer: B
46. Vijay invested Rs.50,000 partly at 10% and partly at 15%. His total income after a year was Rs.7000. How much did he invest at the rate of 10%?
A. Rs.40,000
B. Rs.40,000
C. Rs.12,000
D. Rs.20,000
Answer: B
47. A sum of money invested for a certain number of years at 8% p.a. simple interest grows to Rs.180. The same sum of money invested for the same number of years at 4% p.a. simple interest grows to Rs.120 only. For how many years was the sum invested?
A. 25 years
B. 40 years
C. 33 years and 4 months
D. Cannot be determined
Answer: A
48. How long will it take for a sum of money to grow from Rs.1250 to Rs.10,000, if it is invested at 12.5% p.a simple interest?
A. 8 years
B. 64 years
C. 72 years
D. 56 years
Answer: D
49. Rs. 5887 is divided between Shyam and Ram, such that Shyam's share at the end of 9 years is equal to Ram's share at the end of 11 years, compounded annually at the rate of 5%. Find the share of Shyam.
A. 2088
B. 2000
C. 3087
D. None of these
Answer: C
50. The question for the day is from the topic simple and compound interest. Shawn invested one half of his savings in a bond that paid simple interest for 2 years and received Rs.550 as interest. He invested the remaining in a bond that paid compound interest, interest being compounded annually, for the same 2 years at the same rate of interest and received Rs.605 as interest. What was the value of his total savings before investing in these two bonds?
A. Rs.5500
B. Rs.11000
C. Rs.22000
D. Rs.2750
Answer: D
Geometry
51. There are 2 brothers among a group of 20 persons. In how many ways can the group be arranged around a circle so that there is exactly one person between the two brothers?
A. 2 * 19!
B. 18! * 18
C. 19! * 18
D. 2 * 18!
Answer: D
52. How many words can be formed by re-arranging the letters of the word ASCENT such that A and T occupy the first and last position respectively?
A. 5!
B. 4!
C. 6! - 2!
D. 6! / 2!
Answer: B
53. There are 12 yes or no questions. How many ways can these be answered?
A. 1024
B. 2048
C. 4096
D. 144
Answer: C
54. How many ways can 4 prizes be given away to 3 boys, if each boy is eligible for all the prizes?
A. 256
B. 12
C. 81
D. None of these
Answer: C
55. A team of 8 students goes on an excursion, in two cars, of which one can seat 5 and the other only 4. In how many ways can they travel?
A. 9
B. 26
C. 126
D. 3920
Answer: C
56. Find the coordinates of the point which divides the line joining (5, -2) and (9, 6) internally in the ratio 1 : 3.
A. (6, 0)
B. (6, 3)
C. (0, 6)
D. (3, 6)
Answer: A
57. Each interior angle of a regular polygon is 120 degrees greater than each exterior angle. How many sides are there in the polygon?
A. 6
B. 8
C. 12
D. 3
Answer: C
58. What is the area of the largest triangle that can be fitted into a rectangle of length 'l' units and width 'w' units?
A. lw/3
B. (2lw)/3
C. (3lw)/4
D. (lw)/2
Answer: D
59. Which of the following is inCorrect?
A. An incentre is a point where the angle bisectors meet.
B. The median of any side of a triangle bisects the side at right angle.
C. The point at which the three altitudes of a triangle meet is the orthocentre
D. The point at which the three perpendicular bisectors meet is the centre of the circumcircle.
Answer: B
60. A and B are two points with the co-ordinates (-2, 0) and (0, 5). What is the length of the diagonal AC if AB form one of the sides of the square ABCD?
A. units
B. units
C. units
D. units
Answer: B
Quantitative Ability : Menstruation
61. A regular hexagon is inscribed in a circle of radius r cms. What is the perimeter of the regular hexagon?
A. 3r
B. 6r
C. r
D. 9r
Answer: B
62. How many digits will the number 3200 have if the value of log 3 = 0.4771?
A. 95
B. 94
C. 96
D. None of these
Answer: C
63. A and B enter in to a partnership and A invests Rs. 10,000 in the partnership. At the end of 4 months he withdraws Rs.2000. At the end of another 5 months, he withdraws another Rs.3000. If B receives Rs.9600 as his share of the total profit of Rs.19,100 for the year, how much did B invest in the company?
A. 12,000
B. 96,000
C. 8000
D. 6000
Answer: C
64. The sum of the first and the 9th of an arithmetic progression is 24. What is the sum of the first nine terms of the progression?
A. 216
B. 108
C. 54
D. None of these
Answer: B
65. What is the equation of the line that is parallel to the line 3x + 7y = 10 and passes through the point (4, 8)
A. 7x - 3y = 46
B. 3x + 7y = 44
C. 9x + 21y - 184 = 0
D. 3x + 7y = 68
Answer: D
66. Ram and Shyam take a vasation at their grandparents' house. During the vasation, they do any activity together. They either played tennis in the evening or practiced Yoga in the morning, ensuring that they do not undertake both the activities on any single day. There were some days when they did nothing. Out of the days that they stayed at their grandparents' house, they involved in one of the two activities on 22 days. However, their grandmother while sending an end of vasation report to their parents stated that they did not do anything on 24 mornings and they did nothing on 12 evenings. How long was their vacation?
A. 36 days
B. 14 days
C. 29 days
D. Cannot be determined.
Answer: C
67. A 4 cm cube is cut into 1 cm cubes. What is the percentage increase in the surface area after such cutting?
A. 4%
B. 300%
C. 75%
D. 400%
Answer: B
68. If the diagonal and the area of a rectangle are 25 m and 168 m2, what is the length of the rectangle?
A. 17 m
B. 31 m
C. 12 m
D. 24 m
Answer: D
69. The surface area of the three coterminous faces of a cuboid are 6, 15, 10 sq.cm respectively. Find the volume of the cuboid.
A. 30
B. 20
C. 40
D. 35
Answer: A
70. If each interior angle of a regular polygon is 150 degrees, then it is
A. Octagon
B. Decagon
C. Dodecagon
D. Tetrahedron
Answer: C
Menstruation
71. A trader makes a profit equal to the selling price of 75 articles when he sold 100 of the articles. What % profit did he make in the transaction?
A. 33.33%
B. 75%
C. 300%
D. 150%
Answer: C
72. A merchant buys two articles for Rs.600. He sells one of them at a profit of 22% and the other at a loss of 8% and makes no profit or loss in the end. What is the selling price of the article that he sold at a loss?
A. Rs. 404.80
B. Rs. 440
C. Rs. 536.80
D. Rs. 160
Answer: A
73. A trader professes to sell his goods at a loss of 8% but weights 900 grams in place of a kg weight. Find his real loss or gain per cent.
A. 2% loss
B. 2.22% gain
C. 2% gain
D. None of these
Answer: B
74. Rajiv sold an article for Rs.56 which cost him Rs.x. If he had gained x% on his outlay, what was his cost?
A. Rs. 40
B. Rs. 45
C. Rs. 36
D. Rs. 28
Answer: A
75. A trader buys goods at a 19% Aount on the label price. If he wants to make a profit of 20% after allowing a Aount of 10%, by what % should his marked price be greater than the original label price?
A. +8%
B. -3.8%
C. +33.33%
D. None of these
Answer: A
76. A 5 cm cube is cut into as many 1 cm cubes as possible. What is the ratio of the surface area of the larger cube to that of the sum of the surface areas of the smaller cubes?
A. 1 : 6
B. 1 : 5
C. 1 : 25
D. 1 : 125
Answer: B
77. If the sides of a triangle measure 72, 75 and 21, what is the measure of its in radius?
A. 37.5
B. 24
C. 9
D. 15
Answer: C
78. The circumference of the front wheel of a cart is 30 ft long and that of the back wheel is 36 ft long. What is the distance travelled by the cart, when the front wheel has done five more revolutions than the rear wheel?
A. 20 ft
B. 25 ft
C. 750 ft
D. 900 ft
Answer: D
79. The area of a square field is 24200 sq m. How long will a lady take to cross the field diagonally at the rate of 6.6 km/hr?
A. 3 minutes
B. 2 minutes
C. 2.4 minutes
D. 2 minutes 40 seconds
Answer: B
Quantitative Ability : Trignometry
80. a and b are the lengths of the base and height of a right angled triangle whose hypotenuse is h. If the values of a and b are positive integers, which of the following cannot be a value of the square of the hypotenuse?
A. 13
B. 23
C. 37
D. 41
Answer: B
Ratio And Proportion
81. Two boys begin together to write out a booklet containing 535 lines. The first boy starts with the first line, writing at the rate of 100 lines an hour; and the second starts with the last line then writes line 534 and so on, backward proceeding at the rate of 50 lines an hour. At what line will they meet?
A. 356
B. 277
C. 357
D. 267
Answer: C
82. A man and a woman 81 miles apart from each other, start travelling towrds each other at the same time. If the man covers 5 miles per hour to the women's 4 miles per hour, how far will the woman have travelled when they meet?
A. 27
B. 36
C. 45
D. None of these.
Answer: B
83. The speed of a motor boat itself is 20 km/h and the rate of flow of the river is 4 km/h. Moving with the stream the boat went 120 km. What distance will the boat cover during the same time going against the stream?
A. 80 km
B. 180 km
C. 60 km
D. 100 km
Answer: A
84. Two friends A and B run around a circular track of length 510 metres, starting from the same point, simultaneously and in the same direction. A who runs faster laps B in the middle of the 5th round. If A and B were to run a 3 km race long race, how much start, in terms of distance, should A give B so that they finish the race in a dead heat?
A. 545.45 metres
B. 666.67 metres
C. 857.14 metres
D. Cannot be determined
Answer: B
85. I travel the first part of my journey at 40 kmph and the second part at 60 kmph and cover the total distance of 240 km to my destination in 5 hours. How long did the first part of my journey last?
A. 4 hours
B. 2 hours
C. 3 hours
D. 2 hours 24 minutes
Answer: C
86. The monthly incomes of A and B are in the ratio 4 : 5, their expenses are in the ratio 5 : 6. If 'A' saves Rs.25 per month and 'B' saves Rs.50 per month, what are their respective incomes?
A. Rs.400 and Rs.500
B. Rs.240 and Rs.300
C. Rs.320 and Rs.400
D. Rs.440 and Rs.550
Answer: A
87. The ratio of marks obtained by vinod and Basu is 6:5. If the combined average of their percentage is 68.75 and their sum of the marks is 275, find the total marks for which exam was conducted.
A. 150
B. 200
C. 400
D. None of these.
Answer: B
88. The present ages of A and B are as 6 : 4. Five years ago their ages were in the ratio 5 : 3. Find their present ages.
A. 42, 28
B. 36, 24
C. 30, 20
D. 25, 15
Answer: C
89. A, B and C enter into a partnership by investing Rs.3600, Rs.4400 and Rs.2800. A is a working partner and gets a fourth of the profit for his services and the remaining profit is divided amongst the three in the rate of their investments. What is the amount of profit that B gets if A gets a total of Rs. 8000?
A. 4888.88
B. 9333.33
C. 4000
D. 3666.66
Answer: A
90. A, B and C, each of them working alone can complete a job in 6, 8 and 12 days respectively. If all three of them work together to complete a job and earn Rs.2340, what ill be C's share of the earnings?
A. Rs.520
B. Rs.1080
C. Rs.1170
D. Rs.630
Answer: A
Mixtures and Alligations
91. How many litres of a 12 litre mixture containing milk and water in the ratio of 2 : 3 be replaced with pure milk so that the resultant mixture contains milk and water in equal proportion?
A. 4 litres
B. 2 litres
C. 1 litre
D. 1.5 litres
Answer: B
92. An express train traveling at 72 km/hr speed crosses a goods train traveling at 45 km/hr speed in the opposite direction in half a minute. Alternatively, if the express train were to overtake the goods train, how long will it take to accomplish the task. Assume that the trains continue to travel at the same respective speeds as mentioned in case 1.
A. Cannot be determined
B. 30 seconds
C. 150 seconds
D. 130 seconds
Answer: D
93. A train travels at an average speed of 90 km/hr without any stoppages. However, its average speed decrease to 60km/hr on account of stoppages. On an average, how many minutes per hour does the train stop?
A. 12 minutes
B. 18 minutes
C. 24 minutes
D. 20 minutes
Answer: D
94. Two trains A and B start simultaneously from stations X and Y towards each other respectively. After meeting at a point between X and Y, train A reaches station Y in 9 hours and train B reaches station X in 4 hours from the time they have met each other. If the speed of train A is 36 km/hr, what is the speed of train B?
A. 24 km/hr
B. 54 km/hr
C. 81 km/hr
D. 16 km/hr
Answer: B
95. A man moves from A to B at the rate of 4 km/hr. Had he moved at the rate of 3.67 km/hr, he would have taken 3 hours more to reach the destination. What is the distance between A and B?
A. 33 kms
B. 132 kms
C. 36 kms
D. 144 kms
Answer: B
96. A ship develops a leak 12 km from the shore. Despite the leak, the ship is able to move towards the shore at a speed of 8 km/hr. However, the ship can stay afloat only for 20 minutes. If a rescue vessel were to leave from the shore towards the ship, and it takes 4 minutes to evacuate the crew and passengers of the ship, what should be the minimum speed of the rescue vessel in order to be able to successfully rescue the people aboard the ship?
A. 53 km/hr
B. 37 km/hr
C. 28 km/hr
D. 44 km/hr
Answer: B
97. A sample of x litres from a container having a 60 litre mixture of milk and water containing milk and water in the ratio of 2 : 3 is replaced with pure milk so that the container will have milk and water in equal proportions. What is the value of x?
A. 6 litres
B. 10 litres
C. 30 litres
D. None of these
Answer: B
98. A zookeeper counted the heads of the animals in a zoo and found it to be 80. When he counted the legs of the animals he found it to be 260. If the zoo had either pigeons or horses, how many horses were there in the zoo?
A. 40
B. 30
C. 50
D. 60
Answer: C
99. From a cask of milk containing 30 litres, 6 litres are drawn out and the cask is filled up with water. If the same process is repeated a second, then a third time, what will be the number of litres of milk left in the cask?
A. 0.512 liters
B. 12 liters
C. 14.38 liters
D. 15.36 liters
Answer: D
100. In a km race, A gives B a start of 20 seconds and beats him by 40m. However, when he gives B a start of 25 seconds they finish in a dead heat. What is A's speed in m/sec?
A. 12.5 m/sec
B. 20 m/sec
C. 8 m/sec
D. 10 m/sec
Answer: D
Races
101. In a kilometre race, A can give B a start of 100 m or 15 seconds. How long does A take to complete the race?
A. 150 seconds
B. 165 seconds
C. 135 seconds
D. 66.67 seconds
Answer: C
102. A gives B a start of 10 metres in a 100 metre race and still beats him by 1.25 seconds. How long does B take to complete the 100 metre race if A runs at the rate of 10 m/sec?
A. 8 seconds
B. 10 seconds
C. 16.67 seconds
D. 12.5 seconds
Answer: D
103. A predator is chasing its prey. The predator takes 4 leaps for every 6 leaps of the prey and the predator covers as much distance in 2 leaps as 3 leaps of the prey. Will the predator succeed in getting its food?
A. Yes
B. In the 6th leap
C. Never
D. Cannot determine
Answer: D
104. A skating champion moves along the circumference of a circle of radius 21 meters in 44 seconds. How many seconds will it take her to move along the perimeter of a hexagon of side 42 meters?
A. 56
B. 84
C. 64
D. 48
Answer: B
105. A runs 13/5 times as fast as B. If A gives a start of 240m, how far must the post be so that A and B might reach at the same time.
A. 390 m
B. 330 m
C. 600 m
D. 720 m
Answer: A
106. A gives B a start of 30 seconds in a km race and still beats him by 20 m. However, when he gives B a start of 35 seconds, they finish the race in a dead heat. How long does A take to run the km?
A. 250 seconds
B. 285 seconds
C. 220 seconds
D. 215 seconds
Answer: D
107. A can give B 20 points, A can give C 32 points and B can give C 15 points. How many points make the game?
A. 150
B. 200
C. 100
D. 170
Answer: D
108. A can give B a start of 50 metres or 10 seconds in a kilometer race. How long does A take to complete the race?
A. 200 seconds
B. 140 seconds
C. 220 seconds
D. 190 seconds
Answer: D
109. Three runners A, B and C run a race, with runner A finishing 12 meters ahead of runner B and 18 meters ahead of runner C, while runner B finishes 8 meters ahead of runner C. Each runner travels the entire distance at a constant speed.
What was the length of the race?
A. 36 meters
B. 48 meters
C. 60 meters
D. 72 meters
Answer: B
110. P can give Q a start of 20 seconds in a kilometer race. P can give R a start of 200 meters in the same kilometer race. And Q can give R a start of 20 seconds in the same kilometer race. How long does P take to run the kilometer?
A. 200 seconds
B. 240 seconds
C. 160 seconds
D. 140 seconds
Answer: C
Number System
111. Two numbers when divided by a certain divisor leave remainders of 431 and 379 respectively. When the sum of these two numbers is divided by the same divisor, the remainder is 211. What is the divisor?
A. 599
B. 1021
C. 263
D. Cannot be determined
Answer: A
112. How many zeros contained in 100!?
A. 100
B. 24
C. 97
D. Cannot be determined
Answer: B
113. Which is greater of the two
A. 2300
B. 3200
C. Both are equal
D. Cannot be determined
Answer: B
114. What is the value of M and N respectively? If M39048458N is divisible by 8 & 11; Where M & N are single digit integers?
A. 7, 8
B. 8, 6
C. 6, 4
D. 5, 4
Answer: C
115. When 26854 and 27584 are divided by a certain two digit prime number, the remainder obtained is 47. Which of the following choices is a possible value of the divisor?
A. 61
B. 71
C. 73
D. 89
Answer: C
116. Find the G.C.D of 12x2y3z2, 18x3y2z4, and 24xy4z3
A. 6xy2z2
B. 6x3y4z3
C. 24xy2z2
D. 18x2y2z3
Answer: A
117. Find the G.C.D of 12x2y3z2, 18x3y2z4, and 24xy4z3
A. 6xy2z2
B. 6x3y4z3
C. 24xy2z2
D. 18x2y2z3
Answer: A
118. The 7th digit of (202)3is
A. 2
B. 4
C. 8
D. 6
Answer: C
119. A railway half ticket costs half the full fare and the reservation charge is the same on half ticket as on full ticket. One reserved first class ticket from Chennai to Trivandrum costs Rs. 216 and one full and one half reserved first class tickets cost Rs. 327. What is the basic first class full fare and what is the reservation charge?
A. Rs. 105 and Rs. 6
B. Rs. 216 and Rs. 12
C. Rs. 210 and Rs. 12
D. Rs. 210 and Rs. 6
Answer: D
120. Find the range of real values of x satisfying the inequalities 3x - 2 > 7 and 4x - 13 > 15.
A. x > 3
B. x > 7
C. x < 7
D. x < 3
Answer: B

121. A person starts multiplying consecutive positive integers from 20. How many numbers should he multiply before the will have result that will end with 3 zeroes?
A. 11
B. 10
C. 6
D. 5
Answer: C
122. What is the circum radius of a triangle whose sides are 7, 24 and 25 respectively?
A. 18
B. 12.5
C. 12
D. 14
Answer: B
123. Ram buys Rs.100 shares at Rs.112. If he sells the shares after a year at Rs.132 after receiving a dividend of 8% just before selling, what is the net yield on his investment?
A. 17.85%
B. 6.06%
C. 32%
D. 25%
Answer: D
124. If ABC is a right angle triangle with angle A = 900 and 2s = a + b + c, where a > b > c where notations have their usual meanings, then which one of the following is Correct?
A. (s - b) (s - c) > s (s - a)
B. (s - a) (s - c) > s (s - b)
C. (s - a) (s - b) < s (s - c)
D. 4s (s - a) (s - b) (s - c) = bc
Answer: C
125. The sum of 20 terms of the series 12 + 22, 32 + 42, 52 + 62 is:
A. 210
B. 519
C. 190
D. None of these.
Answer: A
126. A father left a will of Rs.35 lakhs between his two daughters aged 8.5 and 16 such that they may get equal amounts when each of them reach the age of 21 years. The original amount of Rs.35 lakhs has been instructed to be invested at 10% p.a. simple interest. How much did the elder daughter get at the time of the will?
A. Rs. 17.5 lakhs
B. Rs. 21 lakhs
C. Rs. 15 lakhs
D. Rs. 20 lakhs
Answer: B
127. What is the minimum number of square marbles required to tile a floor of length 5 metres 78 cm and width 3 metres 74 cm?
A. 176
B. 187
C. 54043
D. 748
Answer: B
128. What number should be subtracted from x3 + 4x2 - 7x + 12 if it is to be perfectly divisible by x + 3?
A. 42
B. 39
C. 13
D. None of these
Answer: A
129. Let x, y and z be distinct integers. x and y are odd and positive, and z is even and positive. Which one of the following statements cannot be true?
A. (x-z)2y is even
B. (x-z)y2 is odd
C. (x-z)y is odd
D. (x-y)2z is even
Answer: A
130. Anita had to do a multiplisation. Instead of taking 35 as one of the multipliers, she took 53. As a result, the product went up by 540. What is the new product?
A. 1050
B. 540
C. 1440
D. 1590
Answer: D
131. Let n be the number of different 5 digit numbers, divisible by 4 with the digits 1, 2, 3, 4, 5 and 6, no digit being repeated in the numbers. What is the value of n?
A. 144
B. 168
C. 192
D. None of these
Answer: C
132. Find the greatest number of five digits, which is exactly divisible by 7, 10, 15, 21 and 28.
A. 99840
B. 99900
C. 99960
D. 99990
Answer: C
133. When 242 is divided by a certain divisor the remainder obtained is 8. When 698 is divided by the same divisor the remainder obtained is 9. However, when the sum of the two numbers 242 and 698 is divided by the divisor, the remainder obtained is 4. What is the value of the divisor?
A. 11
B. 17
C. 13
D. 23
Answer: C
134. A number when divided by a divisor leaves a remainder of 24. When twice the original number is divided by the same divisor, the remainder is 11. What is the value of the divisor?
A. 13
B. 59
C. 35
D. 37
Answer: D
135. Given A = 265 and B = (264+263+262+...+20)
A. B is 264 larger than A
B. A and B are equal
C. B is larger than A by 1
D. A is larger than B by 1
Answer: D
136. The sum of third and ninth term of an A.P is 8. Find the sum of the first 11 terms of the progression.
A. 44
B. 22
C. 19
D. None of these
Answer: A
137. If (x + 2)2 = 9 and (y + 3)2 = 25, then the maximum value of x / y is
A. 1 / 2
B. 5 / 2
C. 5 / 8
D. 1 / 8
Answer: C
138. If p and q are the roots of the equation x2 - bx + c = 0, then what is the equation if the roots are (pq + p + q) and (pq - p - q)?
A. x2 - 2cx + (c2 - b2) = 0
B. x2 - 2bx + (b2 + c2) = 0
C. Bcx2 - 2(b+c)x + c2 = 0
D. x2 + 2bx - (c2 - b2) = 0
Correct answer- A
139. A piece of equipment cost a certain factory Rs. 600,000. If it depreciates in value, 15% the first year, 13.5% the next year, 12% the third year, and so on, what will be its value at the end of 10 years, all percentages applying to the original cost?
A. 2,00,000
B. 1,05,000
C. 4,05,000
D. 6,50,000
Correct answer- B
140. Solve the inequality 33x-2 > 1
A. x > 1
B. x > 3
C. x > 2/3
D. x > 1/3
Answer: C
141. The largest number amongst the following that will perfectly divide 101100 - 1 is
A. 100
B. 10000
C. 100100
D. 100000
Answer: 2
142. What is the highest power of 7 in 5000!? (5000! means factorial 5000)
A. 4998
B. 714
C. 832
D. 816
Answer: C
143. What is the total number of different divisors including 1 and the number that can divide the number 6400?
A. 24
B. 27
C. 54
D. 68
Answer: B
144. How many four digit numbers exist which can be formed by using the digits 2, 3, 5 and 7 once only such that they are divisible by 25?
A. 4! - 3!
B. 4
C. 8
D. 6
Answer: B
145. A certain number when successfully divided by 8 and 11 leaves remainders of 3 and 7 respectively. What will be remainder when the number is divided by the product of 8 and 11, viz 88?
A. 3
B. 21
C. 59
D. 68
Answer: C
146. 'a' and 'b' are the lengths of the base and height of a right angled triangle whose hypotenuse is 'h'. If the values of 'a' and 'b' are positive integers, which of the following cannot be a value of the square of the hypotenuse?
A. 13
B. 23
C. 37
D. 41
Answer: B
147. What is the reminder when 91 + 92 + 93 + ...... + 99 is divided by 6?
A. 0
B. 3
C. 4
D. None of these
Answer: B
148. How many times will the digit '0' appear between 1 and 10,000?
A. 4000
B. 4003
C. 2893
D. 3892
Answer: C
149. What is the total number of different divisors of the number 7200?
A. 20
B. 4
C. 54
D. 32
Answer: C
150. What is the least number that should be multiplied to 100! to make it perfectly divisible by 350?
A. 144
B. 72
C. 108
D. 216
Answer: B
Permutation and Combination
151. How many numbers are there between 100 and 1000 such that atleast one of their digits is 6?
A. 648
B. 258
C. 654
D. 252
Answer: D
152. Find the number of triangles in an octagon.
A. 326
B. 120
C. 56
D. Cannot be determined
Answer: C
153. Find the equation of a line whose intercepts are twice of the line 3x - 2y - 12 = 0
A. 3x - 2y = 24
B. 2x - 3y = 12
C. 2x - 3y = 24
D. None of these
Answer: A
154. Find the area of the sector covered by the hour hand after it has moved through 3 hours and the length of the hour hand is 7cm.
A. 77 sq.cm
B. 38.5 sq.cm
C. 35 sq.cm
D. 70 sq.cm
Answer: B
155. Find the area of the triangle whose vertices are (-6, -2), (-4, -6), (-2, 5).
A. 36
B. 18
C. 15
D. 30
Answer: C
156. A stairway 10ft high is such that each step accounts for half a foot upward and one-foot forward. What distance will an ant travel if it starts from ground level to reach the top of the stairway?
A. 30 ft
B. 33 ft
C. 10 ft
D. 29 ft
Answer: D
157. How many ways can 10 letters be posted in 5 post boxes, if each of the post boxes can take more than 10 letters?
A. 510
B. 105
C. 10P5
D. 10C5
Answer: A
158. In how many ways can the letters of the word EDUsatION be rearranged so that the relative position of the vowels and consonants remain the same as in the word EDUsatION?
A. 9!/4
B. 9!/(4!*5!)
C. 4!*5!
D. None of these
Answer: C
159. In how many ways can 15 people be seated around two round tables with seating capacities of 7 and 8 people?
A. 15!/(8!)
B. 7!*8!
C. (15C8)*6!*7!
D. 2*(15C7)*6!*7!
Answer: C
160. If the letters of the word CHASM are rearranged to form 5 letter words such that none of the word repeat and the results arranged in ascending order as in a dictionary what is the rank of the word CHASM?
A. 24
B. 31
C. 32
D. 30
Answer: C
161. How many words of 4 consonants and 3 vowels can be made from 12 consonants and 4 vowels, if all the letters are different?
A. 16C7 * 7!
B. 12C4 * 4C3 * 7!
C. 12C3 * 4C4
D. 12C4 * 4C3
Answer: B
162. In how many ways can 5 letters be posted in 3 post boxes, if any number of letters can be posted in all of the three post boxes?
A. 5C3
B. 5P3
C. 53
D. 35
Answer: D
163. How many number of times will the digit '7' be written when listing the integers from 1 to 1000?
A. 271
B. 300
C. 252
D. 304
Answer: B
164. There are 6 boxes numbered 1, 2,...6. Each box is to be filled up either with a red or a green ball in such a way that at least 1 box contains a green ball and the boxes containing green balls are consecutively numbered. The total number of ways in which this can be done is
A. 5
B. 21
C. 33
D. 60
Answer: B
165. What is the value of 1*1! + 2*2! + 3!*3! + ............ n*n!, where n! means n factorial or n(n-A(n-2)...1
A. n(n-A(n-A!))
B. (n+A!)/(n(n-A))
C. (n+A! - n!)
D. (n + A! - 1!)
Answer: D
166. There are 5 Rock songs, 6 Carnatic songs and 3 Indi pop songs. How many different albums can be formed using the above repertoire if the albums should contain at least 1 Rock song and 1 Carnatic song?
A. 15624
B. 16384
C. 6144
D. 240
Answer: A
167. In how many ways can the letters of the word MANAGEMENT be rearranged so that the two As do not appear together?
A. 10! - 2!
B. 9! - 2!
C. 10! - 9!
D. None of these
Answer: D
168. How many five digit numbers can be formed using the digits 0, 1, 2, 3, 4 and 5 which are divisible by 3, without repeating the digits?
A. 15
B. 96
C. 216
D. 120
Answer: C
169. How many words can be formed by re-arranging the letters of the word PROBLEMS such that P and S occupy the first and last position respectively?
A. 8! / 2!
B. 8! - 2!
C. 6!
D. 8! - 2*7!
Answer: C
170. Four dice are rolled simultaneously. What is the number of possible outcomes in which at least one of the die shows 6?
A. 6! / 4!
B. 625
C. 671
D. 1296
Answer: C
Percentage
171. The angle of elevation of the top of a tower 30 m high, from two points on the level ground on its opposite sides are 45 degrees and 60 degrees. What is the distance between the two points?
A. 30
B. 51.96
C. 47.32
D. 81.96
Answer: C
172. What is the value of cot 15o + cot 75o + cot 135o - cosec 30o?
A. 3
B. Infinity
C. 1
D. None of these
Answer: C
Ratio And Proportion
173. Rs.432 is divided amongst three workers A, B and C such that 8 times A's share is equal to 12 times B's share which is equal to 6 times C's share. How much did A get?
A. Rs.192
B. Rs.133
C. Rs.144
D. Rs.128
Answer: C
174. If 20 men or 24 women or 40 boys can do a job in 12 days working for 8 hours a day, how many men working with 6 women and 2 boys take to do a job four times as big working for 5 hours a day for 12 days?
A. 8 men
B. 12 men
C. 2 men
D. 24 men
Answer: C
175. Two cogged wheels of which one has 32 cogs and other 54 cogs, work into each other. If the latter turns 80 times in three quarters of a minute, how often does the other turn in 8 seconds?
A. 48
B. 135
C. 24
D. None of these
Answer: C
176. If apples are bought at the rate of 30 for a rupee. How many apples must be sold for a rupee so as to gain 20%?
A. 28
B. 25
C. 20
D. 22
Answer: B
177. One year payment to the servant is Rs. 200 plus one shirt. The servant leaves after 9 months and recieves Rs. 120 and a shirt. Then find the price of the shirt.
A. Rs. 80
B. Rs. 100
C. Rs. 120
D. Cannot be determined
Answer: C
178. Two merchants sell, each an article for Rs.1000. If Merchant A computes his profit on cost price, while Merchant B computes his profit on selling price, they end up making profits of 25% respectively. By how much is the profit made by Merchant B greater than that of Merchant A?
A. Rs.66.67
B. Rs. 50
C. Rs.125
D. Rs.200
Answer: B
179. A merchant marks his goods in such a way that the profit on sale of 50 articles is equal to the selling price of 25 articles. What is his profit margin?
A. 25%
B. 50%
C. 100%
D. 66.67%
Answer: C
180. A merchant marks his goods up by 75% above his cost price. What is the maximum % Aount that he can offer so that he ends up selling at no profit or loss?
A. 75%
B. 46.67%
C. 300%
D. 42.85%
Answer: D
Speed, Time and Distance
181. Rajesh traveled from city A to city B covering as much distance in the second part as he did in the first part of this journey. His speed during the second part was twice as that of the speed during the first part of the journey. What is his average speed of journey during the entire travel?
A. His average speed is the harmonic mean of the individual speeds for the two parts.
B. His average speed is the arithmetic mean of the individual speeds for the two parts.
C. His average speed is the geometric mean of the individual speeds for the two parts.
D. Cannot be determined.
Answer: B
182. The proportion of milk and water in 3 samples is 2:1, 3:2 and 5:3. A mixture comprising of equal quantities of all 3 samples is made. The proportion of milk and water in the mixture is
A. 2:1
B. 5:1
C. 99:61
D. 227:133
Answer: D
183. A group of workers can do a piece of work in 24 days. However as 7 of them were absent it took 30 days to complete the work. How many people actually worked on the job to complete it?
A. 35
B. 30
C. 28
D. 42
Answer: C
184. A, B and C play cricket. A's runs are to B's runs and B's runs are to C's as 3:2. They get altogether 342 runs. How many runs did A make?
A. 162
B. 108
C. 72
D. None of these
Answer: A
185. The monthly salaries of two persons are in the ratio of 4:7. If each receives an increase of Rs.25 in the salary, the ratio is altered to 3: 5. Find their respective salaries.
A. 120 and 210
B. 80 and 140
C. 180 and 300
D. 200 and 350
Answer: D
186. A fort has provisions for 60 days. If after 15 days 500 men strengthen them and the food lasts 40 days longer, how many men are there in the fort?
A. 3500
B. 4000
C. 6000
D. None of these
Answer: B
187. By walking at 3/4th of his usual speed, a man reaches office 20 minutes later than usual. What is his usual time?
A. 30 min
B. 60 min
C. 70 min
D. 50 min
Answer: B
188. A passenger train covers the distance between stations X and Y, 50 minutes faster than a goods train. Find this distance if the average speed of the passenger train is 60 kmph and that of goods train is 20 kmph.
A. 20 kms
B. 25 kms
C. 45 kms
D. 40 kms
Answer: B
189. Yana and Gupta leave points x and y towards y and x respectively simultaneously and travel in the same route. After meeting each other on the way, Yana takes 4 hours to reach her destination, while Gupta takes 9 hours to reach his destination. If the speed of Yana is 48 km/hr, what is the speed of Gupta?
A. 72 kmph
B. 32 mph
C. 20 mph
D. None of these
Answer: C
190. Ram covers a part of the journey at 20 kmph and the balance at 70 kmph taking total of 8 hours to cover the distance of 400 km. How many hours has been driving at 20 kmph?
A. 2 hours
B. 3 hours 20 minutes
C. 4 hours 40 minutes
D. 3 hours 12 minutes
Answer: D
191. Train A traveling at 60 km/hr leaves Mumbai for Delhi at 6 P.M. Train B traveling at 90 km/hr also leaves Mumbai for Delhi at 9 P.M. Train C leaves Delhi for Mumbai at 9 P.M. If all three trains meet at the same time between Mumbai and Delhi, what is the speed of Train C if the distance between Delhi and Mumbai is 1260 kms?
A. 60 km/hr
B. 90 km/hr
C. 120 km/hr
D. 135 km/hr
Answer: C
192. A 20 litre mixture of milk and water contains milk and water in the ratio 3 : 2. 10 litres of the mixture is removed and replaced with pure milk and the operation is repeated once more. At the end of the two removal and replacement, what is the ratio of milk and water in the resultant mixture?
A. 17 : 3
B. 9 : 1
C. 3 : 17
D. 5 : 3
Answer: B
193. In what ratio must a person mix three kinds of tea costing Rs.60/kg, Rs.75/kg and Rs.100 /kg so that the resultant mixture when sold at Rs.96/kg yields a profit of 20%?
A. 1 : 2 : 4
B. 3 : 7 : 6
C. 1 : 4 : 2
D. None of these
Answer: C
194. A merchant mixes three varieties of rice costing Rs.20/kg, Rs.24/kg and Rs.30/kg and sells the mixture at a profit of 20% at Rs.30 / kg. How many kgs of the second variety will be in the mixture if 2 kgs of the third variety is there in the mixture?
A. 1 kg
B. 5 kgs
C. 3 kgs
D. 6 kgs
Answer: B
195. How many litres of water should be added to a 30 litre mixture of milk and water containing milk and water in the ratio of 7 : 3 such that the resultant mixture has 40% water in it?
A. 7 litres
B. 10 litres
C. 5 litres
D. None of these
Answer: C
196. How many kgs of Basmati rice costing Rs.42/kg should a shopkeeper mix with 25 kgs of ordinary rice costing Rs.24 per kg so that he makes a profit of 25% on selling the mixture at Rs.40/kg?
A. 20 kgs
B. 12.5 kgs
C. 16 kgs
D. 200 kgs
Answer: A
197. Two trains, 200 and 160 meters long take a minute to cross each other while traveling in the same direction and take only 10 seconds when they cross in opposite directions. What are the speeds at which the trains are traveling?
A. 21 m/s; 15 m/s
B. 30 m/s; 24 m/s
C. 18 m/s; 27 m/s
D. 15 m/s; 24 m/s
Answer: A
198. A man driving his bike at 24 kmph reaches his office 5 minutes late. Had he driven 25% faster on an average he would have reached 4 minutes earlier than the scheduled time. How far is his office?
A. 24 km
B. 72 km
C. 18 km
D. Data Insufficient
Answer: C
199. When an object is dropped, the number of feet N that it falls is given by the formula N = ½gt2 where t is the time in seconds from the time it was dropped and g is 32.2. If it takes 5 seconds for the object to reach the ground, how many feet does it fall during the last 2 seconds?
A. 64.4
B. 96.6
C. 161.0
D. 257.6
Answer: D
200. If the wheel of a bicycle makes 560 revolutions in travelling 1.1 km, what is its radius?
A. 31.25 cm
B. 37.75 cm
C. 35.15 cm
D. 11.25 cm
Answer: A
Time and Work
201. Two workers A and B manufactured a batch of identical parts. A worked for 2 hours and B worked for 5 hours and they did half the job. Then they worked together for another 3 hours and they had to do (1/20)th of the job. How much time does B take to complete the job, if he worked alone?
A. 24 hours
B. 12 hours
C. 15 hours
D. 30 hours
Answer: C
202. The average wages of a worker during a fortnight comprising 15 consecutive working days was Rs.90 per day. During the first 7 days, his average wages was Rs.87/day and the average wages during the last 7 days was Rs.92 /day. What was his wage on the 8th day?
A. 83
B. 92
C. 90
D. 97
Answer: D
203. The average of 5 quantities is 6. The average of 3 of them is 8. What is the average of the remaining two numbers?
A. 6.5
B. 4
C. 3
D. 3.5
Answer: C
204. The average temperature on Wednesday, Thursday and Friday was 250. The average temperature on Thursday, Friday and Saturday was 240. If the temperature on Saturday was 270, what was the temperature on Wednesday?
A. 240
B. 210
C. 270
D. 300
Answer: D
205. The average age of a group of 12 students is 20years. If 4 more students join the group, the average age increases by 1 year. The average age of the new students is
A. 24
B. 26
C. 23
D. 22
Answer: A
206. When a student weighing 45 kgs left a class, the average weight of the remaining 59 students increased by 200g. What is the average weight of the remaining 59 students?
A. 57 kgs
B. 56.8 kgs
C. 58.2 kgs
D. 52.2 kgs
Answer: A
207. Pipe A can fill a tank in 'a' hours. On account of a leak at the bottom of the tank it takes thrice as long to fill the tank. How long will the leak at the bottom of the tank take to empty a full tank, when pipe A is kept closed?
A. (3/2)a hours
B. (2/3)a
C. (4/3)a
D. (3/4)a
Answer: A
208. A red light flashes 3 times per minute and a green light flashes 5 times in two minutes at regular intervals. If both lights start flashing at the same time, how many times do they flash together in each hour?
A. 30
B. 24
C. 20
D. 60
Answer: A
209. A and B can do a piece of work in 21 and 24 days respectively. They started the work together and after some days A leaves the work and B completes the remaining work in 9 days. After how many days did A leave?
A. 5
B. 7
C. 8
D. 6
Answer: B
210. Ram, who is half as efficient as Krish, will take 24 days to complete a work if he worked alone. If Ram and Krish worked together, how long will they take to complete the work?
A. 16 days
B. 12 days
C. 8 days
D. 18 days
Answer: C
Probability
211. A number is selected at random from first thirty natural numbers. What is the chance that it is a multiple of either 3 or 13?
A. 17/30
B. 2/5
C. 11/30
D. 4/15
Answer: B
212. What is the probability of getting at least one six in a single throw of three unbiased dice?
A. 1 / 6
B. 125 / 216
C. 1 / 36
D. 91 / 216
Answer: D
213. What is the probability that a two digit number selected at random will be a multiple of '3' and not a multiple of '5'?
A. 2 / 15
B. 4 / 15
C. 1 / 15
D. 4 / 90
Answer: B
214. A man bets on number 16 on a roulette wheel 14 times and losses each time. On the 15th span he does a quick calculation and finds out that the number 12 had appeared twice in the 14 spans and is therefore, unable to decide whether to bet on 16 or 12 in the 15th span. Which will give him the best chance and what are the odds of winning on the bet that he takes? (Roulette has numbers 1 to 36)
A. 16; 22 : 14
B. 12; 72 : 1
C. 12; 7 : 1
D. Either; 35 : 1
Answer: D
215. Two squares are chosen at random on a chessboard. What is the probability that they have a side in common?
A. 1 / 18
B. 64 / 4032
C. 63 / 64
D. 1 / 9
Answer: A
216. When two dice are thrown simultaneously, what is the probability that the sum of the two numbers that turn up is less than 11?
A. 5 / 6
B. 11 / 12
C. 1 / 6
D. 1 / 12
Answer: B
217. When 4 dice are thrown, what is the probability that the same number appears on each of them?
A. 1/36
B. 1/18
C. 1/216
D. 1/5
Answer: C
218. An experiment succeeds twice as often as it fails. What is the probability that in the next 5 trials there will be four successes?
A. 0
B. (2/3)^4
C. 5*((2/3)^4)*(1/3)
D. ((2/3)^4)*(1/3)
Answer: C
219. An anti aircraft gun can fire four shots at a time. If the probabilities of the first, second, third and the last shot hitting the enemy aircraft are 0.7, 0.6, 0.5 and 0.4, what is the probability that four shots aimed at an enemy aircraft will bring the aircraft down?
A. 0.084
B. 0.916
C. 0.036
D. 0.964
Answer: A
220. A can complete a project in 20 days and B can complete the same project in 30 days. If A and B start working on the project together and A quits 10 days before the project is completed, in how many days will the project be completed?
A. 18 days
B. 27 days
C. 26.67 days
D. 16 days
Answer: A
Miscellaneous
221. Which amongst the following investments has the lowest return?
a. 7% Rs.100 shares at Rs.120
b. 8% Rs.10 shares at 13.50
c. 9% Rs.50 shares at Rs.54
A. a
B. b
C. c
D. a and c
Answer: A
222. (1/4)3 + (3/4)3 + 3(1/4)(3/4)(1/4 + 3/4) =?
A. 1/64
B. 27/64
C. 49/64
D. 0
E. 1
Answer: E
223. If the area of two circles are in the ratio 169 : 196 then the ratio of their radii is
A. 10 : 11
B. 11 : 12
C. 12 : 13
D. 13 : 14
E. None of the above
Answer: D
224. A semi-circle is surmounted on the side of a square. The ratio of the area of the semi-circle to the area of the square is

A. 1 : 2
B. 2 : p
C. p : 8
D. 8 : p
E. None of the above
Answer: C
225. Which of the following is the greatest ?
A. 40% of 30
B. 3/5 of 25
C. 6.5% of 200
D. Five more than the square of 3
E. 1/2-4
Answer: E
226. Two identical taps fill 2/5 of a tank in 20 minutes. When one of the taps goes dry in how many minutes will the remaining one tap fill the rest of the tank ?
A. 5 minutes
B. 10 minutes
C. 15 minutes
D. 20 minutes
E. None of the above
Answer: C
227. From the following choices what is the equation of a line whose x intercept is half as that of the line 3x + 4y = 12 and y intercept is twice as that of the same line.
A. 3x + 8y = 24
B. 8x + 3y = 24
C. 16x + 3y = 24
D. 3x + y = 6
Answer: D
228. Pipe A can fill a tank completely in 5 hours. However, on account of a leak at the bottom of the tank, it takes 3 more hours to fill the tank. How long will the leak take to empty a full tank, when pipe A is shut?
A. 13 hours 20 minutes
B. 7.5 hours
C. 14 hours 40 minutes
D. None of these
Answer: A
229. The sum of the first 50 terms common to the series 15,19,23 ... and 14,19,24 ... is
A. 25450
B. 24550
C. 50900
D. Cannot be determined
Answer: A
230. For what values of 'x' will the function be defined in the real domain?
A. -10 < x < 4
B. 4 < x < 10
C. x does not lie between the closed interval 10 and 4
D. x does not lie between the open interval 4 and 10
Answer: D
231. How many different factors are there for the number 48, excluding 1 and 48?
A. 12
B. 4
C. 8
D. None of these
Answer: C
232. The sum of the first 100 numbers, 1 to 100 is divisible by
A. 2, 4 and 8
B. 2 and 4
C. 2 only
D. None of these
Answer: C
233. The sum of the first 100 numbers, 1 to 100 is divisible by
A. 2, 4 and 8
B. 2 and 4
C. 2 only
D. None of these
Answer: C
234. For what value of 'n' will the remainder of 351n and 352n be the same when divided by 7?
A. 2
B. 3
C. 6
D. 4
Answer: B
235. What will Rs.1500 amount to in three years if it is invested in 20% p.a. compound interest, interest being compounded annually?
A. 2400
B. 2592
C. 2678
D. 2540
Answer: B
236. If a sum of money grows to 144/121 times when invested for two years in a scheme where interest is compounded annually, how long will the same sum of money take to treble if invested at the same rate of interest in a scheme where interest is computed using simple interest method?
A. 9 years
B. 22 years
C. 18 years
D. 33 years
Answer: B
237. The population of a town was 3600 three years back. It is 4800 right now. What will be the population three years down the line, if the rate of growth of population has been constant over the years and has been compounding annually?
A. 6000
B. 6400
C. 7200
D. 9600
Answer: B
238. A man invests Rs.5000 for 3 years at 5% p.a. compound interest reckoned yearly. Income tax at the rate of 20% on the interest earned is deducted at the end of each year. Find the amount at the end of the third year.
A. 5624.32
B. 5630.50
C. 5788.125
D. 5627.20
Answer: A
239. A tank is fitted with 8 pipes, some of them that fill the tank and others that are waste pipe meant to empty the tank. Each of the pipes that fill the tank can fill it in 8 hours, while each of those that empty the tank can empty it in 6 hours. If all the pipes are kept open when the tank is full, it will take exactly 6 hours for the tank to empty. How many of these are fill pipes?
A. 2
B. 4
C. 6
D. 5
Answer: B
240. If A and B work together, they will complete a job in 7.5 days. However, if A works alone and completes half the job and then B takes over and completes the remaining half alone, they will be able to complete the job in 20 days. How long will B alone take to do the job if A is more efficient than B?
A. 20 days
B. 40 days
C. 30 days
D. 24 days
Answer: C
Profit and Loss
241. A trader makes a profit equal to the selling price of 75 articles when he sold 100 of the articles. What % profit did he make in the transaction?
A. 33.33%
B. 75%
C. 300%
D. 150%
Answer: C
242. A rectangle is 14 cm long and 10 cm wide. If the length is reduced by x cms and its width is increased also by x cms so as to make it a square then its area changes by :
A. 4
B. 144
C. 12
D. 2
E. None of the above.
Answer: A
243. A motorcycle stunts man belonging to a fair, rides over the vertical walls of a circular well at an average speed of 54 kph for 5 minutes. If the radius of the well is 5 meters then the distance traveled is:
A. 2.5 kms
B. 3.5 kms
C. 4.5 kms
D. 5.5 kms
E. None of the above
Answer: C
244. If 1 cm on a map corresponds to an actual distance of 40 kms. And the distance on the map between Bombay and Calcutta is 37.5 cms., the actual distance between them is :
A. 375 kms
B. 3750 kms
C. 1500 kms
D. 1375 kms
E. None of the above
Answer: C
245. A box contains 90 mts each of 100 gms and 100 bolts each of 150 gms. If the entire box weighs 35.5 kg., then the weight of the empty box is :
A. 10 kg
B. 10.5 kg
C. 11 kg
D. 11.5 kg
E. None of the above
Answer: D
246. If the radius of a circle is increased by 20% then the area is increased by :
A. 44%
B. 120%
C. 144%
D. 40%
E. None of the above
Answer: A
247. One year payment to the servant is Rs. 200 plus one shirt. The servant leaves after 9 months and recieves Rs. 120 and a shirt. Then find the price of the shirt.
A. Rs. 80
B. Rs. 100
C. Rs. 120
D. Cannot be determined
Answer: C
248. Two merchants sell, each an article for Rs.1000. If Merchant A computes his profit on cost price, while Merchant B computes his profit on selling price, they end up making profits of 25% respectively. By how much is the profit made by Merchant B greater than that of Merchant A?
A. Rs.66.67
B. Rs. 50
C. Rs.125
D. Rs.200
Answer: B
249. A merchant marks his goods in such a way that the profit on sale of 50 articles is equal to the selling price of 25 articles. What is his profit margin?
A. 25%
B. 50%
C. 100%
D. 66.67%
Answer: C
250. A merchant marks his goods up by 75% above his cost price. What is the maximum % Aount that he can offer so that he ends up selling at no profit or loss?
A. 75%
B. 46.67%
C. 300%
D. 42.85%
Answer: D
Quantitative Comparisons
Following are some SAT sample quantitative ability questions.
Directions:
In this section you will be given two quantities, one in column A and one in column B. You are to determine a relationship between the two quantities and mark.
A. If the quantity in column A is greater than the quantity in column B.
B. If the quantity in column B is greater than the quantity in column A.
C. If the quantities are equal.
D. If the comparison cannot be determined from the information that is given.
1. Quantity A: The average (arithmetic mean) of x and y
Quantity B: The average (arithmetic mean) of x - 1 and y + 1
A. The quantity in Column A is greater.
B. The quantity in Column B is greater.
C. The quantities are equal.
D. The relationship cannot be determined from the information given.
Answer: C
2. The integer (x - 1) is a prime number between 40 and 50.
Quantity A: The sum of all different prime factors of x
Quantity B: 14
A. The quantity in Column A is greater.
B. The quantity in Column B is greater.
C. The quantities are equal.
D. The relationship cannot be determined from the information given.
Answer: B
3. A < C
B > D > 0
Quantity A: A - B
Quantity B: C - D
A. The quantity in Column A is greater.
B. The quantity in Column B is greater.
C. The quantities are equal.
D. The relationship cannot be determined from the information given.
Answer: B
4. Quantity A: (9/13)2
Quantity B: (9/13)1/2
A. Quantity A equals Quantity B
B. Relationship Indeterminate
C. Quantity B is greater
D. Quantity A is greater
Answer: C
5. In a particular jellybean jar, the number of red jellybeans exceeds the number of white ones by a ratio of 3:2. If two red jellybeans were removed, the ratio of red to white jellybeans would be 1:1.
Quantity A: The number of white jellybeans in the jar
Quantity B: 4
A. The quantity in Column A is greater.
B. The quantity in Column B is greater.
C. The quantities are equal.
D. The relationship cannot be determined from the information given.
Answer: C
6. x + 2y > 8
Quantity A: 2x + 4y
Quantity B: 20
A. if the quantity in Column A is greater;
B. if the quantity in Column B is greater;
C. if the two quantities are equal;
D. if the relationship cannot be determined from the information given.
Answer: D
7. Quantity A: The number of months in 7 years
Quantity B: The number of days in 12 weeks
A. if the quantity in Column A is greater;
B. if the quantity in Column B is greater;
C. if the two quantities are equal;
D. if the relationship cannot be determined from the information given
Answer: C
8. Quantity A: 1-1/27
Quantity B: 8/9 + 1/81
A. if the quantity in is greater;
B. if the quantity in is greater;
C. if the two quantities are equal;
D. if the relationship cannot be determined from the information given.
Answer: A
9. r/>s/>0/>
Quantity A: rs/r
Quantity B: rs/s
A. if the quantity A is greater;
B. if the quantity B is greater;
C. if the two quantities are equal;
D. if the relationship cannot be determined from the information given.
Answer: B
10. Quantity A: 0.83
Quantity B: 0.81/3
A. Quantity B is greater
B. Relationship Indeterminate
C. Quantity A is greater
D. Quantity A equals Quantity B
Answer: A
11.
Diagram is illustrative and is not drawn to scale.
Quantity A: Measure of angle 3 - Measure of angle 2
Quantity B: Measure of angle 5 - Measure of angle 6
A. Relationship Indeterminate
B. Quantity A equals Quantity B
C. Quantity B is greater
D. Quantity A is greater
Answer: B
12. Quantity A: 29
Quantity B: 92
A. Quantity B is greater
B. Quantity A is greater
C. Relationship Indeterminate
D. Quantity A equals Quantity B
Answer: B
13. 0 < -x < 10
11 < -y < 20
Quantity A: x
Quantity B: y
A. Relationship Indeterminate
B. Quantity A equals Quantity B
C. Quantity B is greater
D. Quantity A is greater
Answer: D
14.
Diagram is illustrative and is not drawn to scale.
Given angles 1 and 2 are equal,
length of side AB = x, length of side BC = y, length of side AC = z.
Quantity A: x + y
Quantity B: y + z
A. Quantity A is greater
B. Quantity A equals Quantity B
C. Quantity B is greater
D. Relationship Indeterminate
Answer: B
15.
Diagram is illustrative and is not drawn to scale.
In triangle ABC, AB = AC and measure of angle 1 = 100o.
Quantity A: Measure of angle 2 + Measure of angle 3
Quantity B: 90o
A. Quantity B is greater
B. Relationship Indeterminate
C. Quantity A is greater
D. Quantity A equals Quantity B
Answer: A
16. t is a positive integer.
4/7 = t/s
Quantity A: s
Quantity B:7
A. if the quantity in Column A is greater;
B. if the quantity in Column B is greater;
C. if the two quantities are equal;
D. if the relationship cannot be determined from the information given
Answer: D
17. Quantity A: (0.82)2(0.82)3
Quantity B:(0.82)6
A. if the quantity in Column A is greater;
B. if the quantity in Column B is greater;
C. if the two quantities are equal;
D. if the relationship cannot be determined from the information given.
Answer: A
18. For all real numbers a, let a* = 1 - a.
Quantity A: ((-1)*)*
Quantity B: 2*
A. if the quantity in Column A is greater;
B. if the quantity in Column B is greater;
C. if the two quantities are equal;
D. if the relationship cannot be determined from the information given.
Answer: C
19. Quantity A: (x - 1)(x)(x + 1)
Quantity B:(x)(x)(x)
A. if the quantity in Column A is greater;
B. if the quantity in Column B is greater;
C. if the two quantities are equal;
D. if the relationship cannot be determined from the information given.
Answer: D
20. y2 + z2 = 34 and yz = 15
Quantity A: y2 + 2yz + z2
Quantity B: (y + z)2
A. Quantity B is greater
B. Relationship Indeterminate
C. Quantity A is greater
D. Quantity A equals Quantity B
Answer: D
21. Quantity A: (y + 5)2
Quantity B: (y - 5)2
A. Quantity A equals Quantity B
B. Quantity A is greater
C. Relationship Indeterminate
D. Quantity B is greater
Answer: C
22.
Diagram is illustrative and is not drawn to scale.
Quantity A: Measure of angle 1 + Measure of angle 3
Quantity B: Measure of angle 2 + Measure of angle 4
A. Relationship Indeterminate
B. Quantity A is greater
C. Quantity A equals Quantity B
D. Quantity B is greater
Answer: A
23.
Diagram is illustrative and is not drawn to scale.
In triangle ABC, AB = AC and measure of angle 1 = 100o.
Quantity A: Measure of angle 2 + Measure of angle 3
Quantity B: 90o
A. Quantity B is greater
B. Quantity A equals Quantity B
C. Relationship Indeterminate
D. Quantity A is greater
Answer: A
24.
Diagram is illustrative and is not drawn to scale.
Given angles 1 and 2 are equal,
length of side AB = x, length of side BC = y, length of side AC = z.
Quantity A: x + y
Quantity B: y + z
A. Quantity B is greater
B. Quantity A equals Quantity B
C. Quantity A is greater
D. Relationship Indeterminate
Answer: B
25. x and y are both positive and x/y > 5
Quantity A: 0.2x
Quantity B: y
A. Quantity B is greater
B. Relationship Indeterminate
C. Quantity A equals Quantity B
D. Quantity A is greater
Answer: D
26. yz < 0
Quantity A: (y - z)2
Quantity B: y2 + z2
A. Quantity A is greater
B. Quantity B is greater
C. Quantity A equals Quantity B
D. Relationship Indeterminate
Answer: A
27. Consider a rectangle. The length of its shorter side is 8, and the length of its diagonal is 16.
Quantity A: 30o
Quantity B: measure of angle formed by diagonal and shorter side
A. Quantity A is greater
B. Quantity A equals Quantity B
C. Quantity B is greater
D. Relationship Indeterminate
Answer: C
28. The sum of three consecutive even numbers is 18.
Quantity A: Their average
Quantity B: 6
A. Relationship Indeterminate
B. Quantity A is greater
C. Quantity A equals Quantity B
D. Quantity B is greater
Answer: C
29. x - y > 10
Quantity A: y - x
Quantity B: 12
A. Quantity B is greater
B. Quantity A is greater
C. Quantity A equals Quantity B
D. Relationship Indeterminate
Answer: A
30. x = 0, y > 0
Quantity A: xy
Quantity B: yx
A. Quantity A equals Quantity B
B. Quantity A is greater
C. Quantity B is greater
D. Relationship Indeterminate
Answer: C
31. x and y are both positive and x/y > 5
Quantity A: 0.2x
Quantity B: y
A. Quantity A is greater
B. Quantity B is greater
C. Relationship Indeterminate
D. Quantity A equals Quantity B
Answer: A
32. Quantity A: (3 x 4 x 17) / (121 x 100)
Quantity B: (4 x 5 x 19) / (1000 x 121)
A. Quantity A is greater
B. Quantity A equals Quantity B
C. Relationship Indeterminate
D. Quantity B is greater
Answer: A
33. Consider a triangle PQR.
Quantity A: length of PQ + length of QR
Quantity B: length of PR
A. Quantity A is greater
B. Quantity B is greater
C. Relationship Indeterminate
D. Quantity A equals Quantity B
Answer: A
34. Quantity A: (27 - 13) (296 + 534)
Quantity B: (27 + 13) (534 + 296)
A. Quantity B is greater
B. Quantity A equals Quantity B
C. Relationship Indeterminate
D. Quantity A is greater
Answer: D
35. Quantity A: A = 1.1
Quantity B: 12.11/2
A. Relationship Indeterminate
B. Quantity B is greater
C. Quantity A equals Quantity B
D. Quantity A is greater
Answer: B
36. 100 < y < 200 and 100 < z < 210
Quantity A: y
Quantity B: z
A. Quantity A is greater
B. Quantity A equals Quantity B
C. Quantity B is greater
D. Relationship Indeterminate
Answer: D
37.
Diagram is illustrative and is not drawn to scale.
Given AB = AC and angle BAC = 60o
Quantity A: Length of side AB
Quantity B: Length of side BC
A. Quantity A equals Quantity B
B. Quantity B is greater
C. Relationship Indeterminate
D. Quantity A is greater
Answer: A
38. y2 = 36
Quantity A: y
Quantity B: 6
A. Relationship Indeterminate
B. Quantity A is greater
C. Quantity B is greater
D. Quantity A equals Quantity B
Answer: A
39.
Diagram is illustrative and is not drawn to scale.
Quantity A: Measure of angle 1 + Measure of angle 2 + Measure of angle 4
Quantity B: 180o
A. Relationship Indeterminate
B. Quantity A is greater
C. Quantity B is greater
D. Quantity A equals Quantity B
Answer: D
40.
Diagram is illustrative and is not drawn to scale.
In triangle ABC, angle A = 60o and AB = AC.
Quantity A: Measure of angle 1 + Measure of angle 2
Quantity B: 120o
A. Relationship Indeterminate
B. Quantity A is greater
C. Quantity A equals Quantity B
D. Quantity B is greater
Answer: C
41.
Diagram is illustrative and is not drawn to scale.
Quantity A: Measure of angle 2 + Measure of angle 3
Quantity B: 180o
A. Quantity B is greater
B. Quantity A is greater
C. Quantity A equals Quantity B
D. Relationship Indeterminate
Answer: C
42. Consider a rectangle. The length of its shorter side is 8, and the length of its diagonal is 16.
Quantity A: 30o
Quantity B: measure of angle formed by diagonal and shorter side
A. Relationship Indeterminate
B. Quantity A equals Quantity B
C. Quantity A is greater
D. Quantity B is greater
Answer: D
43. Quantity A: (y + 5)2
Quantity B: (y - 5)2
A. Quantity B is greater
B. Relationship Indeterminate
C. Quantity A equals Quantity B
D. Quantity A is greater
Answer: B
44. Quantity A: (1/25)1/2 + (1/144)1/2
Quantity B: [(1/25) + (1/144)]1/2
A. Relationship Indeterminate
B. Quantity A is greater
C. Quantity B is greater
D. Quantity A equals Quantity B
Answer: A
45. y2 + z2 = 34 and yz = 15
Quantity A: y2 + 2yz + z2
Quantity B: (y + z)2
A. Quantity A is greater
B. Relationship Indeterminate
C. Quantity A equals Quantity B
D. Quantity B is greater
Answer: C
46. 100 < y < 200 and 100 < z < 210
Quantity A: y
Quantity B: z
A. Quantity A is greater
B. Quantity A equals Quantity B
C. Quantity B is greater
D. Relationship Indeterminate
Answer: D
47.
Diagram is illustrative and is not drawn to scale.
AB is the diameter of the circle.
Quantity A: Measure of angle 1
Quantity B: Measure of angle 2
A. Relationship Indeterminate
B. Quantity A is greater
C. Quantity B is greater
D. Quantity A equals Quantity B
Answer: D
48. For any positive integer n,
n! is the product of all positive integers less than or equal to n.
Quantity A: 20! / 17!
Quantity B: 80! / 78!
A. Quantity A is greater
B. Quantity B is greater
C. Quantity A equals Quantity B
D. Relationship Indeterminate
Answer: A
49. 2 < z < 4
Quantity A: π2z3
Quantity B: π3z2
A. Quantity A is greater
B. Quantity B is greater
C. Quantity A equals Quantity B
D. Relationship Indeterminate
Answer: D
50. Amy, Beth and Charlie divided a pizza amongst themselves.
Amy took 30% of the pizza and ate (3/4) of what she took.
Beth took 20% of the pizza.
Charlie ate (2/5) of what he took.
Quantity A: The amount Amy ate
Quantity B: The amount Charlie ate
A. Quantity A is greater
B. Quantity B is greater
C. Quantity A equals Quantity B
D. Relationship Indeterminate
Answer: A
51. Quantity A: (-6)4
Quantity B: (-6)5
A. if the quantity A is greater;
B. if the quantity B is greater;
C. if the two quantities are equal;
D. if the relationship cannot be determined from the information given.
Answer: A
52. Quantity A: Time to travel 95 miles at 50 miles per hour
Quantity B: Time to travel 125 miles at 60 miles per hour
A. Quantity A is greater
B. Quantity A equals Quantity B
C. Quantity B is greater
D. Relationship Indeterminate
Answer: C
53. Quantity A: 4 / 100
Quantity B: 0.012 / 3
A. Quantity B is greater
B. Quantity A equals Quantity B
C. Quantity A is greater
D. Relationship Indeterminate
Answer: C
54. x = 2y + 3
y = -2
Quantity A: x
Quantity B: -1
A. if the quantity in Column A is greater;
B. if the quantity in Column B is greater;
C. if the two quantities are equal;
D. if the relationship cannot be determined from the information given
Answer: C
55. p > 0 > q
Quantity A: p + q
Quantity B: pq
A. The quantity in Column A is greater.
B. The quantity in Column B is greater.
C. The quantities are equal.
D. The relationship cannot be determined from the information given.
Answer: D

No comments: